Annotation of db/baza/izmi1617.txt, revision 1.4

1.1       rubashki    1: Чемпионат:
                      2: Синхронный турнир "Из Минска с любовью" - 2016/17
                      3: 
                      4: Дата:
                      5: 08-Oct-2016 - 15-Apr-2017
                      6: 
                      7: Тур:
                      8: 1 этап
                      9: 
                     10: Дата:
                     11: 08-Oct-2016
                     12: 
                     13: Редактор:
                     14: 1-18 - Ксения Езерская (Осиповичи - Барановичи - Минск) и Евгений
                     15: Лешкович (Минск); 19-36 - Дмитрий Буценец (Пинск - Минск - Брест)
                     16: 
                     17: Инфо:
                     18: Ксения Езерская и Евгений Лешкович благодарят за тестирование вопросов и
                     19: ценные замечания Ивана Топчия, Юрия Разумова, Сергея Дубелевича, Даниила
                     20: Шункевича, Александра Шустера, Владислава Латынского, Дмитрия Буценца,
                     21: Андрея Танану, Анну Якушевич, Ивана Зайкова, Юрия Ананича, а также
                     22: команды "Ultima Ratio", "Лунь" и "На заре". Дмитрий Буценец благодарит
                     23: за тестирование и помощь в подготовке пакета: Юрия Разумова, Артема
                     24: Гулецкого, Марию Кленицкую, Александра Матюхина, Евгения Миротина, Ивана
                     25: Зайкова, Сергея Дубелевича, Евгения Лешковича, Анастасию Балмакову,
                     26: Дарью Соловей, Александру Ермалович, Михаила Карпука, а также команды
                     27: "Нехорошая квартира" и "На заре" (все - Минск).
                     28: 
                     29: Вопрос 1:
                     30: Футбольный тренер Луи ван Гал уделяет большое внимание созданию
                     31: комфортной атмосферы в команде. Возглавив "Манчестер Юнайтед", ван Гал
                     32: первым делом распорядился поместить в клубной столовой ЕГО. Более
                     33: известный ОН, по мнению историка Криса Гидлоу, располагался в 67
                     34: километрах от Манчестера. Назовите ЕГО двумя словами.
                     35: 
                     36: Ответ:
                     37: Круглый стол.
                     38: 
                     39: Комментарий:
                     40: Ван Гал распорядился поставить в столовой круглый стол, чтобы все
                     41: футболисты чувствовали себя на равных. Круглый стол короля Артура, как
                     42: утверждает Крис Гидлоу, располагался в городке Честер, расположенном в
                     43: 66 километрах от Манчестера. Редакторы желают, чтобы комфортная
                     44: атмосфера сохранялась в вашей команде на протяжении всего турнира!
                     45: 
                     46: Источник:
                     47:    1. М. Мейер. Луи ван Гал: биография. http://flibusta.is/b/454715/read
                     48:    2. https://www.newsru.com/world/12jul2010/roundtable.html
                     49:    3. https://www.distancecalculator.net/from-chester-to-manchester
                     50: 
                     51: Автор:
                     52: Евгений Лешкович (Минск)
                     53: 
                     54: Вопрос 2:
                     55: Министр внутренних дел Франции Мишель Альо-Мари предложила составить
                     56: базу данных со всей информацией о гражданах страны: отпечатки пальцев,
                     57: политические предпочтения, история семьи. За это журналисты присудили
                     58: Альо-Мари неофициальную премию ЕГО. Назовите ЕГО двумя словами,
                     59: начинающимися на одну и ту же букву.
                     60: 
                     61: Ответ:
                     62: Большой брат.
                     63: 
                     64: Комментарий:
                     65: Премия Большого брата ежегодно вручается лицам, "отличившимся" в
                     66: наиболее грубых нарушениях неприкосновенности свободы и частной жизни.
                     67: 
                     68: Источник:
                     69: Дж. Кампфнер. Свобода на продажу. Как мы разбогатели - и лишились
                     70: независимости. http://flibusta.is/b/459855/read
                     71: 
                     72: Автор:
                     73: Евгений Лешкович (Минск)
                     74: 
                     75: Вопрос 3:
                     76: [Ведущему: четко прочитать название статьи.]
                     77:    Статья "Швейцарский нож самочки" рассказывает, что на самом деле ОН
                     78: состоит из нескольких игл, насоса и распорок. Назовите ЕГО двумя
                     79: словами.
                     80: 
                     81: Ответ:
                     82: Хоботок комара.
                     83: 
                     84: Зачет:
                     85: Жало комара; комариный хоботок; комариное жало.
                     86: 
                     87: Комментарий:
                     88: Кровь пьют именно самки комаров. Хоботок на самом деле состоит из шести
                     89: частей. На концах двух из них есть мелкие зубья, которыми он разрезает
                     90: кожу. Через другую иглу комар вводит в кровь антикоагулянты, которые не
                     91: дают крови свернуться и облегчают сосание. Еще одна игла - гибкий насос,
                     92: через который происходит перекачка крови. А две другие иглы служат
                     93: распорками и не дают ране закрыться.
                     94: 
                     95: Источник:
                     96: https://mi3ch.livejournal.com/3394606.html
                     97: 
                     98: Автор:
                     99: Илья Колодник (Минск)
                    100: 
                    101: Вопрос 4:
                    102: Автор статьи о Цутому Ямагути, выжившем после двух атомных
                    103: бомбардировок, логично называет его "ИКСОМ". На рекламном плакате
                    104: осиповичской службы такси "ИКС" можно увидеть подкову. Какое слово мы
                    105: заменили ИКСОМ?
                    106: 
                    107: Ответ:
                    108: Везунчик.
                    109: 
                    110: Комментарий:
                    111: Цутому Ямагути побывал и в Хиросиме, и в Нагасаки - настоящий везунчик.
                    112: Подкова - традиционный символ удачи. В названии службы такси "Везунчик"
                    113: подкова заменяет букву У.
                    114: 
                    115: Источник:
                    116:    1. https://razdumyvatel.livejournal.com/12296.html
                    117:    2. https://www.youtube.com/watch?v=ggwPHk3AoiA
                    118: 
                    119: Автор:
                    120: Евгений Лешкович (Минск), Ксения Езерская (Осиповичи - Барановичи -
                    121: Минск)
                    122: 
                    123: Вопрос 5:
                    124: Журналист Алексей Беляков написал письмо своему другу. В нем он
                    125: сообщает, что жизнь в Москве очень живая и разнообразная: для кого-то
                    126: театральная, а для кого-то спортивная, для кого-то академическая, а для
                    127: кого-то и пролетарская. В одном из следующих абзацев он рассказывает о
                    128: девушке. Каким эпитетом он ее описывает?
                    129: 
                    130: Ответ:
                    131: Тургеневская.
                    132: 
                    133: Зачет:
                    134: Чертановская (в заметке девушка называется и так).
                    135: 
                    136: Комментарий:
                    137: Вся заметка построена на обыгрывании названий станций Московского
                    138: метрополитена. "Театральная", "Спортивная", "Академическая",
                    139: "Пролетарская" и "Тургеневская" - это всё станции метро. "Тургеневская
                    140: барышня" - устойчивое выражение, описывающее женские манеры. Ночью
                    141: девушка автора бывает "той еще Чертановской".
                    142: 
                    143: Источник:
                    144: https://www.facebook.com/permalink.php?story_fbid=1090313124358859&id=100001405462980
                    145: 
                    146: Автор:
                    147: Павел Кебец (Барселона)
                    148: 
                    149: Вопрос 6:
                    150: Одной из достопримечательностей Яффы является плодоносящее апельсиновое
                    151: дерево. По мнению ведущих передачи "Орел и решка", оно символизирует
                    152: израильский народ, сумевший выжить вдали от родины. Назовите вымышленный
                    153: остров, обладавший той же особенностью, что и это дерево.
                    154: 
                    155: Ответ:
                    156: Лапута.
                    157: 
                    158: Комментарий:
                    159: Это дерево парит в воздухе и не касается земли. Остров Лапута из
                    160: "Путешествий Гулливера" также парил в воздухе. Летающий остров Лапута
                    161: присутствует в полнометражном аниме-фильме Хаяо Миядзаки "Небесный замок
                    162: Лапута".
                    163: 
                    164: Источник:
                    165:    1. "Орел и решка". 1 сезон, 7 серия. Израиль.
                    166: https://www.youtube.com/watch?v=VENHncG84Eo
                    167:    2. https://ru.wikipedia.org/wiki/Лапута
                    168: 
                    169: Автор:
                    170: Артур Астапенко (Минск)
                    171: 
                    172: Вопрос 7:
                    173: Дуплет.
                    174:    В одном из эпизодов мультсериала "Покемон" герои попадают в город,
                    175: где не любят тренеров покемонов. В ресторане к ним подходит женщина и
                    176: пытается выяснить их имена. Желая сохранить свою репутацию, герои
                    177: поспешно придумывают себе псевдонимы.
                    178:    1. Эш Кетчум представился как Том. Какую фамилию он себе придумал?
                    179:    2. Тренер водных покемонов Мисти сказала, что ее зовут Анн. Какую
                    180: фамилию она для себя придумала?
                    181: 
                    182: Ответ:
                    183:    1. Ато.
                    184:    2. Чоус.
                    185: 
                    186: Зачет:
                    187: Всё, что напоминает правильные ответы и соответствует логике
                    188: комментария/мультфильма.
                    189: 
                    190: Комментарий:
                    191: В критической ситуации ребята быстро посмотрели, что есть вокруг. А
                    192: поскольку они были в ресторане, то и образовали свои псевдонимы от
                    193: продуктов питания.
                    194: 
                    195: Источник:
                    196: Мультсериал "Покемон: Лига Индиго", серия "Тайна Мрачного города".
                    197: 
                    198: Автор:
                    199: Павел Кебец (Барселона)
                    200: 
                    201: Вопрос 8:
                    202: В одном из эпизодов мультсериала "Человек-паук" главный герой ведет в
                    203: Москве суровый бой с шестью суперзлодеями. Один из них предлагает
                    204: сыграть в ТАКОЙ ИКС, отводя Человеку-пауку роль АЛЬФЫ. Примечательно,
                    205: что обычно в ТАКОМ ИКСЕ используют не АЛЬФУ, да и для комплекта
                    206: необходимо почти вдвое больше суперзлодеев. Что мы заменили словами
                    207: "ТАКОЙ ИКС"?
                    208: 
                    209: Ответ:
                    210: Русский хоккей.
                    211: 
                    212: Зачет:
                    213: Бенди; хоккей с мячом.
                    214: 
                    215: Комментарий:
                    216: ТАКОЙ ИКС - русский хоккей, АЛЬФА - шайба. В русский хоккей, он же
                    217: бенди, он же хоккей с мячом, играют мячом. В команде по хоккею с шайбой
                    218: играет шесть человек, в русском хоккее - одиннадцать. Слова "ведет
                    219: суровый бой" являются намеком на песню о том, как "суровый бой ведет
                    220: ледовая дружина".
                    221: 
                    222: Источник:
                    223:    1. Мультсериал "Человек-Паук" (1994-1997), 55-я серия "Шесть забытых
                    224: воинов 2: Невостребованное наследие", примерно 9-я - 10-я минуты.
                    225:    2. https://ru.wikipedia.org/wiki/Хоккей_с_мячом
                    226: 
                    227: Автор:
                    228: Павел Кебец (Барселона)
                    229: 
                    230: Вопрос 9:
                    231: Программы распознавания изображений увидели в ИКСАХ "чудовище", "птицу,
                    232: летящую над часами", "ракушку" и даже "орхидею". ИКСЫ героя
                    233: экранизированного комикса появлялись из двух жидкостей, помещенных между
                    234: слоями латекса. Какие два слова, начинающиеся на соседние буквы, мы
                    235: заменили ИКСАМИ?
                    236: 
                    237: Ответ:
                    238: Пятна Роршаха.
                    239: 
                    240: Источник:
                    241:    1. https://geektimes.ru/post/276980/
                    242:    2. https://ru.wikipedia.org/wiki/Роршах_(Хранители)
                    243: 
                    244: Автор:
                    245: Илья Колодник (Минск)
                    246: 
                    247: Вопрос 10:
                    248: В 1980-е годы финский футбольный клуб "Куусюси" славился надежной игрой
                    249: в защите. Какая фамилия фигурировала в прозвище, которое дали этому
                    250: клубу журналисты?
                    251: 
                    252: Ответ:
                    253: Маннергейм.
                    254: 
                    255: Комментарий:
                    256: Журналисты сравнивали непроходимую оборону этой команды со знаменитой
                    257: линией Маннергейма.
                    258: 
                    259: Источник:
                    260: https://www.sports.ru/tribuna/blogs/footballeshkovich/411593.html
                    261: 
                    262: Автор:
                    263: Евгений Лешкович (Минск)
                    264: 
                    265: Вопрос 11:
                    266: Рассказывая о сцене рисования, Ольга Суркова упоминает обгоревшую
                    267: головку космического корабля и алюминиевый пол. Первое рисование
                    268: совершила экспедиция Шеннона. Какие семь букв мы дважды пропустили в
                    269: этом вопросе?
                    270: 
                    271: Ответ:
                    272: присоля.
                    273: 
                    274: Комментарий:
                    275: Так по аналогии с приземлением назвали сцену посадки на Солярис члены
                    276: съемочной группы фильма Андрея Тарковского.
                    277: 
                    278: Источник:
                    279:    1. http://kinoart.ru/archive/2002/04/n4-article16
                    280:    2. С. Лем. Солярис. http://flibusta.is/b/232728/read
                    281: 
                    282: Автор:
                    283: Александр Шустер (Минск)
                    284: 
                    285: Вопрос 12:
                    286: В 1715 году Петр I издал указ, согласно которому любой крепостной,
                    287: совершивший донос на своего барина, получал вольную. По оригинальной
                    288: версии журналистки Лилии Гущиной, именно тогда русские дворяне начали
                    289: ДЕЛАТЬ ЭТО. Мать Наполеона Бонапарта ДЕЛАТЬ ЭТО так и не научилась. Что
                    290: мы заменили словами "ДЕЛАТЬ ЭТО"?
                    291: 
                    292: Ответ:
                    293: Говорить по-французски.
                    294: 
                    295: Зачет:
                    296: По словам "говорить" и "французский".
                    297: 
                    298: Комментарий:
                    299: Как нетрудно догадаться, после петровского указа по России прокатилась
                    300: волна доносов. Чтобы успешнее "шифроваться" от слуг, русские дворяне
                    301: стали говорить по-французски. Мать Наполеона Бонапарта была корсиканской
                    302: националисткой и до конца жизни так и не выучилась говорить
                    303: по-французски.
                    304: 
                    305: Источник:
                    306:    1. Журнал "Story", 1994, N 8. - С. 8.
                    307:    2. https://ru.wikipedia.org/wiki/Рамолино,_Мария_Летиция
                    308: 
                    309: Автор:
                    310: Евгений Лешкович (Минск)
                    311: 
                    312: Вопрос 13:
                    313: Поэт и живописец XVI века Карел ван Мандер написал биографии многих
                    314: современников: Рогира ван дер Вейдена, Иеронима Босха, Питера
                    315: Брейгеля-старшего. Искусствовед Паола Волкова называет ван Мандера ТАКИМ
                    316: ИМ. Какие два слова, которые начинаются на соседние буквы алфавита, мы
                    317: заменили словами "ТАКОЙ ОН"?
                    318: 
                    319: Ответ:
                    320: Голландский Вазари.
                    321: 
                    322: Комментарий:
                    323: Карел ван Мандер сделал то же, что и его куда более известный коллега
                    324: Джорджо Вазари - автор "Жизнеописаний прославленных живописцев,
                    325: скульпторов и архитекторов".
                    326: 
                    327: Источник:
                    328:    1. https://ru.wikipedia.org/wiki/Мандер,_Карел_ван
                    329:    2. П.Д. Волкова. Мост через бездну. Мистики и гуманисты.
                    330: http://flibusta.is/b/454040/read
                    331: 
                    332: Автор:
                    333: Евгений Лешкович (Минск)
                    334: 
                    335: Вопрос 14:
                    336:    <раздатка>
                    337:    Deep View
                    338:    </раздатка>
                    339:    Из-за сложного рельефа Фарерские острова очень долго не были
                    340: представлены в проекте Google Street View. Как правило, панорамная
                    341: съемка в рамках этого проекта проводится с помощью автомобилей, но на
                    342: Фарерах это технически невозможно. Решение нашли местные жители, которые
                    343: организовали кампанию "Deep View". Какие две буквы в этом вопросе мы
                    344: заменили одной?
                    345: 
                    346: Ответ:
                    347: Sh.
                    348: 
                    349: Комментарий:
                    350: Фарерцы догадались приделывать камеры к овцам, которых на островах в два
                    351: раза больше, чем людей. Собственно, само название "Фареры" означает
                    352: "Овечьи острова".
                    353: 
                    354: Источник:
                    355: Журнал "GEO", 2016, N 9. - С. 26.
                    356: 
                    357: Автор:
                    358: Евгений Лешкович (Минск)
                    359: 
                    360: Вопрос 15:
                    361: Джон Кампфнер пишет, что на время Олимпиады-2008 в пекинских киосках
                    362: прекратилась продажа пива, вина и джина. В объяснении причин такого
                    363: запрета упоминается фамилия уроженца Вятской губернии. Назовите эту
                    364: фамилию.
                    365: 
                    366: Ответ:
                    367: Молотов.
                    368: 
                    369: Комментарий:
                    370: Официальный Пекин боялся, как бы во время Игр не случилось демонстраций
                    371: и уличных столкновений, поэтому отказался от продажи напитков в
                    372: стеклянной таре. Это позволило не бояться так называемых "коктейлей
                    373: Молотова". Вячеслав Молотов родился в слободе Кукарка Вятской губернии.
                    374: 
                    375: Источник:
                    376:    1. Дж. Кампфнер. Свобода на продажу. Как мы разбогатели - и лишились
                    377: независимости. http://flibusta.is/b/459855/read
                    378:    2. https://ru.wikipedia.org/wiki/Молотов,_Вячеслав_Михайлович
                    379: 
                    380: Автор:
                    381: Евгений Лешкович (Минск)
                    382: 
                    383: Вопрос 16:
                    384: Герой Тома Хэнкса неудачно провел презентацию в Саудовской Аравии. Его
                    385: дочь шутит, что за это его ОТЧИТАЛИ. Какое слово мы заменили словом
                    386: "ОТЧИТАЛИ"?
                    387: 
                    388: Ответ:
                    389: Пропесочили.
                    390: 
                    391: Комментарий:
                    392: Королевство Саудовская Аравия, как известно, находится в пустыне.
                    393: 
                    394: Источник:
                    395: Х/ф "Голограмма для короля" (2016), реж. Том Тыквер.
                    396: 
                    397: Автор:
                    398: Александр Шустер (Минск)
                    399: 
                    400: Вопрос 17:
                    401: После успешного клонирования овечки Долли генетик Артур Каплан получил
                    402: несколько одинаковых предложений, на которые ответил: "Вы хотите помочь
                    403: единственному, кто может отлично справиться без нас!". Какой город
                    404: упоминался во всех этих предложениях?
                    405: 
                    406: Ответ:
                    407: Турин.
                    408: 
                    409: Комментарий:
                    410: Религиозные фанатики зажглись идеей клонировать Христа. По мнению
                    411: Каплана, это глупо, потому что Иисус, если верить Новому завету, без
                    412: проблем может воскреснуть и без помощи генетиков. По канонической
                    413: версии, ДНК Христа содержится на туринской плащанице.
                    414: 
                    415: Источник:
                    416: С. Кин. Синдром Паганини и другие правдивые истории о гениальности,
                    417: записанные в нашем генетическом коде. http://flibusta.is/b/426306/read
                    418: 
                    419: Автор:
                    420: Евгений Лешкович (Минск)
                    421: 
                    422: Вопрос 18:
                    423: В 2014 году белорусский программист Геннадий Короткевич победил в
                    424: нескольких престижных турнирах по спортивному программированию: Google
                    425: Code Jam, Facebook Hacker Cup, TopCoder Open и Яндекс: Алгоритм.
                    426: Согласно шутке одного журналиста, Геннадий выиграл программистский ИКС.
                    427: Какие два слова мы заменили ИКСОМ?
                    428: 
                    429: Ответ:
                    430: Большой шлем.
                    431: 
                    432: Комментарий:
                    433: Прямая аналогия с теннисом, где Большой шлем можно получить, выиграв
                    434: четыре крупнейших турнира.
                    435: 
                    436: Источник:
                    437:    1. http://www.russoft.ru/tops/2157
                    438:    2. https://ru.wikipedia.org/wiki/Турниры_Большого_шлема_(теннис)
                    439: 
                    440: Автор:
                    441: Павел Кебец (Барселона)
                    442: 
                    443: Вопрос 19:
                    444: Романо Муссолини вспоминал, что его отец рассчитывал с небольшим
                    445: количеством людей добраться до Вальтеллины в Альпах и сопротивляться там
                    446: до последнего. Бенито Муссолини сравнивал Вальтеллину с НИМИ. Назовите
                    447: ИХ.
                    448: 
                    449: Ответ:
                    450: Фермопилы.
                    451: 
                    452: Комментарий:
                    453: История напоминает ситуацию во время сражения в Фермопилах и историю о
                    454: трехстах спартанцах.
                    455: 
                    456: Источник:
                    457: Р. Муссолини. Дуче, мой отец.
                    458: 
                    459: Автор:
                    460: Никита Шевела (Минск)
                    461: 
                    462: Вопрос 20:
                    463: Персонажи Самюэля Беккета замечают, что ОНА не плачет. После этого один
                    464: из них предполагает, что ОНА умерла. Назовите ЕЕ коротким словом.
                    465: 
                    466: Ответ:
                    467: Ива.
                    468: 
                    469: Комментарий:
                    470: Плакучая.
                    471: 
                    472: Источник:
                    473: С. Беккет. В ожидании Годо.
                    474: https://books.google.ru/books?id=B91TKeLQ54EC&pg=PT30#v=onepage&q&f=false
                    475: 
                    476: Автор:
                    477: Дмитрий Буценец (Пинск - Минск - Брест)
                    478: 
                    479: Вопрос 21:
                    480: Джордж Байрон, описывая сражение, пишет, что на месте погибших солдат
                    481: тут же появлялись другие. Ответьте одним словом: с кем поэт сравнил
                    482: двигающиеся по берегу полкИ?
                    483: 
                    484: Ответ:
                    485: Гидра.
                    486: 
                    487: Комментарий:
                    488: Гидра - змееподобное чудовище из греческой мифологии, чье имя можно
                    489: перевести как "водяная змея". Байрон по этому поводу писал: "Пускай
                    490: героев головы слетают - // Немедленно другие вырастают".
                    491: 
                    492: Источник:
                    493: Дж. Байрон. Дон Жуан. http://flibusta.is/b/424289/read
                    494: 
                    495: Автор:
                    496: Дмитрий Буценец (Пинск - Минск - Брест)
                    497: 
                    498: Вопрос 22:
                    499: Тэнгу - существо из японских поверий - летает при помощи НЕГО. Персонажи
                    500: одного романа использовали ЕГО, чтобы переместить обездвиженного
                    501: призрака. Назовите ЕГО одним словом.
                    502: 
                    503: Ответ:
                    504: Веер.
                    505: 
                    506: Комментарий:
                    507: Несмотря на то что у тэнгу есть крылья, летает он именно с помощью
                    508: веера. Веера - неотъемлемая часть японской культуры. Для перемещения
                    509: бесплотного призрака герои Роулинг использовали ветер от огромного
                    510: веера.
                    511: 
                    512: Источник:
                    513:    1. https://ru.wikipedia.org/wiki/Тэнгу
                    514:    2. Дж.К. Роулинг. Гарри Поттер и Тайная комната (перевод М.Д.
                    515: Литвиновой). http://flibusta.is/b/265596/read
                    516: 
                    517: Автор:
                    518: Дмитрий Буценец (Пинск - Минск - Брест)
                    519: 
                    520: Вопрос 23:
                    521: Артем Заяц рассказывает, как при показе "Прибытия поезда" люди в ужасе
                    522: выбегали из зала, и отмечает, что этот поезд СДЕЛАЛ ЭТО. Ответьте тремя
                    523: словами: что сделал?
                    524: 
                    525: Ответ:
                    526: Сломал четвертую стену.
                    527: 
                    528: Зачет:
                    529: Разрушил четвертую стену и т.п.
                    530: 
                    531: Комментарий:
                    532: Разрушение четвертой стены - это прямое обращение героя художественного
                    533: произведения к автору или аудитории. Фильм "Прибытие поезда" заставил
                    534: многих зрителей в ужасе выбежать из зала.
                    535: 
                    536: Источник:
                    537: https://www.film.ru/articles/kinoslovar-chetvertaya-stena
                    538: 
                    539: Автор:
                    540: Дмитрий Буценец (Пинск - Минск - Брест)
                    541: 
                    542: Вопрос 24:
                    543: В цикле "Темная башня" Стивен Кинг неоднократно ломает четвертую стену.
                    544: Персонажи цикла замечают, что порой спасались в последний момент, и
                    545: сравнивают писателя с НИМ. Назовите ЕГО тремя словами.
                    546: 
                    547: Ответ:
                    548: Бог из машины.
                    549: 
                    550: Зачет:
                    551: Deus ex machina.
                    552: 
                    553: Комментарий:
                    554: "Бог из машины" - выражение, означающее неожиданную, нарочитую развязку
                    555: той или иной ситуации, с привлечением внешнего, ранее не действовавшего
                    556: в ней фактора.
                    557: 
                    558: Источник:
                    559: С. Кинг. Темная башня.
                    560: https://books.google.ru/books?id=dcy5AAAAQBAJ&pg=PT167#v=onepage&q&f=false
                    561: 
                    562: Автор:
                    563: Дмитрий Буценец (Пинск - Минск - Брест)
                    564: 
                    565: Вопрос 25:
                    566: Персидская легенда называет деревню около ЭТОГО ОБЪЕКТА колыбелью
                    567: человечества. В курдском названии ЭТОГО ОБЪЕКТА есть слово "огненная".
                    568: Под каким названием ЭТОТ ОБЪЕКТ известен нам?
                    569: 
                    570: Ответ:
                    571: Арарат.
                    572: 
                    573: Комментарий:
                    574: Курдское название горы свидетельствует о ее вулканическом происхождении.
                    575: Согласно персидской легенде, считалось, что деревня Аргури на склоне
                    576: Арарата находится на месте, где Ной построил алтарь и посадил первый
                    577: виноградник.
                    578: 
                    579: Источник:
                    580:    1. https://www.britannica.com/place/Mount-Ararat
                    581:    2. https://ru.wikipedia.org/wiki/Арарат
                    582: 
                    583: Автор:
                    584: Дарья Данилевич (Минск), Дмитрий Буценец (Пинск - Минск - Брест)
                    585: 
                    586: Вопрос 26:
                    587: Персонаж фильма "Приключение" говорит, что людьми, которые ежегодно
                    588: пропадают в Италии, можно заполнить ЕГО. В оформлении музея при НЕМ
                    589: преобладают красный, черный и синий цвета. Назовите ЕГО.
                    590: 
                    591: Ответ:
                    592: [Стадион] "Сан-Сиро".
                    593: 
                    594: Зачет:
                    595: [Стадион] "Джузеппе Меацца".
                    596: 
                    597: Комментарий:
                    598: Персонаж фильма говорит, что людьми можно заполнить стадион Сан-Сиро.
                    599: Стадион является домашним для двух клубов - "Милана" и "Интера". Их
                    600: цвета - красно-черные и сине-черные соответственно.
                    601: 
                    602: Источник:
                    603:    1. Х/ф "Приключение" (1960), реж. Микеланджело Антониони.
                    604:    2. https://ru-travel.livejournal.com/13418232.html
                    605: 
                    606: Автор:
                    607: Дмитрий Буценец (Пинск - Минск - Брест)
                    608: 
                    609: Вопрос 27:
                    610: Описывая ЭТОТ ПРОЦЕСС, высадившийся на берег персонаж Милорада Павича
                    611: образно говорит, что рыбьи кости постепенно обрастали мясом. Назовите
                    612: ЭТОТ ПРОЦЕСС.
                    613: 
                    614: Ответ:
                    615: Постройка корабля.
                    616: 
                    617: Зачет:
                    618: Синонимичные ответы.
                    619: 
                    620: Источник:
                    621: М. Павич. Пейзаж, нарисованный чаем. http://flibusta.is/b/41905/read
                    622: 
                    623: Автор:
                    624: Дмитрий Буценец (Пинск - Минск - Брест)
                    625: 
                    626: Вопрос 28:
                    627: Название древнего города Нан-МадОл, расположенного в Микронезии, в
                    628: переводе означает "промежутки". Какой европейский топоним упоминается в
                    629: его прозвище?
                    630: 
                    631: Ответ:
                    632: Венеция.
                    633: 
                    634: Комментарий:
                    635: Город расположен на искусственном архипелаге из 92 островов, связанных
                    636: системой "промежутков" - искусственных каналов.
                    637: 
                    638: Источник:
                    639: https://ru.wikipedia.org/wiki/Нан-Мадол
                    640: 
                    641: Автор:
                    642: Никита Шевела (Минск), Дмитрий Буценец (Пинск - Минск - Брест)
                    643: 
                    644: Вопрос 29:
                    645: В Великобритании даже отдельные сцены впервые были показаны только
                    646: спустя двадцать лет в документальном фильме "Запретный плод". Какому
                    647: фильму был посвящен "Запретный плод"?
                    648: 
                    649: Ответ:
                    650: "Заводной апельсин".
                    651: 
                    652: Комментарий:
                    653: Несмотря на протесты режиссера, кадры из фильма были использованы.
                    654: Запретный плод "апельсина" оказался сладок.
                    655: 
                    656: Источник:
                    657: https://en.wikipedia.org/wiki/A_Clockwork_Orange_(film)
                    658: 
                    659: Автор:
                    660: Дмитрий Буценец (Пинск - Минск - Брест)
                    661: 
                    662: Вопрос 30:
                    663:    <раздатка>
                    664: |   Год                 Живые языки
                    665: |   2015     ~46 млн.     ~6800
                    666: |   2050    ~131 млн.     ~3400
                    667:    </раздатка>
                    668:    Во втором столбце можно увидеть данные, которые приводят нейробиологи
                    669: Мартин Принц и Андерс ВИмо. Ответьте двумя словами, начинающимися на
                    670: соседние буквы алфавита: распространение чего он показывает?
                    671: 
                    672: Ответ:
                    673: Болезнь Альцгеймера.
                    674: 
                    675: Комментарий:
                    676: Существуют серьезные статистические научные исследования на тему того,
                    677: что владение несколькими языками препятствует или как минимум
                    678: затормаживает развитие болезни Альцгеймера. Сокращение живых языков,
                    679: скорее всего, будет способствовать тому, что люди реже будут знать
                    680: несколько языков одновременно.
                    681: 
                    682: Источник:
                    683:    1. http://miresperanto.com/aliaj_lingvoj/najdjot.htm
                    684:    2. http://www.vesti.ru/doc.html?id=1152674
                    685:    3. http://medportal.ru/mednovosti/news/2015/08/26/745dementia/
                    686: 
                    687: Автор:
                    688: Никита Шевела (Минск)
                    689: 
                    690: Вопрос 31:
                    691: Словами "ИКС" и "ИГРЕК" заменены другие слова.
                    692:    Среди персонажей одной манги есть русский боец. Одна из его атак -
                    693: комбинация ударов ладонью и кулаком носит название "ИКС и ИГРЕК". Что
                    694: сделал персонаж другого произведения после отправления из "ИКСА и
                    695: ИГРЕКА"?
                    696: 
                    697: Ответ:
                    698: И немедленно выпил.
                    699: 
                    700: Зачет:
                    701: Немедленно выпил; выпил.
                    702: 
                    703: Комментарий:
                    704: ИКС и ИГРЕК - это серп и молот. Произведение - "Москва - Петушки". Речь
                    705: идет о главе "Серп и молот - Карачарово", которая состоит из одного
                    706: предложения: "И немедленно выпил".
                    707: 
                    708: Источник:
                    709:    1. http://mangachan.me/online/105658-strongest-disciple-kenichi_v32_ch297.html#page=13
                    710:    2. В.В. Ерофеев. Москва - Петушки. http://flibusta.is/b/282345/read
                    711: 
                    712: Автор:
                    713: Никита Шевела (Минск)
                    714: 
                    715: Вопрос 32:
                    716: Внимание, в вопросе словом "ПРОПУСК" мы заменили два слова.
                    717:    В своем труде Фрейд рассуждал о кажущейся пассивности в создании
                    718: сновидений. В качестве примера он приводил часто произносимые нами слова
                    719: "ПРОПУСК". В русскоязычной песне после слов "ПРОПУСК" упоминается то,
                    720: что можно увидеть в серии работ известного француза. Что же именно?
                    721: 
                    722: Ответ:
                    723: Небо Лондона.
                    724: 
                    725: Комментарий:
                    726: Фрейд отмечает в словах "мне приснилось" пассивный контекст. Известный
                    727: француз - Клод Моне, а серия - "Здание парламента в Лондоне".
                    728: 
                    729: Источник:
                    730:    1. З. Фрейд. Толкование сновидений.
                    731: https://archive.org/stream/Freud_1900_Die_Traumdeutung_k#page/32/mode/2up
                    732:    2. http://www.karaoke.ru/artists/zemfira/text/london/
                    733:    3. https://en.wikipedia.org/wiki/Houses_of_Parliament_(Monet_series)
                    734: 
                    735: Автор:
                    736: Ренат Рустамов (Минск)
                    737: 
                    738: Вопрос 33:
                    739: 10 января 1912 года, когда до цели оставалось около 75 миль, был заложен
                    740: склад, который получил название "Полтора градуса". Назовите эту цель.
                    741: 
                    742: Ответ:
                    743: Южный полюс.
                    744: 
                    745: Комментарий:
                    746: В подобном духе называла свои склады группа Роберта Скотта. Координаты
                    747: южного полюса - 90°00'00" южной широты. Долготы, как известно, полюс не
                    748: имеет, чем и обусловлена идея названия складов.
                    749: 
                    750: Источник:
                    751: https://ru.wikipedia.org/wiki/Скотт,_Роберт
                    752: 
                    753: Автор:
                    754: Дмитрий Буценец (Пинск - Минск - Брест)
                    755: 
                    756: Вопрос 34:
                    757: Ребенок из книги Эммы Донохью всю жизнь прожил взаперти вдвоем с мамой.
                    758: Когда мальчик впервые говорит с другими людьми о матери, ему не сразу
                    759: удается использовать ЭТО. На известном объекте ИМ можно назвать Теодора
                    760: Рузвельта. Назовите ЭТО двумя словами.
                    761: 
                    762: Ответ:
                    763: Третье лицо.
                    764: 
                    765: Комментарий:
                    766: Мальчику не сразу удается сказать о матери "она", вместо этого он дважды
                    767: произносит "ты". Гора Рашмор известна своим скульптурным барельефом
                    768: четырех президентов США. Если смотреть слева направо, "третьим лицом"
                    769: будет Теодор Рузвельт.
                    770: 
                    771: Источник:
                    772:    1. Э. Донохью. Комната.
                    773: https://books.google.ru/books?id=Kw9aDwAAQBAJ&pg=PT196#v=onepage&q&f=false
                    774:    2. https://ru.wikipedia.org/wiki/Рашмор
                    775: 
                    776: Автор:
                    777: Оксана Бандура (Минск)
                    778: 
                    779: Вопрос 35:
                    780: На картине конца восемнадцатого века мужчина играет в шахматы. Описывая
                    781: прическу мужчины - по бокам волосы длинные, а сзади подстрижены коротко,
                    782: - Википедия отмечает, что такая прическа могла облегчить ЕГО работу.
                    783: Назовите ЕГО.
                    784: 
                    785: Ответ:
                    786: Палач.
                    787: 
                    788: Комментарий:
                    789: Такая мода появилась во времена французской революции. Модники
                    790: демонстративно показывали свое неприятие революции. Перед
                    791: гильотинированием палач сзади состригал волосы. На картине используется
                    792: известный сюжет игры в шахматы со смертью.
                    793: 
                    794: Источник:
                    795: https://ru.wikipedia.org/wiki/Портрет_доктора_де_С.,_играющего_в_шахматы_со_Смертью
                    796: 
                    797: Автор:
                    798: Дмитрий Буценец (Пинск - Минск - Брест)
                    799: 
                    800: Вопрос 36:
                    801: В противостоянии ИКСА остальным Поль ВалерИ видел символ разумного
                    802: сознания. Малайцы называют ИКС старшим братом. Какие два слова мы
                    803: заменили ИКСОМ?
                    804: 
                    805: Ответ:
                    806: Большой палец.
                    807: 
                    808: Комментарий:
                    809: В биологии термин звучит как "противопоставленный большой палец". В
                    810: результате эволюции это стало характерно только для человека и некоторых
                    811: других приматов.
                    812: 
                    813: Источник:
                    814:    1. М. Турнье. Зеркало идей. http://flibusta.is/b/313487/read
                    815:    2. http://russian7.ru/post/o-chem-mozhet-rasskazat-bolshoj-palec/
                    816: 
                    817: Автор:
                    818: Ренат Рустамов (Минск), Дмитрий Буценец (Пинск - Минск - Брест)
                    819: 
                    820: Тур:
                    821: 2 этап
                    822: 
                    823: Дата:
                    824: 12-Nov-2016
                    825: 
                    826: Редактор:
                    827: 1-18 - Валерий Семёнов (Минск - Могилев); 19-36 - Вадим Кузмич (Логойск)
                    828: 
                    829: Инфо:
                    830: Валерий Семёнов благодарит команду "Дятлы" за присланные вопросы, а
                    831: команду "Бренно лаская" за тестирование пакета. Вадим Кузмич благодарит
                    832: Даниила Шункевича, Андрея Танану, Аню Якушевич, Виталия Калачёва,
                    833: Евгения Лешковича за помощь в подготовке пакета и ценные замечания,
                    834: команды "New-реанимация" и "Aspers" за присланные вопросы, команду
                    835: "Бренно лаская" за тестирование пакета.
                    836: 
                    837: Вопрос 1:
                    838: На карикатуре Жана Эффеля ЕЕ поздравляют с удачным выбором, ведь лучшего
                    839: человека на свете не найти. Назовите ЕЕ.
                    840: 
                    841: Ответ:
                    842: Ева.
                    843: 
                    844: Комментарий:
                    845: Лучшим мужем на свете был Адам. Жан Эффель известен своей серией
                    846: рисунков "Сотворение мира".
                    847: 
                    848: Источник:
                    849: Ж. Эффель. Сотворение мира. - М.: Изобразительное искусство, 1989. - С.
                    850: 554.
                    851: 
                    852: Автор:
                    853: Сергей Кушмар (Минск)
                    854: 
                    855: Вопрос 2:
                    856: Этнолог Джаред Даймонд пишет, что для высокогорных племен Новой Гвинеи в
                    857: силу отсутствия у них домашних животных был характерен низкобелковый
                    858: рацион. Так ученый объясняет широкую распространенность ЭТОГО среди
                    859: новогвинейских племен. Назовите ЭТО одним словом.
                    860: 
                    861: Ответ:
                    862: Людоедство.
                    863: 
                    864: Зачет:
                    865: Каннибализм.
                    866: 
                    867: Комментарий:
                    868: Больше белок взять негде.
                    869: 
                    870: Источник:
                    871: Дж. Даймонд. Ружья, микробы и сталь. Судьбы человеческих обществ.
                    872: http://flibusta.is/b/237834/read
                    873: 
                    874: Автор:
                    875: Валерий Семёнов (Минск - Могилев)
                    876: 
                    877: Вопрос 3:
                    878: Саудовский экономист Абдалла Муса бен Тайр рассказывает, что обострение
                    879: политического конфликта в 2014 году для его страны не было критичным,
                    880: так как в то время ОН состоял преимущественно из фисташек, шафрана и
                    881: ковров. Назовите ЕГО тремя словами, начинающимися на одну и ту же букву.
                    882: 
                    883: Ответ:
                    884: Импорт из Ирана.
                    885: 
                    886: Комментарий:
                    887: Разрыв отношений с Ираном экономика Саудовской Аравии не почувствовала,
                    888: чего нельзя сказать об иранской экономике.
                    889: 
                    890: Источник:
                    891: https://meduza.io/feature/2016/01/11/eto-voyna-kak-vnutri-strany-tak-i-za-ee-predelami
                    892: 
                    893: Автор:
                    894: Валерий Семёнов (Минск - Могилев)
                    895: 
                    896: Вопрос 4:
                    897: Игорь Малевич был одним из первых советских ученых, командированным в
                    898: НАСА. По его словам, когда он впервые появился в НАСА, у него осторожно
                    899: спросили, не умер ли у него кто-нибудь. Какие слова мы пропустили в
                    900: вопросе?
                    901: 
                    902: Ответ:
                    903: В костюме.
                    904: 
                    905: Зачет:
                    906: В черном костюме; в деловом костюме; в галстуке.
                    907: 
                    908: Комментарий:
                    909: В НАСА все ходили в свободной одежде, обычно в джинсах и футболке.
                    910: 
                    911: Источник:
                    912: https://www.kp.by/daily/23873.3/144169/
                    913: 
                    914: Автор:
                    915: Валерий Семёнов (Минск - Могилев)
                    916: 
                    917: Вопрос 5:
                    918: После Всемирной Парижской выставки 1900 года пресса назвала ЕЕ
                    919: "позвоночным хребтом русского великана". Назовите ЕЕ максимально точно.
                    920: 
                    921: Ответ:
                    922: Транссибирская магистраль.
                    923: 
                    924: Зачет:
                    925: Транссиб; Сибирская железная дорога.
                    926: 
                    927: Комментарий:
                    928: Считалось, что Транссиб обеспечит России приоритет во всём
                    929: Азиатско-Тихоокеанском регионе, а Маньчжурию превратит в русскую
                    930: провинцию. На выставке 1900 года Транссибирская магистраль была
                    931: представлена общественности.
                    932: 
                    933: Источник:
                    934: Изобретение века. Проблемы и модели времени в России и Европе XIX
                    935: столетия. - М.: Новое литературное обозрение, 2013. - С. 248.
                    936: 
                    937: Автор:
                    938: Элеонора Кутас, Сергей Кушмар (Минск)
                    939: 
                    940: Вопрос 6:
                    941: Кирилл Бабаев пишет, что от инфекций, вызванных ржавыми ритуальными
                    942: ножами и грязной "священной" водой, в Африке продолжают умирать тысячи
                    943: инфицированных детей. В предыдущем предложении мы немного изменили одно
                    944: слово. Восстановите это слово.
                    945: 
                    946: Ответ:
                    947: Инициированных.
                    948: 
                    949: Комментарий:
                    950: Обряд инициации у многих народов связан с обрезанием или ритуальными
                    951: надрезами, в результате которых можно получить заражение крови.
                    952: 
                    953: Источник:
                    954: http://prochtenie.ru/passage/28336
                    955: 
                    956: Автор:
                    957: Валерий Семёнов (Минск - Могилев)
                    958: 
                    959: Вопрос 7:
                    960: Внимание, в вопросе есть замена.
                    961:    Статья в журнале "Вокруг света" об ЭТОМ ГОРОДЕ называлась "Флоридские
                    962: бордюры". Назовите ЭТОТ ГОРОД.
                    963: 
                    964: Ответ:
                    965: Сент-Питерсберг.
                    966: 
                    967: Зачет:
                    968: Санкт-Петербург.
                    969: 
                    970: Комментарий:
                    971: Статья называлась "Флоридские поребрики".
                    972: 
                    973: Источник:
                    974: http://www.vokrugsveta.ru/article/242489/
                    975: 
                    976: Автор:
                    977: Сергей Кушмар (Минск)
                    978: 
                    979: Вопрос 8:
                    980: Игорь Можейко пишет, что правивший во второй половине XII века император
                    981: Ши-Цзун восстановил Великий шелковый путь, и в китайских торговых
                    982: городах стала слышна монгольская, арабская, персидская и даже ТАКАЯ
                    983: речь. Какая - ТАКАЯ?
                    984: 
                    985: Ответ:
                    986: Итальянская.
                    987: 
                    988: Комментарий:
                    989: Чуть позже туда уже по проторенной другими купцами дорожке вроде как
                    990: доехал Марко Поло.
                    991: 
                    992: Источник:
                    993: И.В. Можейко. 1185 год. Восток - Запад. Истоки. Мир ислама. Между двух
                    994: миров. http://flibusta.is/b/418253/read
                    995: 
                    996: Автор:
                    997: Валерий Семёнов (Минск - Могилев)
                    998: 
                    999: Вопрос 9:
                   1000: В покере комбинацию из девятки и короля иногда называют двумя словами,
                   1001: начинающимися на соседние буквы алфавита. Какими?
                   1002: 
                   1003: Ответ:
                   1004: Собачья работа.
                   1005: 
                   1006: Комментарий:
                   1007: Название комбинации намекает на название фильма "К-9: Собачья работа".
                   1008: 
                   1009: Источник:
                   1010: https://ru.wikipedia.org/wiki/К-9_(фильм)
                   1011: 
                   1012: Автор:
                   1013: Анна Ермачёнок (Минск)
                   1014: 
                   1015: Вопрос 10:
                   1016: Распутная римлянка из романа Генриха Сенкевича рассказала о сне, в
                   1017: котором видела себя ЕЮ. Это вызвало всеобщий смех. Назовите ЕЕ одним
                   1018: словом максимально точно.
                   1019: 
                   1020: Ответ:
                   1021: Весталка.
                   1022: 
                   1023: Комментарий:
                   1024: В романе описывается Рим первого века нашей эры. Весталки должны были
                   1025: вести целомудренный образ жизни.
                   1026: 
                   1027: Источник:
                   1028:    1. Г. Сенкевич. Камо грядеши. http://flibusta.is/b/279412/read
                   1029:    2. http://ru.wikipedia.org/wiki/Весталки
                   1030: 
                   1031: Автор:
                   1032: Дмитрий Вайтович (Минск)
                   1033: 
                   1034: Вопрос 11:
                   1035: В 1973 году житель Кингстона Роберт Марлин заявил, что в целях
                   1036: самозащиты застрелил некоего Джона Брауна. Назовите профессию Джона
                   1037: Брауна.
                   1038: 
                   1039: Ответ:
                   1040: Полицейский.
                   1041: 
                   1042: Зачет:
                   1043: Шериф.
                   1044: 
                   1045: Комментарий:
                   1046: Этот случай послужил вдохновением Бобу Марли для написания песни "I Shot
                   1047: The Sheriff".
                   1048: 
                   1049: Источник:
                   1050: https://song-story.ru/i-shot-sheriff-bob-marley/
                   1051: 
                   1052: Автор:
                   1053: Михаил Басс (Ярцево)
                   1054: 
                   1055: Вопрос 12:
                   1056: Работая в Корнелльском университете, Ричард Фейнман получил предложение
                   1057: из Калифорнийского Технологического института. Корнелл не хотел терять
                   1058: Фейнмана и поэтому сделал ему встречное предложение. Но стоило Фейнману
                   1059: надумать остаться в Корнелле, как поступало новое предложение от
                   1060: Калтеха, и так далее. Описывая эти события, Фейнман упоминает ЕГО,
                   1061: столкнувшегося с дополнительными трудностями. Назовите ЕГО.
                   1062: 
                   1063: Ответ:
                   1064: [Буриданов] осел.
                   1065: 
                   1066: Комментарий:
                   1067: Эту ситуацию Фейнман уподобил истории с ослом, стоящим между двумя
                   1068: охапками сена, но только столкнувшимся с дополнительным затруднением:
                   1069: стоит ему шагнуть к одной, как другая вырастает в размерах.
                   1070: 
                   1071: Источник:
                   1072: Р. Фейнман. "Вы, разумеется, шутите, мистер Фейнман!".
                   1073: http://flibusta.is/b/497060/read
                   1074: 
                   1075: Автор:
                   1076: Дмитрий Вайтович (Минск)
                   1077: 
                   1078: Вопрос 13:
                   1079: Обычный американский астроном может поработать с телескопом Хейла всего
                   1080: несколько смен в году. Поэтому во время своей смены астрономы надеются,
                   1081: что будет ОНА. "ОНА" - название произведения 1889 года. Назовите ЕЕ.
                   1082: 
                   1083: Ответ:
                   1084: Звездная ночь.
                   1085: 
                   1086: Комментарий:
                   1087: Ты около года ждешь своей очереди, поэтому очень обидно, если небо в
                   1088: твою смену закрыто облаками.
                   1089: 
                   1090: Источник:
                   1091:    1. М. Браун. Как я убил Плутон и почему это было неизбежно.
                   1092: http://flibusta.is/b/297958/read
                   1093:    2. https://ru.wikipedia.org/wiki/Звёздная_ночь
                   1094: 
                   1095: Автор:
                   1096: Валерий Семёнов (Минск - Могилев)
                   1097: 
                   1098: Вопрос 14:
                   1099: Однажды в разговоре с молодым марксистом Энгельс заявил: "Надо, чтобы
                   1100: русские поменьше ПРОПУСК и Маркса, а мыслили так, как это делал бы Маркс
                   1101: на их месте". В этом вопросе мы ПРОПУСК. Какие два слова мы пропустили?
                   1102: 
                   1103: Ответ:
                   1104: Цитировали Энгельса.
                   1105: 
                   1106: Комментарий:
                   1107: Приведенная нами цитата из Энгельса звучит так: "Надо, чтобы русские
                   1108: поменьше цитировали Энгельса и Маркса, а мыслили так, как это делал бы
                   1109: Маркс на их месте".
                   1110: 
                   1111: Источник:
                   1112: http://propaganda-journal.net/834.html
                   1113: 
                   1114: Автор:
                   1115: Валерий Семёнов (Минск - Могилев)
                   1116: 
                   1117: Вопрос 15:
                   1118: Добраться в Норильск можно либо воздушным транспортом, либо водным,
                   1119: поэтому в Норильске остальную часть России называют "ИКСОМ". Назовите
                   1120: тот ИКС, который в 2015-2016 годах посетило около 40 тысяч туристов.
                   1121: 
                   1122: Ответ:
                   1123: Антарктида.
                   1124: 
                   1125: Комментарий:
                   1126: ИКС - это материк.
                   1127: 
                   1128: Источник:
                   1129:    1. https://ru.wikipedia.org/wiki/Норильск
                   1130:    2. http://2.russia.tv/article/show/article_id/208/
                   1131:    3. http://www.iaato.org/tourism-statistics
                   1132: 
                   1133: Автор:
                   1134: Сергей Кушмар (Минск)
                   1135: 
                   1136: Вопрос 16:
                   1137: Дуплет.
                   1138:    1. Председатель Китайской Народной Республики Си Цзиньпин - страстный
                   1139: поклонник футбола. После переговоров в Германии по вопросам фармакологии
                   1140: ему подарили ЕЕ. Назовите ЕЕ двумя словами.
                   1141:    2. Председатель Китайской Народной Республики Си Цзиньпин - страстный
                   1142: поклонник футбола. После переговоров в Германии по вопросам
                   1143: автомобилестроения ему подарили ЕЕ. Назовите ЕЕ двумя словами,
                   1144: начинающимися на парные согласные.
                   1145: 
                   1146: Ответ:
                   1147:    1. Футболка "Байера".
                   1148:    2. Футболка "Вольфсбурга".
                   1149: 
                   1150: Комментарий:
                   1151: (pic: 20161019.jpg)
                   1152:    Основные спонсоры этих команд - крупные фармакологическая и
                   1153: автомобилестроительная компании соответственно. К слову, спонсор
                   1154: "Вердера" - крупная аграрная компания, не имеющая отношения к
                   1155: автомобилестроению.
                   1156: 
                   1157: Источник:
                   1158: https://www.sports.ru/tribuna/blogs/sportbizinfo/850776.html
                   1159: 
                   1160: Автор:
                   1161: Валерий Семёнов (Минск - Могилев)
                   1162: 
                   1163: Вопрос 17:
                   1164: Персонаж Леонида Андреева узнаёт, что его спутница, внешне безгрешная,
                   1165: является распутной женщиной. Каждый вечер он бил ее, однако каждое утро,
                   1166: забыв вчерашнее, он начинал верить ей снова. Назовите двумя словами,
                   1167: начинающимися с одной буквы, то, с чем персонаж сравнивал свою веру.
                   1168: 
                   1169: Ответ:
                   1170: Печень Прометея.
                   1171: 
                   1172: Комментарий:
                   1173: "Как печень у Прометея, за ночь вырастала моя вера, и как коршун, целый
                   1174: день она терзала ее".
                   1175: 
                   1176: Источник:
                   1177: Л.Н. Андреев. Дневник Сатаны.
                   1178: http://az.lib.ru/a/andreew_l_n/text_0850.shtml
                   1179: 
                   1180: Автор:
                   1181: Дмитрий Вайтович (Минск)
                   1182: 
                   1183: Вопрос 18:
                   1184: В одном российском романе утверждается, что для надежности ЕГО следует
                   1185: запить канистрой самогона. Назовите ЕГО двумя короткими словами.
                   1186: 
                   1187: Ответ:
                   1188: Пуд соли.
                   1189: 
                   1190: Комментарий:
                   1191: Цитата из романа: "Пословица про пуд соли права, но, на мой взгляд, эти
                   1192: шестнадцать кило надо еще запить канистрой самогонки - вот тогда точно
                   1193: поймешь, с кем имеешь дело".
                   1194: 
                   1195: Источник:
                   1196: Д. Донцова. Тормоза для блудного мужа.
                   1197: https://books.google.ru/books?id=TLCvIma27FMC&pg=PT41#v=onepage&q&f=false
                   1198: 
                   1199: Автор:
                   1200: Ольга Ярошенко (Могилев)
                   1201: 
                   1202: Вопрос 19:
                   1203: В 2013 году организаторы юбилейной велогонки "Тур де Франс" имели
                   1204: грандиозные планы и решили провести начало гонки в новом для велогонки
                   1205: регионе. Назовите этот регион.
                   1206: 
                   1207: Ответ:
                   1208: Корсика.
                   1209: 
                   1210: Комментарий:
                   1211: Планы ну просто наполеоновские. Наполеон, как известно, родился на
                   1212: Корсике.
                   1213: 
                   1214: Источник:
                   1215:    1. https://fr.wikipedia.org/wiki/Tour_de_France_2013
                   1216:    2. Журнал "Discovery", 2015, N 8 (78). - С. 40.
                   1217: 
                   1218: Автор:
                   1219: Владимир Чернушкин (Минск)
                   1220: 
                   1221: Вопрос 20:
                   1222: Скульптор, персонаж сказки Петра Бормора, использует ЕЕ для создания
                   1223: максимально реалистичных скульптур. Назовите ЕЕ двумя словами,
                   1224: начинающимися на одну и ту же букву.
                   1225: 
                   1226: Ответ:
                   1227: Голова Горгоны.
                   1228: 
                   1229: Комментарий:
                   1230: Скульптор усаживал натурщицу в нужную ему позу, после чего показывал ей
                   1231: голову Медузы Горгоны, и натурщица обращалась в каменную скульптуру.
                   1232: 
                   1233: Источник:
                   1234: П. Бормор. Книга на третье. http://flibusta.is/b/191906/read
                   1235: 
                   1236: Автор:
1.2       rubashki 1237: Павел Новиков (Витебск - Минск)
1.1       rubashki 1238: 
                   1239: Вопрос 21:
                   1240: Иоханнес Шефферус предполагал, что скандинавские языческие боги Тор,
                   1241: Один и Фрейя символизируют ЭТО. Назовите ЭТО двумя словами.
                   1242: 
                   1243: Ответ:
                   1244: Тре Крунур.
                   1245: 
                   1246: Зачет:
                   1247: Три короны.
                   1248: 
                   1249: Комментарий:
                   1250: Тре Крунур (Три короны) - национальный геральдический символ Швеции.
                   1251: 
                   1252: Источник:
                   1253: https://ru.wikipedia.org/wiki/Три_короны
                   1254: 
                   1255: Автор:
                   1256: Андрей Танана (Минск)
                   1257: 
                   1258: Вопрос 22:
                   1259: Внимание, в вопросе есть замены.
                   1260:    Эрик Вейнер рассказывает, что видел в городе Ханчжоу автобус с
                   1261: номером KI55 [кей-ай пятьдесят пять]. Местные жители называют этот
                   1262: автобус ТАКИМ. В названии какой российской достопримечательности есть
                   1263: слово "ТАКОЙ"?
                   1264: 
                   1265: Ответ:
                   1266: Поцелуев мост.
                   1267: 
                   1268: Комментарий:
                   1269: KI55 очень похоже на английское слово "kiss" [кисс], поэтому жители
                   1270: Ханчжоу называют свой автобус "поцелуйным". Поцелуев мост находится в
                   1271: Санкт-Петербурге.
                   1272: 
                   1273: Источник:
                   1274:    1. Э. Вейнер. География гениальности. Где и почему рождаются великие
                   1275: идеи. http://flibusta.is/b/465171/read
                   1276:    2. https://ru.wikipedia.org/wiki/Поцелуев_мост_(Санкт-Петербург)
                   1277: 
                   1278: Автор:
                   1279: Евгений Лешкович (Минск)
                   1280: 
                   1281: Вопрос 23:
                   1282: В конце 2000 года на создание самого большого ЕГО из мороженого пошло
                   1283: около 300 килограммов продукта: сливочного, черничного и клубничного. В
                   1284: одном из слов этого вопроса мы пропустили несколько букв. Напишите это
                   1285: слово в первоначальном виде.
                   1286: 
                   1287: Ответ:
                   1288: Снеговика.
                   1289: 
                   1290: Комментарий:
                   1291: Канун Нового года как-никак. Классический снеговик состоит из трех
                   1292: шаров, для каждого из которых использовали свой тип мороженого.
                   1293: 
                   1294: Источник:
                   1295: https://www.newsru.com/russia/31dec2000/icecream.html
                   1296: 
                   1297: Автор:
                   1298: Вадим Кузмич (Логойск)
                   1299: 
                   1300: Вопрос 24:
                   1301: Дуплет.
                   1302:    1. Когда илионцы вышли на берег, то увидели, что дар Богам,
                   1303: оставленный там, никуда не годится. Какие шесть слов мы заменили в этом
                   1304: вопросе?
                   1305:    2. За последние две тысячи лет ЭТО уменьшилось в размере в несколько
                   1306: раз. В статье Википедии об ЭТОМ упоминаются, в частности, канаты,
                   1307: сплетенные из верблюжьего волоса. Назовите ЭТО двумя словами.
                   1308: 
                   1309: Ответ:
                   1310:    1. Ни в какие ворота не лезет.
                   1311:    2. Игольное ушко.
                   1312: 
                   1313: Комментарий:
                   1314:    2. По одной из версий, самые маленькие городские ворота Иерусалима
                   1315: носили название "Игольное ушко". Верблюд мог пройти в них только без
                   1316: поклажи и наездника. Отсюда и происхождение известной фразы.
                   1317: 
                   1318: Источник:
                   1319:    1. http://история-вещей.рф/instrumentyi/shveynyie-iglyi.html
                   1320:    2. https://ru.wikipedia.org/wiki/Игольное_ушко
                   1321: 
                   1322: Автор:
                   1323: Александр Санин (Минск)
                   1324: 
                   1325: Вопрос 25:
                   1326: В 1979 году погиб Роберт Уильямс, рабочий завода Ford Motor Co. [форд
                   1327: мОтор кАмпани]. Это был первый официально зарегистрированный случай
                   1328: нарушения закона. Наказания, впрочем, не последовало, так как самим
                   1329: Законом оно не предусмотрено. Назовите этот закон абсолютно точно.
                   1330: 
                   1331: Ответ:
                   1332: Первый закон робототехники.
                   1333: 
                   1334: Комментарий:
                   1335: Персонаж книги "Я робот" погибает от рук робота, тем самым нарушившего
                   1336: первый закон робототехники, запрещающий причинять вред человеку.
                   1337: 
                   1338: Источник:
                   1339:    1. https://en.wikipedia.org/wiki/Robert_Williams_(robot_fatality)
                   1340:    2. https://en.wikipedia.org/wiki/Three_Laws_of_Robotics
                   1341: 
                   1342: Автор:
                   1343: Александр Санин (Минск)
                   1344: 
                   1345: Вопрос 26:
                   1346: Действие компьютерной игры "Skyrim" [скАйрим] происходит в вымышленной
                   1347: северной провинции. По сюжету одного из заданий герою поручено выкрасть
                   1348: родословную знаменитого скакуна. Кто значится в родословной самым ранним
                   1349: предком этого скакуна?
                   1350: 
                   1351: Ответ:
                   1352: Слейпнир.
                   1353: 
                   1354: Комментарий:
                   1355: Игра имеет множество аллюзий на историю и мифологию Скандинавии. В
                   1356: частности, знаменитый скакун ведет свою родословную от самого коня
                   1357: Одина.
                   1358: 
                   1359: Источник:
                   1360: http://ru.elderscrolls.wikia.com/wiki/Мороз
                   1361: 
                   1362: Автор:
                   1363: Даниил Шункевич (Минск)
                   1364: 
                   1365: Вопрос 27:
                   1366: По мнению героя одной книги, неприятный запах в автомобиле не смог бы
                   1367: замаскировать целый ИКС освежителей воздуха. Назовите ИКС коротким
                   1368: словом.
                   1369: 
                   1370: Ответ:
                   1371: Лес.
                   1372: 
                   1373: Комментарий:
                   1374: Вонь стояла такая, что даже лес освежителей воздуха не смог бы ее
                   1375: замаскировать. В автомобилях широко распространены освежители воздуха в
                   1376: виде елочек.
                   1377: 
                   1378: Источник:
                   1379: Р. Риггз. Дом странных детей. http://flibusta.is/b/299551/read
                   1380: 
                   1381: Автор:
                   1382: Вадим Кузмич (Логойск)
                   1383: 
                   1384: Вопрос 28:
                   1385: (pic: 20161020.jpg)
                   1386:    На розданном шуточном изображении мы скрыли от вас термин из двух
                   1387: слов, обозначающий современную область информатики. Напишите этот
                   1388: термин.
                   1389: 
                   1390: Ответ:
                   1391: Машинное обучение.
                   1392: 
                   1393: Зачет:
                   1394: Machine learning.
                   1395: 
                   1396: Комментарий:
                   1397: Мониторы на картинке расставлены так, как будто они внимают речам более
                   1398: широкого телевизора.
                   1399: 
                   1400: Источник:
                   1401:    1. (pic: 20161021.jpg)
                   1402:    2. https://ru.wikipedia.org/wiki/Машинное_обучение
                   1403: 
                   1404: Автор:
                   1405: Даниил Шункевич (Минск)
                   1406: 
                   1407: Вопрос 29:
                   1408: Один из комментаторов на сайте habrahabr.ru [хабрахАбр точка ру] шутит,
                   1409: что основная проблема с НИМ - это понять, работает он или нет. Назовите
                   1410: предмет шутки двумя словами, начинающимися на одну и ту же букву.
                   1411: 
                   1412: Ответ:
                   1413: Квантовый компьютер.
                   1414: 
                   1415: Комментарий:
                   1416: Состояние квантового компьютера, как и кота Шрёдингера, описывается
                   1417: квантовой суперпозицией. Если кот Шрёдингера одновременно жив и мертв,
                   1418: то, возможно, квантовый компьютер одновременно работает и не работает.
                   1419: 
                   1420: Источник:
                   1421: https://geektimes.ru/post/267324/#comment_8907406
                   1422: 
                   1423: Автор:
                   1424: Андрей Танана (Минск)
                   1425: 
                   1426: Вопрос 30:
                   1427: Согласно книге "Одноэтажная Америка", чрезвычайно практичные американцы
                   1428: повесили возле НЕЕ табличку, сообщающую, что из НЕЕ можно было бы
                   1429: построить 40 домов по пять комнат в каждом. Назовите ЕЕ максимально
                   1430: точно.
                   1431: 
                   1432: Ответ:
                   1433: [Секвойя] "Генерал Шерман".
                   1434: 
                   1435: Комментарий:
                   1436: Секвойя "Генерал Шерман" - первое по объему и массе дерево на Земле.
                   1437: Высота "Генерала Шермана" - 83,8 метра, масса оценивается в 1900 тонн, а
                   1438: объем ствола - в 1487 кубических метров.
                   1439: 
                   1440: Источник:
                   1441: И. Ильф, Е. Петров. Одноэтажная Америка.
                   1442: http://flibusta.is/b/168602/read
                   1443: 
                   1444: Автор:
                   1445: Вадим Кузмич (Логойск)
                   1446: 
                   1447: Вопрос 31:
                   1448: Влад Цепеш является прототипом графа Дракулы. Статья о Цепеше и его
                   1449: потомках, опубликованная в журнале "Вокруг света", называется "НАСТОЯЩИЕ
                   1450: ДЕТЕКТИВЫ". Название какого фильма мы заменили?
                   1451: 
                   1452: Ответ:
                   1453: "Реальные упыри".
                   1454: 
                   1455: Комментарий:
                   1456: В отличие от Дракулы, Цепеш существовал в реальности. Слова "настоящие"
                   1457: и "реальные" - синонимы.
                   1458: 
                   1459: Источник:
                   1460: "Вокруг света", 2016, N 8, обложка.
                   1461: 
                   1462: Автор:
                   1463: Евгений Лешкович (Минск)
                   1464: 
                   1465: Вопрос 32:
                   1466: (pic: 20161022.jpg)
                   1467:    В последней строке раздаточного материала упоминается изобретение
                   1468: Альфреда Баттса, придуманное им в 1938 году. Назовите это изобретение.
                   1469: 
                   1470: Ответ:
                   1471: Scrabble.
                   1472: 
                   1473: Зачет:
                   1474: Скрэббл; Эрудит.
                   1475: 
                   1476: Комментарий:
                   1477: Последняя строка: "Сыграть партию в Скрэббл и не получить по лицу".
                   1478: 
                   1479: Источник:
                   1480:    1. https://xkcd.com/1750/
                   1481:    2. https://en.wikipedia.org/wiki/Scrabble
                   1482: 
                   1483: Автор:
                   1484: Виталий Калачёв (Минск)
                   1485: 
                   1486: Вопрос 33:
                   1487: В августе 1893 года известный человек остановился в отеле "Du Sauvage"
                   1488: [дю совАж]. Прогуливаясь по окрестностям с друзьями, этот человек
                   1489: посетовал: "Он отвлекает мои мысли от более важных дел", после чего
                   1490: человека отвели к ИКСУ. Назовите ИКС двумя словами.
                   1491: 
                   1492: Ответ:
                   1493: Рейхенбахский водопад.
                   1494: 
                   1495: Комментарий:
                   1496: Артур Конан Дойл жаловался друзьям, что не знает, как избавиться от
                   1497: Шерлока Холмса. Сэр Генри Ланн посоветовал писателю столкнуть его в
                   1498: Рейхенбахский водопад, который затем ему и показал вживую.
                   1499: 
                   1500: Источник:
                   1501: http://www.vokrugsveta.ru/article/259112/
                   1502: 
                   1503: Автор:
1.2       rubashki 1504: Павел Новиков (Витебск - Минск)
1.1       rubashki 1505: 
                   1506: Вопрос 34:
                   1507: Недавно к НИМ официально присоединился пингвин, появившийся благодаря
                   1508: обнаруженной в архивах детской фотографии. Назовите ИХ, использовав два
                   1509: дефиса.
                   1510: 
                   1511: Ответ:
                   1512: Все-все-все.
                   1513: 
                   1514: Зачет:
                   1515: "Винни-Пух и все-все-все".
                   1516: 
                   1517: Комментарий:
                   1518: ОНИ - друзья Винни-Пуха. В названии перевода Бориса Заходера - те самые
                   1519: "все-все-все". К 90-летию персонажа выпустили новый официальный сборник
                   1520: рассказов.
                   1521: 
                   1522: Источник:
                   1523: https://www.theguardian.com/books/2016/sep/19/winnie-the-pooh-penguin-anniversary-aa-milne
                   1524: 
                   1525: Автор:
                   1526: Виталий Калачёв (Минск)
                   1527: 
                   1528: Вопрос 35:
                   1529: Уроженец Ханьчжоу Шень Ко был выдающимся математиком, астрологом,
                   1530: этнографом, поэтом и дипломатом. Рассказывая о Шень Ко, Эрик Вейнер
                   1531: называет его ТАКИМ ИМ. Ответьте, используя два слова, начинающиеся на
                   1532: соседние буквы: что мы заменили словами "ТАКОЙ ОН"?
                   1533: 
                   1534: Ответ:
                   1535: Китайский Леонардо.
                   1536: 
                   1537: Комментарий:
                   1538: Шень Ко был настоящим человеком Возрождения. Правда, родился он за
                   1539: триста лет до Возрождения.
                   1540: 
                   1541: Источник:
                   1542: Э. Вейнер. География гениальности. Где и почему рождаются великие идеи.
                   1543: http://flibusta.is/b/465171/read
                   1544: 
                   1545: Автор:
                   1546: Евгений Лешкович (Минск)
                   1547: 
                   1548: Вопрос 36:
                   1549: Владелец футбольного клуба "Уимблдон" Сэм Хаммам прописывал в контракте
                   1550: каждого игрока, что в случае победы с разницей в четыре мяча тот пойдет
                   1551: в оперу. Какое слово в предыдущем предложении мы заменили?
                   1552: 
                   1553: Ответ:
                   1554: Поражения.
                   1555: 
                   1556: Комментарий:
                   1557: Хаммам неплохо знал своих футболистов и хорошо понимал, что высидеть
                   1558: оперу для них будет не наградой, а наказанием.
                   1559: 
                   1560: Источник:
                   1561: И. Калашников. Мир английского футбола. Знаменитые клубы, легендарные
                   1562: игроки и драматичные сюжеты.
                   1563: https://books.google.ru/books?id=kcI_DQAAQBAJ&pg=PT81#v=onepage&q&f=false
                   1564: 
                   1565: Автор:
                   1566: Евгений Лешкович (Минск)
                   1567: 
                   1568: Тур:
                   1569: 3 этап
                   1570: 
                   1571: Дата:
                   1572: 10-Dec-2016
                   1573: 
                   1574: Редактор:
                   1575: 1-18 - Аркадий Рух (Минск); 19-36 - Павел Свердлов (Минск)
                   1576: 
                   1577: Инфо:
                   1578: Аркадий Рух благодарит за тестирование и ценные замечания: Алексея
                   1579: Морозова (Тула), Ирину Зубкову (Нижний Новгород), Андрея Кокуленко
                   1580: (Омск), Виталия Пронькина (Реутов), Андрея Данченко (Винница),
                   1581: Александра Немировского (Киев), Серафима Шибанова (Москва), Дмитрия
                   1582: Когана (Штутгарт), Александра Кудрявцева (Николаев), Бориса Моносова
                   1583: (Санкт-Петербург), Яну Азриэль (Хайфа), Николая Коврижных (Москва),
                   1584: Артема Корсуна (Харьков), Анну Виниченко (Минск). Особая благодарность
                   1585: Ивану Топчию (Минск).
                   1586: 
                   1587: Вопрос 1:
                   1588: В анекдоте ТАКИЕ парашютисты полностью деморализовали армию противника.
                   1589: ТАКОЕ королевство существовало с 854 по 1707 год. Какое - ТАКОЕ?
                   1590: 
                   1591: Ответ:
                   1592: Шотландское.
                   1593: 
                   1594: Комментарий:
                   1595: В 1707 году Шотландское и Английское королевства были объединены в
                   1596: Великобританию. Килт - традиционная шотландская мужская юбка - носится
                   1597: без нижнего белья.
                   1598: 
                   1599: Источник:
                   1600:    1. https://www.anekdot.ru/id/634868/
                   1601:    2. https://ru.wikipedia.org/wiki/Шотландское_королевство
                   1602: 
                   1603: Автор:
                   1604: Аркадий Рух (Минск)
                   1605: 
                   1606: Вопрос 2:
                   1607: В статье, опубликованной летом 2016 года, американский журналист Карен
                   1608: Зраик пишет, что "многие уже попали на первые полосы газет из-за
                   1609: сомнительного решения ДЕЛАТЬ ЭТО в Освенциме, на Арлингтонском кладбище
                   1610: и на мемориале 11 сентября". Ответьте двумя словами: что делать?
                   1611: 
                   1612: Ответ:
                   1613: Ловить покемонов.
                   1614: 
                   1615: Комментарий:
                   1616: Речь в статье идет о сверхпопулярной игре "Покемон Го", увлечение
                   1617: которой стало настоящей эпидемией. Покемонов, в самом деле, порою ловят
                   1618: в самых неподходящих для этого местах.
                   1619: 
                   1620: Источник:
                   1621: https://www.inopressa.ru/article/21Jul2016/nytimes/pokem.html
                   1622: 
                   1623: Автор:
                   1624: Аркадий Рух (Минск)
                   1625: 
                   1626: Вопрос 3:
                   1627: В фильме "Маленькие пальчики" Гэри Олдмен и Питер Динклейдж сыграли
                   1628: членов одной семьи. На съемках Олдмену приходилось всё время УНИЖАТЬСЯ.
                   1629: Какие три слова мы заменили словом "УНИЖАТЬСЯ"?
                   1630: 
                   1631: Ответ:
                   1632: Стоять на коленях.
                   1633: 
                   1634: Комментарий:
                   1635: Фильм рассказывает о семье лилипутов. Невысокий рост Питера Динклейджа
                   1636: хорошо известен, например, по роли Тириона Ланнистера в телесериале
                   1637: "Игра Престолов". Гэри Олдмен - актер нормального роста, тем не менее,
                   1638: приглашенный играть брата главного героя. Поэтому его никогда не
                   1639: показывают в полный рост, а чтобы актер казался ниже, он постоянно
                   1640: находился на коленях.
                   1641: 
                   1642: Источник:
                   1643: https://www.youtube.com/watch?v=S36wab0hrNE
                   1644: 
                   1645: Автор:
                   1646: Аркадий Рух (Минск)
                   1647: 
                   1648: Вопрос 4:
                   1649: Персонаж цикла романов "Песнь Льда и Огня" рыцарь ПАтрек с Королевской
                   1650: Горы получил свое имя в честь ПатрИка Сен-ДенИ, друга Джорджа Мартина.
                   1651: Назовите родной город ПатрИка Сен-ДенИ.
                   1652: 
                   1653: Ответ:
                   1654: Монреаль.
                   1655: 
                   1656: Комментарий:
                   1657: Столица Квебека - франкоговорящей провинции Канады. Собственно, название
                   1658: города и переводится как "Королевская гора".
                   1659: 
                   1660: Источник:
                   1661: http://7kingdoms.ru/wiki/Патрек_с_Королевской_Горы
                   1662: 
                   1663: Автор:
                   1664: Аркадий Рух (Минск)
                   1665: 
                   1666: Вопрос 5:
                   1667: Джордж Мартин пишет, что ЭТО короткое СЛОВО часто сопровождается
                   1668: разочарованиями и огорчениями. Напишите ЭТО СЛОВО, которое в оригинале в
                   1669: два раза короче, чем в русском переводе, и часто используется в качестве
                   1670: примера лаконичности.
                   1671: 
                   1672: Ответ:
                   1673: Если.
                   1674: 
                   1675: Зачет:
                   1676: If.
                   1677: 
                   1678: Комментарий:
                   1679: "Если" - это еще и знаменитый ответ спартанцев Филиппу Македонскому на
                   1680: угрозу уничтожить город, если он его захватит.
                   1681: 
                   1682: Источник:
                   1683:    1. Дж. Мартин. Чумная звезда.
                   1684: https://books.google.ru/books?id=UfmmAAAAQBAJ&pg=PA5-IA9#v=onepage&q&f=false
                   1685:    2. https://ru.wikipedia.org/wiki/Лаконичность
                   1686: 
                   1687: Автор:
                   1688: Аркадий Рух (Минск)
                   1689: 
                   1690: Вопрос 6:
                   1691: В песенке команды КВН "Рижские готы" говорится о том, что одна женщина
                   1692: настолько плохо справляется со своими обязанностями, что ей не страшен
                   1693: даже электрический стул. Назовите ее профессию.
                   1694: 
                   1695: Ответ:
                   1696: Проводник.
                   1697: 
                   1698: Зачет:
                   1699: Проводница.
                   1700: 
                   1701: Комментарий:
                   1702:    Проводнице по имени Даша
                   1703:    Электрический стул не так страшен.
                   1704:    Хоть разряд был велик,
                   1705:    В Дашу он не проник:
                   1706:    Даша очень плохой проводник!
                   1707: 
                   1708: Источник:
                   1709: КВН-2012. Сборная Прибалтики. 1/8 финала. Приветствие.
                   1710: https://www.youtube.com/watch?v=D0SnueSqxrU&t=3m07s
                   1711: 
                   1712: Автор:
                   1713: Аркадий Рух (Минск)
                   1714: 
                   1715: Вопрос 7:
                   1716: В январе 1980 года в Белграде были внимательно изучены видеозаписи
                   1717: Уинстона Черчилля, Джона Кеннеди, графа Маунтбеттена и других. В ноябре
                   1718: 1982 году в СССР без подобных записей смогли обойтись. Какое слово мы
                   1719: пропустили в вопросе?
                   1720: 
                   1721: Ответ:
                   1722: Похорон.
                   1723: 
                   1724: Комментарий:
                   1725: За долгие годы правления Иосипа Броза Тито в Югославии была напрочь
                   1726: утрачена традиция похорон первых лиц государства. Пришлось обращаться к
                   1727: "зарубежному опыту". В 1982 году, когда в СССР умер Брежнев, таких
                   1728: проблем не возникло.
                   1729: 
                   1730: Источник:
                   1731: https://ru.wikipedia.org/wiki/Смерть_и_похороны_Иосипа_Броза_Тито
                   1732: 
                   1733: Автор:
                   1734: Аркадий Рух (Минск)
                   1735: 
                   1736: Вопрос 8:
                   1737: Владелец одного британского бара заметил, что даже самые сплоченные
                   1738: семьи и дружные компании в его заведении превращаются в сборище
                   1739: одиночек. Чтобы вернуть старые добрые времена, владелец воспользовался
                   1740: изобретением, сделанным 180 лет назад. Ответьте двумя словами, одно из
                   1741: которых - имя собственное: как называется это изобретение?
                   1742: 
                   1743: Ответ:
                   1744: Клетка Фарадея.
                   1745: 
                   1746: Зачет:
                   1747: Комната Фарадея; щит Фарадея.
                   1748: 
                   1749: Комментарий:
                   1750: Клетка Фарадея, экранирующая электромагнитное поле, не позволяет
                   1751: посетителям бара пользоваться мобильной связью, wi-fi и прочими
                   1752: "достижениями цивилизации", вынуждая людей общаться между собой.
                   1753: 
                   1754: Источник:
                   1755:    1. https://www.telegraph.co.uk/science/2016/08/02/landlord-installs-faraday-cage-to-block-phone-signals-because-so/
                   1756:    2. https://ru.wikipedia.org/wiki/Клетка_Фарадея
                   1757: 
                   1758: Автор:
                   1759: Аркадий Рух (Минск)
                   1760: 
                   1761: Вопрос 9:
                   1762: [Ведущему: кавычки не озвучивать.]
                   1763:    Знакомая автора вопроса жалуется, что ее ребенок-подросток слишком
                   1764: увлечен "Вселенной Стивена" и вместо книг читает только ИХ. Назовите ИХ
                   1765: словом французского происхождения.
                   1766: 
                   1767: Ответ:
                   1768: Субтитры. Незачет: Титры.
                   1769: 
                   1770: Комментарий:
                   1771: Девочка увлечена мультсериалом "Вселенная Стивена" и читает
                   1772: преимущественно субтитры к нему. Титры, показываемые перед и после
                   1773: серии, ее не особо интересуют.
                   1774: 
                   1775: Источник:
                   1776:    1. http://memnoga.livejournal.com/26748.html
                   1777:    2. https://ru.wikipedia.org/wiki/Субтитры
                   1778:    3. https://ru.wikipedia.org/wiki/Титры
                   1779: 
                   1780: Автор:
                   1781: Аркадий Рух (Минск)
                   1782: 
                   1783: Вопрос 10:
                   1784: Внимание, в вопросе есть замены.
                   1785:    Статья на сайте Газета.ру, посвященная кризису экономики одной из
                   1786: нефтедобывающих стран, называется "ПЕРВЫЕ становятся ВТОРЫМИ". В
                   1787: экранизации 1972 года Витя Смирнов сыграл и ПЕРВОГО, и ВТОРОГО. Какие
                   1788: слова мы заменили словами "ПЕРВЫЙ" и "ВТОРОЙ"?
                   1789: 
                   1790: Ответ:
                   1791: Принц, нищий.
                   1792: 
                   1793: Комментарий:
                   1794: Статья "Принцы становятся нищими" посвящена экономическому кризису в
                   1795: Саудовской Аравии, вызванному падением цен на нефть. "Принц и нищий" -
                   1796: неоднократно экранизированный роман Марка Твена о том, как маленький
                   1797: попрошайка Том Кенти внезапно оказался двойником наследника английского
                   1798: престола.
                   1799: 
                   1800: Источник:
                   1801:    1. https://www.gazeta.ru/business/2016/09/29/10221485.shtml
                   1802:    2. https://ru.wikipedia.org/wiki/Принц_и_нищий_(фильм,_1972)
                   1803: 
                   1804: Автор:
                   1805: Аркадий Рух (Минск)
                   1806: 
                   1807: Вопрос 11:
                   1808: [Ведущему: в словах "АЛЬФЕ" и "АЛЬФОМ" как можно точнее проговорить
                   1809: окончания.]
                   1810:    Внимание, в вопросе есть замены.
                   1811:    В материале, посвященном Всемирному дню почты, говорится, что на
                   1812: первой "АЛЬФЕ с АЛЬФОМ" были изображены "Обрученные и Эйфелева башня". А
                   1813: вот на белорусской "АЛЬФЕ с АЛЬФОМ", появившейся в 1993 году, изображена
                   1814: "Прогулка". Какое слово мы заменили словом "АЛЬФА"?
                   1815: 
                   1816: Ответ:
                   1817: Марка.
                   1818: 
                   1819: Комментарий:
                   1820: Речь идет о "марках с Марком" - филателистической продукции, посвященной
                   1821: творчеству великого белорусского художника Марка Шагала.
                   1822: 
                   1823: Источник:
                   1824: http://fun.rambler.ru/special/vsemirnyj-den-pochty/marka-s-markom/
                   1825: 
                   1826: Автор:
                   1827: Аркадий Рух (Минск)
                   1828: 
                   1829: Вопрос 12:
                   1830: Французское слово "шифр" происходит от арабского "сифр", обозначающего
                   1831: ЕГО. В латинских переводах XII века для ЕГО обозначения использовалось
                   1832: слово "циркулюс". Назовите ЕГО.
                   1833: 
                   1834: Ответ:
                   1835: Ноль.
                   1836: 
                   1837: Комментарий:
                   1838: Ту же этимологию имеет и слово "цифра". Циркулюс, или "кружок", - вполне
                   1839: подходящее название для ноля.
                   1840: 
                   1841: Источник:
                   1842:    1. https://ru.wikipedia.org/wiki/Шифр
                   1843:    2. https://ru.wikipedia.org/wiki/Ноль_(цифра)
                   1844: 
                   1845: Автор:
                   1846: Аркадий Рух (Минск)
                   1847: 
                   1848: Вопрос 13:
                   1849: В произведении Рэя Брэдбери ОН сначала "смаковал картины Пикассо и
                   1850: Матисса", а затем перекрашивал портьеры в спальне. По одной из версий,
                   1851: слово "ОН" происходит от имени древнеиндийского божества. Назовите ЕГО.
                   1852: 
                   1853: Ответ:
                   1854: Огонь. Незачет: Пожар.
                   1855: 
                   1856: Комментарий:
                   1857: Брэдбери описывает пожар. Русское слово "огонь" родственно санскритскому
                   1858: Агни - богу огня.
                   1859: 
                   1860: Источник:
                   1861:    1. Р. Брэдбери. Будет ласковый дождь.
                   1862: http://flibusta.is/b/181956/read
                   1863:    2. https://ru.wikipedia.org/wiki/Огонь
                   1864:    3. https://ru.wikipedia.org/wiki/Агни
                   1865: 
                   1866: Автор:
                   1867: Аркадий Рух (Минск)
                   1868: 
                   1869: Вопрос 14:
                   1870: В 1921 году некий Бартоломеус Шпакович застрелил последнего беловежского
                   1871: зубра. Зоолог Эрна Морн сравнила Шпаковича с НИМ, хотя руководствовалась
                   1872: причинами, прямо противоположными тем, благодаря которым ОН попал в
                   1873: историю. Назовите ЕГО.
                   1874: 
                   1875: Ответ:
                   1876: Герострат.
                   1877: 
                   1878: Комментарий:
                   1879: Эрна Морн считает, что имя Шпаковича, как и имя Герострата, должно жить
                   1880: в веках, как предостережение. Самого Герострата, напротив, было
                   1881: приказано забыть.
                   1882: 
                   1883: Источник:
                   1884: https://life.ru/t/наука/903075/zdies_byli_lvy_kratkaia_istoriia_brakonierstva
                   1885: 
                   1886: Автор:
                   1887: Аркадий Рух (Минск)
                   1888: 
                   1889: Вопрос 15:
                   1890: В рассказе Евгения Лукина орел, терзавший Прометея, носит имя арабского
                   1891: происхождения. Назовите это имя.
                   1892: 
                   1893: Ответ:
                   1894: Алкоголь.
                   1895: 
                   1896: Комментарий:
                   1897: Орел, как известно, клевал Прометею печень. А "цирроз" - слово
                   1898: греческого происхождения.
                   1899: 
                   1900: Источник:
                   1901:    1. Е.Ю. Лукин. Произведение искусства.
                   1902: http://flibusta.is/b/327116/read#t35
                   1903:    2. https://ru.wikipedia.org/wiki/Алкоголь
                   1904: 
                   1905: Автор:
                   1906: Аркадий Рух (Минск)
                   1907: 
                   1908: Вопрос 16:
                   1909: Герой новеллы Теодора Гофмана, впервые опубликованной во "Всеобщей
                   1910: музыкальной газете", рассказывает о том, как в 1809 году герой встретил
                   1911: ЕГО спустя двадцать лет после ЕГО смерти. Назовите ЕГО односложную
                   1912: фамилию.
                   1913: 
                   1914: Ответ:
                   1915: Глюк.
                   1916: 
                   1917: Комментарий:
                   1918: По сюжету великий австрийский композитор Кристофор Виллибальд Глюк был
                   1919: так возмущен берлинской постановкой своей оперы, что решил вмешаться.
                   1920: "Глюк" - это еще и разговорное название галлюцинации.
                   1921: 
                   1922: Источник:
                   1923:    1. https://ru.wikipedia.org/wiki/Кавалер_Глюк
                   1924:    2. https://ru.wikipedia.org/wiki/Глюк,_Кристоф_Виллибальд
                   1925:    3. https://ru.wikipedia.org/wiki/Глюк_(значения)
                   1926: 
                   1927: Автор:
                   1928: Аркадий Рух (Минск)
                   1929: 
                   1930: Вопрос 17:
                   1931: Британская "Дейли мейл" отмечает роль феминизма в том, что современные
                   1932: женщины стали заботиться о своем ИКСЕ. Можно сказать, что без
                   1933: изобретения другого ИКСА никакой "Дейли мейл" бы не было. Какое слово мы
                   1934: заменили ИКСОМ?
                   1935: 
                   1936: Ответ:
                   1937: Пресс.
                   1938: 
                   1939: Комментарий:
                   1940: Газета пишет, что с ростом влияния феминизма женщины стали больше
                   1941: следить за своей физической формой, в частности - за состоянием мышц
                   1942: брюшного пресса. Без изобретения печатного пресса появление "Дейли мейл"
                   1943: было бы невозможно.
                   1944: 
                   1945: Источник:
                   1946:    1. https://lenta.ru/articles/2015/10/21/waist/
                   1947:    2. http://finances.social/biznes_760_762/pechatnyiy-press-39703.html
                   1948: 
                   1949: Автор:
                   1950: Аркадий Рух (Минск)
                   1951: 
                   1952: Вопрос 18:
                   1953: Один из элементов награды, вручавшейся в Советском Союзе с 1932 по 1941
                   1954: год, состоял из нескольких концентрических кругов. Мы не спрашиваем,
                   1955: какое имя собственное входило в название награды. Ответьте, за
                   1956: достижения в какой области она вручалась.
                   1957: 
                   1958: Ответ:
                   1959: Стрельба.
                   1960: 
                   1961: Комментарий:
                   1962: В нижней части значка "Ворошиловский стрелок", вручаемой за меткость,
                   1963: находилось изображение круглой мишени. Редактор тура надеется, что
                   1964: сегодня команды поразили все мишени.
                   1965: 
                   1966: Источник:
                   1967: https://ru.wikipedia.org/wiki/Ворошиловский_стрелок
                   1968: 
                   1969: Автор:
                   1970: Аркадий Рух (Минск)
                   1971: 
                   1972: Вопрос 19:
                   1973:    <раздатка>
                   1974:    Победитель - тот, кто солнце на один раз больше, чем давление.
                   1975:    </раздатка>
                   1976:    Какие два глагола мы поменяли на существительные в цитате из книги
                   1977: Мии Хэмм?
                   1978: 
                   1979: Ответ:
                   1980: Встает, падает.
                   1981: 
                   1982: Зачет:
                   1983: Поднимается, падает и т.п. по смыслу.
                   1984: 
                   1985: Комментарий:
                   1986: Солнце тоже встает, а давление тоже падает.
                   1987: 
                   1988: Источник:
                   1989: https://www.goodreads.com/quotes/324217-a-winner-is-that-person-who-gets-up-one-more
                   1990: 
                   1991: Автор:
                   1992: Ольга Потапова ("Семь сорок")
                   1993: 
                   1994: Вопрос 20:
                   1995: Тим Бернерс-Ли как-то сказал, что если бы он в свое время мог
                   1996: предположить, сколько людей будет пользоваться Интернетом, то ни за что
                   1997: не стал бы использовать... Что? Изобразите ответ на своих карточках.
                   1998: 
                   1999: Ответ:
                   2000: //
                   2001: 
                   2002: Зачет:
                   2003: :// Незачет: www, http и т.д.
                   2004: 
                   2005: Комментарий:
                   2006: Изобретатель Интернета сделал бы синтаксис проще, в частности,
                   2007: отказавшись от двух слэшей в URL.
                   2008: 
                   2009: Источник:
                   2010: https://citaty.info/quote/364474
                   2011: 
                   2012: Автор:
                   2013: Павел Свердлов ("Семь сорок")
                   2014: 
                   2015: Вопрос 21:
                   2016: Жесткая конкуренция со "Standard Oil" привела к слиянию двух компаний,
                   2017: одна из которых в год своего основания заручилась поддержкой короля
                   2018: Виллема III. Назовите то, что получилось после слияния.
                   2019: 
                   2020: Ответ:
                   2021: "Royal Dutch Shell".
                   2022: 
                   2023: Зачет:
                   2024: "Shell".
                   2025: 
                   2026: Комментарий:
                   2027: Поддержка короля Нидерландов Виллема III отразилась на названии компании
                   2028: "Royal Dutch". В 1907 году RD объединилась с "The Shell Transport and
                   2029: Trading Company". Новая компания стала называться "Royal Dutch Shell".
                   2030: Или просто "Shell".
                   2031: 
                   2032: Источник:
                   2033: http://www.fundinguniverse.com/company-histories/royal-dutch-petroleum-company-the-shell-transport-and-trading-company-p-l-c-history/
                   2034: 
                   2035: Автор:
                   2036: Павел Свердлов ("Семь сорок")
                   2037: 
                   2038: Вопрос 22:
                   2039: В 1950-х годах ученые, в лабораторных условиях исследовавшие поведение,
                   2040: смогли управлять АЛЬФОЙ как ИГРЕКОМ. Что мы заменили АЛЬФОЙ и ИГРЕКОМ?
                   2041: 
                   2042: Ответ:
                   2043: Мышь, джойстик.
                   2044: 
                   2045: Зачет:
                   2046: Мышь, курсор.
                   2047: 
                   2048: Комментарий:
                   2049: В лабораториях бывают мыши. Бывает, их там бьют током. Бывает, что после
                   2050: этого они ходят так, как этого хотят ученые. Мышь и джойстик - два
                   2051: компьютерных девайса.
                   2052: 
                   2053: Источник:
                   2054: https://psyfactor.org/lib/dofamin.htm
                   2055: 
                   2056: Автор:
                   2057: Александра Бурчалова ("Семь сорок")
                   2058: 
                   2059: Вопрос 23:
                   2060: Очевидцы сообщали, что однажды в начале 1918 года на Кузнецком мосту в
                   2061: Москве из-за НЕЕ образовался небольшой затор. С каким глаголом в том же
                   2062: году срифмовал ЕЕ известный поэт?
                   2063: 
                   2064: Ответ:
                   2065: Клёшить.
                   2066: 
                   2067: Комментарий:
                   2068: Уставшую лошадь, упавшую на Кузнецком мосту, Маяковский сделал героем
                   2069: стихотворения "Хорошее отношение к лошадям". В нем есть строки:
                   2070:    "Лошадь на круп грохнулась,
                   2071:    и сразу за зевакой зевака,
                   2072:    штаны пришедшие Кузнецким клёшить,
                   2073:    сгрудились, смех зазвенел и зазвякал:
                   2074:    - Лошадь упала! - Упала лошадь!".
                   2075: 
                   2076: Источник:
                   2077:    1. http://www.itaka.msk.ru/archives/617
                   2078:    2. В.В. Маяковский. Хорошее отношение к лошадям.
                   2079: http://www.feb-web.ru/feb/mayakovsky/texts/ms0/ms2/ms2-010-.htm
                   2080: 
                   2081: Автор:
                   2082: Павел Свердлов ("Семь сорок")
                   2083: 
                   2084: Вопрос 24:
                   2085: В вопросе есть замена.
                   2086:    Николай Кононов пишет, что название "ИКС" пришло в голову сразу.
                   2087: Изначально предполагалось добавлять к "ИКСУ" городА (например, "ИКС с
                   2088: Хабаровском"), но от этой идеи быстро отказались. Также герой книги
                   2089: рассуждает о пользе заимствования технологий из других стран. Что мы
                   2090: заменили ИКСОМ?
                   2091: 
                   2092: Ответ:
                   2093: "ВКонтакте".
                   2094: 
                   2095: Комментарий:
                   2096: Речь в вопросе идет про книгу Николая Кононова "Код Дурова", посвященную
                   2097: истории создания соцсети. В одном из отрывков книги приводится точка
                   2098: зрения Дурова на обвинения в заимствовании идеи и функционала соцсети у
                   2099: американской Facebook.
                   2100: 
                   2101: Источник:
                   2102: http://www.forbes.ru/sobytiya-opinion/lyudi/212150-kod-pavla-durova-pyat-istorii-iz-zhizni-vkontakte-i-ee-sozdatelya
                   2103: 
                   2104: Автор:
                   2105: Станислав Габрусевич ("Семь сорок")
                   2106: 
                   2107: Вопрос 25:
                   2108: Согласно анонимной цитате из Интернета, если ваша АЛЬФА толстая, то вам
                   2109: не хватает физических нагрузок. Назовите композитора, с которым с 1992
                   2110: года ассоциируется одна АЛЬФА.
                   2111: 
                   2112: Ответ:
                   2113: Бетховен.
                   2114: 
                   2115: Комментарий:
                   2116: АЛЬФА - это собака. Бетховен - это не только композитор, но и кличка
                   2117: собаки из одноименного фильма.
                   2118: 
                   2119: Источник:
                   2120:    1. https://www.dogquotations.com/dog-quotes-author-unknown2.html
                   2121:    2. https://en.wikipedia.org/wiki/Beethoven_(film)
                   2122: 
                   2123: Автор:
                   2124: Алексей Евдоченко ("Семь сорок")
                   2125: 
                   2126: Вопрос 26:
                   2127: Советский крейсер, построенный в 1937 году, назвали в честь Вячеслава
                   2128: Молотова. Через двадцать лет, после разгрома "антипартийной группы"
                   2129: Молотова, Кагановича и Маленкова, корабль переименовали. Новое название
                   2130: выбрали то ли с издевкой, то ли чтобы на всякий случай не обидеть
                   2131: опального министра. Какое название стал носить крейсер?
                   2132: 
                   2133: Ответ:
                   2134: "Слава".
                   2135: 
                   2136: Источник:
                   2137: https://mi3ch.livejournal.com/3569303.html
                   2138: 
                   2139: Автор:
                   2140: Егор Сидорович ("Семь сорок")
                   2141: 
                   2142: Вопрос 27:
                   2143: (pic: 20161023.jpg)
                   2144:    Андрей Громыко больше трех десятков лет находился на вершине
                   2145: советской партийной номенклатуры. Описывая стиль группы "Громыка",
                   2146: журналист Кирилл Ступников добавляет одну букву к понятию советского
                   2147: времени. Назовите получившийся в результате неологизм.
                   2148: 
                   2149: Ответ:
                   2150: Политбюрок.
                   2151: 
                   2152: Источник:
                   2153: http://www.km.ru/muzyka/2016/03/18/persony-i-sobytiya-v-mire-muzyki/773707-gromyka-gromyka
                   2154: 
                   2155: Автор:
                   2156: Егор Сидорович ("Семь сорок")
                   2157: 
                   2158: Вопрос 28:
                   2159: ЕГО старшего брата, который погиб на фронтах Второй мировой войны, звали
                   2160: Маркс. В 2007 году ОН, в числе других ученых, подписал открытое письмо
                   2161: Владимиру Путину против клерикализации общества. Назовите ЕГО.
                   2162: 
                   2163: Ответ:
                   2164: [Жорес] Алфёров.
                   2165: 
                   2166: Комментарий:
                   2167: Будущего физика родители тоже назвали в честь известного коммуниста -
                   2168: Жана Жореса. Алфёров - физик, поэтому его борьба с клерикализмом
                   2169: выглядит вполне органично.
                   2170: 
                   2171: Источник:
                   2172: https://ru.wikipedia.org/wiki/Алфёров,_Жорес_Иванович
                   2173: 
                   2174: Автор:
                   2175: Павел Свердлов ("Семь сорок")
                   2176: 
                   2177: Вопрос 29:
                   2178: Российский антифашист Петр Силаев вспоминал, как в школе тайком прочитал
                   2179: одну книгу, распечатав ее на принтере. Помимо драк в барах и суровых
                   2180: военных сцен, внимание в книге уделялось подавлению одной советской
                   2181: республики. По мнению Силаева, это событие сыграло едва ли не поворотную
                   2182: роль в мировоззрении автора. В каком городе располагалась столица этой
                   2183: республики?
                   2184: 
                   2185: Ответ:
                   2186: В Мюнхене.
                   2187: 
                   2188: Комментарий:
                   2189: В вопросе речь идет о книге "Майн Кампф", которую Силаев, желая "изучить
                   2190: врага", тайно распечатал в школьном кабинете информатики. В книге Гитлер
                   2191: уделял внимание разгрому в 1919 году Баварской советской Республики, чье
                   2192: краткое существование стало для будущего лидера нацистов доказательством
                   2193: реальности советско-еврейской угрозы.
                   2194: 
                   2195: Источник:
                   2196: https://daily.afisha.ru/archive/vozduh/books/8-knig-o-separatizme/
                   2197: 
                   2198: Автор:
                   2199: Станислав Габрусевич ("Семь сорок")
                   2200: 
                   2201: Вопрос 30:
                   2202: ТАКОЙ ОН, изображенный на рекламе мобильного оператора, вызывает
                   2203: ассоциации с двумя крупными конкурирующими сегментами рынка гаджетов. Из
                   2204: рекламы следует, что первый взнос за смартфон составит ноль рублей. Что
                   2205: мы заменили словами "ТАКОЙ ОН"?
                   2206: 
                   2207: Ответ:
                   2208: Надкусанный пончик.
                   2209: 
                   2210: Зачет:
                   2211: Надкусанный донат.
                   2212: 
                   2213: Комментарий:
                   2214: Ассоциации сразу и с логотипом "Apple", и с ОС Android (Android 1.6
                   2215: назывался "Donut", и вообще все версии Android называются в честь
                   2216: чего-нибудь сладенького).
                   2217: 
                   2218: Источник:
                   2219: Реклама velcom в Минском метро.
                   2220: 
                   2221: Автор:
                   2222: Павел Свердлов ("Семь сорок")
                   2223: 
                   2224: Вопрос 31:
                   2225: В вопросе есть замена.
                   2226:    Журнал "Arzamas" рассказывает о том, как торговцы из одной страны
                   2227: заметили, что рыба лучше сохраняется, если пересыпать ее ИКСОМ. Поначалу
                   2228: ИКС выбрасывали, как использованную упаковку, но потом нашли лучшее
                   2229: применение. Мы не спрашиваем, что мы заменили ИКСОМ. Ответьте, в какой
                   2230: стране это происходило.
                   2231: 
                   2232: Ответ:
                   2233: Япония.
                   2234: 
                   2235: Комментарий:
                   2236: Торговцы из японского портового города Осака возили в удаленные места
                   2237: рыбу, для большей сохранности пересыпанную ферментированным рисом. Со
                   2238: временем рыбно-рисовую смесь стали есть, что впоследствии
                   2239: трансформировалось в суши.
                   2240: 
                   2241: Источник:
                   2242: https://arzamas.academy/materials/745
                   2243: 
                   2244: Автор:
                   2245: Станислав Габрусевич ("Семь сорок")
                   2246: 
                   2247: Вопрос 32:
                   2248: Психолог Гелена Савицкая проводит аналогии между людьми и архитектурными
                   2249: элементами. С чем она сравнивает женщин, которые много на себя берут и
                   2250: пытаются всё контролировать, но реальной пользы почти не приносят?
                   2251: 
                   2252: Ответ:
                   2253: С кариатидой.
                   2254: 
                   2255: Комментарий:
                   2256: В реальном мире кариатида пользы приносит куда меньше, чем можно
                   2257: предположить по виду.
                   2258: 
                   2259: Источник:
                   2260: https://gelena-s.livejournal.com/440983.html
                   2261: 
                   2262: Автор:
                   2263: Ольга Потапова ("Семь сорок")
                   2264: 
                   2265: Вопрос 33:
                   2266: Спортивный журналист Кирилл Новокщёнов сравнил ИКС хоккеиста Наиля
                   2267: Якупова со среднегодовой температурой на Аляске. Назовите ИКС двумя
                   2268: словами, начинающимися на одну и ту же букву.
                   2269: 
                   2270: Ответ:
                   2271: Показатель полезности.
                   2272: 
                   2273: Зачет:
                   2274: Показатель плюс-минус.
                   2275: 
                   2276: Комментарий:
                   2277: Среднегодовая температура на Аляске - так себе.
                   2278: 
                   2279: Источник:
                   2280: https://www.sports.ru/tribuna/blogs/thethinblueline/1075534.html
                   2281: 
                   2282: Автор:
                   2283: Алексей Евдоченко ("Семь сорок")
                   2284: 
                   2285: Вопрос 34:
                   2286: Сотрудница Третьяковской галереи Ирина Вакар рассказывает, что ОН
                   2287: предлагал разные интерпретации своего произведения, в том числе и в
                   2288: иронической манере. Этим можно объяснить наличие под слоем краски
                   2289: надписи, отсылающей к картине французского художника. Кто фигурирует в
                   2290: названии этой картины?
                   2291: 
                   2292: Ответ:
                   2293: Негры.
                   2294: 
                   2295: Комментарий:
                   2296: В конце 2015 года сотрудники Третьяковской галереи, проводившие
                   2297: исследования "Черного квадрата" Малевича, обнаружили, что то, что
                   2298: считалось ранее подписью художника, является надписью "битва негров
                   2299: ночью". Надпись отсылает к картине эксцентричного французского художника
                   2300: Альфонса Алле "Битва негров в пещере глубокой ночью". Долгое время связь
                   2301: "Черного квадрата" и "Битвы негров..." расценивалась как исторический
                   2302: анекдот. Теперь же стало известно, что Малевич как минимум знал про
                   2303: картину Алле.
                   2304: 
                   2305: Источник:
                   2306: https://meduza.io/feature/2015/11/19/kazimir-malevich-v-peschere-glubokoy-nochyu
                   2307: 
                   2308: Автор:
                   2309: Станислав Габрусевич ("Семь сорок")
                   2310: 
                   2311: Вопрос 35:
                   2312: Сравнивая бизнес-тренинги, необходимые для того, чтобы не отстать от
                   2313: конкурентов, с тренировкой на беговой дорожке, Райля Фофанова упоминает
                   2314: реплику из книги, изданной в 1871 году. Назовите персонажа, который
                   2315: произносит эту реплику.
                   2316: 
                   2317: Ответ:
                   2318: Черная королева.
                   2319: 
                   2320: Комментарий:
                   2321: Она говорила, что "приходится бежать со всех ног, чтобы только остаться
                   2322: на том же месте". Совсем как на беговой дорожке. А у нас в пакете два
                   2323: вопроса подряд про черный цвет.
                   2324: 
                   2325: Источник:
                   2326:    1. "Бизнес-журнал", 2007/18: Архангельская область.
                   2327: https://books.google.ru/books?id=_hgOCgAAQBAJ&pg=PA16#v=onepage&q&f=false
                   2328:    2. https://en.wikipedia.org/wiki/Through_the_Looking-Glass
                   2329: 
                   2330: Автор:
                   2331: Александра Бурчалова, Павел Свердлов ("Семь сорок")
                   2332: 
                   2333: Вопрос 36:
                   2334: Согласно одному толкованию, долгожданное событие произойдет не раньше,
                   2335: чем все евреи вернутся в Израиль. Поэтому львовянин Павел Машиах шутит,
                   2336: что переедет на ПМЖ в Землю Обетованную... Каким?
                   2337: 
                   2338: Ответ:
                   2339: Последним.
                   2340: 
                   2341: Комментарий:
                   2342: Событие, о котором идет речь, - приход Мессии. Фамилия "Машиах"
                   2343: переводится с иврита как "Мессия", да и звучит похоже. Павел собирается
                   2344: переехать в Израиль с пафосом, а у нас этот вопрос, как и ответ, был
                   2345: последним.
                   2346: 
                   2347: Источник:
                   2348:    1. http://www.moshiach.ru/moshiach/KingMoshiach/28.html
                   2349:    2. Беседа автора вопроса с Павлом Машиахом.
                   2350: 
                   2351: Автор:
                   2352: Павел Свердлов ("Семь сорок")
                   2353: 
                   2354: Тур:
                   2355: 4 этап
                   2356: 
                   2357: Дата:
                   2358: 25-Feb-2017
                   2359: 
                   2360: Редактор:
                   2361: Иван Топчий и Павел Малецкий (Минск)
                   2362: 
                   2363: Инфо:
                   2364: Ведущие команды: "Хронически Разумные United", "Автостопом по
                   2365: Прибалтике", "Не вопрос", "Zемляне", "Ять". Редакторы благодарят за
                   2366: помощь и ценные советы: Валерия Семёнова, Дарью Соловей, Марию
                   2367: Петровскую, команды "Садик Хана Дзавы", "Конструктор" и "Кто все эти
                   2368: люди?".
                   2369: 
                   2370: Вопрос 1:
                   2371: [Ведущему: четко прочитать "и" в слове "Кастилтэка".]
                   2372:    ПЕРВЫЕ назвали ВТОРЫХ - КастилтЭка. Назовите и ПЕРВЫХ, и ВТОРЫХ.
                   2373: 
                   2374: Ответ:
                   2375: Ацтеки, испанцы.
                   2376: 
                   2377: Зачет:
                   2378: Ацетки, кастильцы/конкистадоры/конквистадоры. Незачет: Индейцы вместо
                   2379: ацтеков.
                   2380: 
                   2381: Комментарий:
                   2382: Испанцы всё еще называли свою страну Кастилией в честь центрального
                   2383: королевства.
                   2384: 
                   2385: Источник:
                   2386: https://www.reddit.com/r/AskHistorians/comments/5dtdbx/did_the_aztecs_understand_where_the_spanish_were/da7wb0t/
                   2387: 
                   2388: Автор:
                   2389: Алексей Волчок ("Хронически разумные United")
                   2390: 
                   2391: Вопрос 2:
                   2392:    <раздатка>
                   2393:    "__________, My D&eacute;j&agrave; Vu"
                   2394:    </раздатка>
                   2395:    Перед вами название одного из эпизодов телесериала "Клиника".
                   2396: Напишите три слова, которые мы пропустили в этом названии.
                   2397: 
                   2398: Ответ:
                   2399: My D&eacute;j&agrave; Vu.
                   2400: 
                   2401: Зачет:
                   2402: По словам "D&eacute;j&agrave; Vu".
                   2403: 
                   2404: Комментарий:
                   2405: Серия называется "My D&eacute;j&agrave; Vu, My D&eacute;j&agrave; Vu".
                   2406: 
                   2407: Источник:
                   2408: https://en.wikipedia.org/wiki/List_of_Scrubs_episodes
                   2409: 
                   2410: Автор:
                   2411: Павел Малецкий ("Хронически разумные United")
                   2412: 
                   2413: Вопрос 3:
                   2414: В средневековой Англии служители церкви имели право избежать светского
                   2415: суда и тем самым смертной казни. Начиная с XV века под действие этой
                   2416: привилегии стали попадать многие аристократы, а иногда даже люди низших
                   2417: сословий. Положение было отменено только в 1827 году, когда ДЕЛАТЬ ЭТО
                   2418: могла уже бОльшая часть населения. Что именно делать?
                   2419: 
                   2420: Ответ:
                   2421: Читать.
                   2422: 
                   2423: Зачет:
                   2424: Писать.
                   2425: 
                   2426: Комментарий:
                   2427: До появления книгопечатания читать умело только духовенство, поэтому
                   2428: считалось, что все, кто умеют читать - духовенство.
                   2429: 
                   2430: Источник:
                   2431: А.Ф. Кистяковский. Исследования о смертной казни. - М: Директ-Медиа,
                   2432: 2010. - С. 148.
                   2433: 
                   2434: Автор:
                   2435: Елизавета Лысенко ("Не вопрос")
                   2436: 
                   2437: Вопрос 4:
                   2438: [Ведущему: обязательно прочитать комментарий!]
                   2439:    В нижней части картины Василия Балабанова "Пловец" можно увидеть как
                   2440: бы отраженное в воде известное архитектурное сооружение. Говоря о своей
                   2441: картине, Василий Балабанов упомянул Второе пришествие Христа. Какие два
                   2442: слова мы пропустили в предыдущем предложении?
                   2443: 
                   2444: Ответ:
                   2445: Храм, Спасителя.
                   2446: 
                   2447: Комментарий:
                   2448: По замыслу автора Храм Христа Спасителя как бы отражается в водах
                   2449: бассейна "Москва", который изображен в верхней части картины.
                   2450: 
                   2451: Источник:
                   2452: https://ru.wikipedia.org/wiki/Пловец_(картина)
                   2453: 
                   2454: Автор:
                   2455: Павел Малецкий, Герман Чепиков ("Хронически разумные United")
                   2456: 
                   2457: Вопрос 5:
                   2458:    <раздатка>
                   2459:    Дело объяснялось очень просто: "король" - было только такое слово, и
                   2460: больше ничего. Никакой власти королю не давалось. Всякий делал, что
                   2461: хотел, и это называлось DURA LEX... И частенько по ночам плакали короли
                   2462: в подушку от этого DURA LEX...
                   2463:    </раздатка>
                   2464:    Перед вами цитата из пародийной "Всеобщей истории, обработанной
                   2465: "Сатириконом"". Что мы заменили словами "dura lex" [дУра лекс]?
                   2466: 
                   2467: Ответ:
                   2468: Liberum veto.
                   2469: 
                   2470: Зачет:
                   2471: Либерум вето.
                   2472: 
                   2473: Комментарий:
                   2474: Система права Речи Посполитой, позволявшая любому депутату Сейма
                   2475: наложить вето на общее решение. Королевская власть в Речи Посполитой
                   2476: была крайне слаба.
                   2477: 
                   2478: Источник:
                   2479: Всеобщая история, обработанная "Сатириконом".
                   2480: http://az.lib.ru/a/awerchenko_a_t/text_0120.shtml
                   2481: 
                   2482: Автор:
                   2483: Иван Топчий ("Хронически разумные United")
                   2484: 
                   2485: Вопрос 6:
                   2486: Британец Алекс Янгер в интервью заявил, что благодарен ЕМУ за
                   2487: популярность и разделяет ЕГО любовь к техническим новинкам. Тем не
                   2488: менее, Янгер признаётся, что никогда не нанял бы ЕГО на работу из-за
                   2489: опрометчивости и аморальности. Мы не просим вас назвать ЕГО. Назовите
                   2490: организацию, которую возглавляет Янгер.
                   2491: 
                   2492: Ответ:
                   2493: МИ-6.
                   2494: 
                   2495: Зачет:
                   2496: MI6; Секретная разведывательная служба; Secret Intelligence Service;
                   2497: Военная разведка; Military Intelligence.
                   2498: 
                   2499: Комментарий:
                   2500: Таким образом глава британской спецслужбы отозвался о Джеймсе Бонде.
                   2501: 
                   2502: Источник:
                   2503: https://www.theguardian.com/uk-news/2016/dec/08/james-bond-would-not-get-job-with-real-mi6-says-spy-chief
                   2504: 
                   2505: Автор:
                   2506: Алексей Волчок ("Хронически разумные United")
                   2507: 
                   2508: Вопрос 7:
                   2509: Современники отмечали, что в ЕГО квартире повсюду можно было увидеть
                   2510: чертежи, комки глины, грязь и даже навоз. Назовите ЕГО фамилию.
                   2511: 
                   2512: Ответ:
                   2513: Клодт.
                   2514: 
                   2515: Комментарий:
                   2516: Своих знаменитых лошадей он изображал с натуры, для чего приводил их в
                   2517: свою комнату.
                   2518: 
                   2519: Источник:
                   2520: http://www.cablook.com/design-art/lyubimye-loshadi-barona-klodta/
                   2521: 
                   2522: Автор:
                   2523: Сергей Поляков ("Summa Technologiae"), в редакции Павла Малецкого
                   2524: ("Хронически разумные United")
                   2525: 
                   2526: Вопрос 8:
                   2527: Описывая поездку Пушкина по Заволжью в 1833 году, современный критик
                   2528: Алексей Иванов так отозвался о слове, впервые услышанном поэтом: "Слово,
                   2529: пойманное в степи, Пушкин добросил до орбиты Земли". Назовите это слово.
                   2530: 
                   2531: Ответ:
                   2532: Буран.
                   2533: 
                   2534: Комментарий:
                   2535: В поездке Пушкин собирал материал о пугачёвском восстании и дивное слово
                   2536: запомнил: "Ну барин", - закричал ямщик, - "беда: буран!" ("Капитанская
                   2537: дочка"). А Иванов намекает на аэрокосмический челнок "Буран".
                   2538: 
                   2539: Источник:
                   2540: А.В. Иванов. Вилы.
                   2541: https://books.google.ru/books?id=TTlCDQAAQBAJ&pg=PT19#v=onepage&q&f=false
                   2542: 
                   2543: Автор:
                   2544: Надежда Лейчинская ("Ять")
                   2545: 
                   2546: Вопрос 9:
                   2547: В юности баскетболист Джеральд Грин тренировал броски сверху, используя
                   2548: некачественные самодельные баскетбольные кольца. В 2007 году Грин стал
                   2549: первым в истории ТАКИМ победителем конкурса слэм-данков на матче всех
                   2550: звезд НБА. Назовите любой сериал, большинство героев которого - ТАКИЕ.
                   2551: 
                   2552: Ответ:
                   2553: "Симпсоны".
                   2554: 
                   2555: Зачет:
                   2556: "Футурама"; "Гриффины".
                   2557: 
                   2558: Комментарий:
                   2559: ТАКИЕ - это четырехпалые. Джеральд Грин потерял один из пальцев на руке,
                   2560: когда забил сверху в самодельное кольцо, из которого торчал шуруп. У
                   2561: большинства персонажей "Симпсонов" (кроме Бога и Иисуса) на всех
                   2562: конечностях по четыре пальца.
                   2563: 
                   2564: Источник:
                   2565:    1. https://shkola2-0.ru/ucheniku/lifestyle/bullying
                   2566:    2. https://en.wikipedia.org/wiki/List_of_recurring_The_Simpsons_characters
                   2567: 
                   2568: Автор:
                   2569: Валентин Копочель ("Автостопом по Прибалтике")
                   2570: 
                   2571: Вопрос 10:
                   2572: Светлокожий баскетболист Джейсон Уильямс за уличный стиль игры и частые
                   2573: нарушения дисциплины получил в НБА прозвище ИКС. Впервые ИКС был выпущен
                   2574: в 1930-х годах в Швейцарии. Какие два слова мы заменили ИКСОМ?
                   2575: 
                   2576: Ответ:
                   2577: Белый шоколад.
                   2578: 
                   2579: Комментарий:
                   2580: Джейсон Уильямс является белокожим, что не мешало ему своим поведением
                   2581: походить на типичного афроамериканца.
                   2582: 
                   2583: Источник:
                   2584:    1. https://www.sports.ru/tribuna/blogs/spursmania/861325.html
                   2585:    2. https://ru.wikipedia.org/wiki/Белый_шоколад
                   2586: 
                   2587: Автор:
                   2588: Валентин Копочель ("Автостопом по Прибалтике")
                   2589: 
                   2590: Вопрос 11:
                   2591: [Ведущему: обязательно прочитать комментарий!]
                   2592:    Рассказывая о послевоенных годах, Иван Лихач приводит в пример
                   2593: человека по фамилии Мычко, который знал пять ИХ, а работал председателем
                   2594: сельсовета. Назовите любую из этих пяти ИХ.
                   2595: 
                   2596: Ответ:
                   2597: М.
                   2598: 
                   2599: Зачет:
                   2600: ы; ч; к; о.
                   2601: 
                   2602: Комментарий:
                   2603: Мычко знал только, как писать собственную фамилию. Сам Лихач закончил
                   2604: четыре класса школы и считался человеком грамотным.
                   2605: 
                   2606: Источник:
                   2607: https://news.tut.by/society/488638.html
                   2608: 
                   2609: Автор:
                   2610: Алексей Гончаров ("Хронически разумные United")
                   2611: 
                   2612: Вопрос 12:
                   2613: [Ведущему: обязательно прочитать комментарий!]
                   2614:    Прибывший после войны в родную деревню Иван Лихач был назначен ЕЕ
                   2615: руководителем. Поскольку на тот момент ОНА была пустой комнатой, можно
                   2616: сказать, что ОНА дважды не соответствовала своему названию. Назовите ЕЕ
                   2617: словом, пишущимся через дефис.
                   2618: 
                   2619: Ответ:
                   2620: Изба-читальня.
                   2621: 
                   2622: Комментарий:
                   2623: Как вы уже знаете, Иван Лихач слыл человеком грамотным. В комнате не
                   2624: было книг и газет, поэтому сложно было назвать ее как избой, так и
                   2625: читальней. Заканчивая историю про Ивана Лихача, можно сказать, что
                   2626: впоследствии он сделал удачную карьеру, в конце которой стал первым
                   2627: секретарем Центризбиркома независимой Беларуси.
                   2628: 
                   2629: Источник:
                   2630: https://news.tut.by/society/488638.html
                   2631: 
                   2632: Автор:
                   2633: Алексей Гончаров ("Хронически разумные United")
                   2634: 
                   2635: Вопрос 13:
                   2636: (pic: 20161024.jpg)
                   2637:    Будущий вице-президент США Элбридж Герри применил методику,
                   2638: называемую теперь его именем. На картинке выделен один из НИХ в Чикаго.
                   2639: Назовите ЕГО двумя словами.
                   2640: 
                   2641: Ответ:
                   2642: Избирательный округ.
                   2643: 
                   2644: Зачет:
                   2645: Мажоритарный округ. Незачет: Избирательный участок.
                   2646: 
                   2647: Комментарий:
                   2648: Герри применил практику джерримандеринга - перераспределения территорий
                   2649: избирательных округов с целью манипуляции результатами выборов по
                   2650: мажоритарной системе. Так, 100 человек, из которых 70 белых и 30 черных,
                   2651: можно объединить в 10 равных групп так, что черные будут большинством в
                   2652: половине групп либо ни в одной. На рисунке один-единственный
                   2653: избирательный округ накрывает большинство районов Чикаго, где живут
                   2654: латиноамериканцы.
                   2655: 
                   2656: Источник:
                   2657:    1. https://en.wikipedia.org/wiki/Illinois's_4th_congressional_district
                   2658:    2. https://ru.wikipedia.org/wiki/Герри,_Элбридж_Томас
                   2659: 
                   2660: Автор:
                   2661: Алексей Волчок ("Хронически разумные United")
                   2662: 
                   2663: Вопрос 14:
                   2664: Известен курьезный случай, когда картина Анри Матисса "Лодка" провисела
                   2665: в нью-йоркском музее 47 дней вверх ногами, прежде чем ее догадались
                   2666: перевернуть. Назовите картину, которую в конце XX века предложил
                   2667: перевернуть Алексий II.
                   2668: 
                   2669: Ответ:
                   2670: "Пловец".
                   2671: 
                   2672: Комментарий:
                   2673: Картина Валерия Балабанова "Пловец" известна вам из четвертого вопроса.
                   2674: В 1994 году на месте бассейна "Москва" вновь началось возведение Храма
                   2675: Христа Спасителя. Произведение тоже на водную тематику, и отражение
                   2676: присутствует.
                   2677: 
                   2678: Источник:
                   2679:    1. https://en.wikipedia.org/wiki/Le_Bateau
                   2680:    2. https://ru.wikipedia.org/wiki/Пловец_(картина)
                   2681: 
                   2682: Автор:
                   2683: Герман Чепиков, Алексей Гончаров ("Хронически разумные United")
                   2684: 
                   2685: Вопрос 15:
                   2686: "Морской бой" - это один из самых популярных советских игровых
                   2687: автоматов. Существовала даже его специальная версия для подводных лодок
                   2688: под названием "Электронный тренажер". Она отличалась отсутствием
                   2689: выдвигающейся подставки для детей и... Чего еще?
                   2690: 
                   2691: Ответ:
                   2692: Прорези для монет.
                   2693: 
                   2694: Зачет:
                   2695: Монетоприемника.
                   2696: 
                   2697: Источник:
                   2698:    1. https://engineering-ru.livejournal.com/221481.html
                   2699:    2. https://relikva.com/@pgmuseum/r/igrovoy_avtomat_morskoy_boy
                   2700: 
                   2701: Автор:
                   2702: Надежда Лейчинская ("Ять")
                   2703: 
                   2704: Вопрос 16:
                   2705: В одном из положений итальянского избирательного закона 2015 года
                   2706: говорится о том, что молодые итальянцы, проживающие за пределами страны
                   2707: не менее трех месяцев с определенной целью, имеют право голосовать в так
                   2708: называемом "зарубежном округе". Это положение получило название
                   2709: "поправка ПРОПУСК". Заполните пропуск латинизированным именем.
                   2710: 
                   2711: Ответ:
                   2712: Erasmus.
                   2713: 
                   2714: Зачет:
                   2715: Эразмус.
                   2716: 
                   2717: Комментарий:
                   2718: Одна из приведенных целей - учеба. "Erasmus" - это некоммерческая
                   2719: программа Европейского союза по обмену студентами и преподавателями
                   2720: между университетами стран членов Евросоюза, а также латинизированная
                   2721: форма имени Эразм.
                   2722: 
                   2723: Источник:
                   2724: https://ru.wikipedia.org/wiki/Итальянский_избирательный_закон_(2015)
                   2725: 
                   2726: Автор:
                   2727: Александр Морозов ("Хронически разумные United")
                   2728: 
                   2729: Вопрос 17:
                   2730: В конце 1950-х годов в США активно искали способы использования ядерного
                   2731: оружия в мирных целях. Название одного из таких проектов можно перевести
                   2732: на русский язык устаревшим словом среднего рода. Назовите это слово.
                   2733: 
                   2734: Ответ:
                   2735: Орало.
                   2736: 
                   2737: Комментарий:
                   2738: Проект носил название "Plowshare", в переводе на русский - "лемех" или
                   2739: "орало" из библейской фразы "перековать мечи на орала".
                   2740: 
                   2741: Источник:
                   2742: https://republic.ru/posts/71669
                   2743: 
                   2744: Автор:
                   2745: Надежда Лейчинская ("Ять")
                   2746: 
                   2747: Вопрос 18:
                   2748: Дуплет.
                   2749:    1. На картинке в сообществе "Химик-психопат" ОН сообщает, что сделал
                   2750: тебя мастером высокоэффективной жидкостной хроматографии. Назовите ЕГО.
                   2751:    2. В ЕГО оригинальное название входит латинское наименование слона.
                   2752: Собственный корреспондент газеты "Аргументы и Факты" в Амстердаме
                   2753: утверждает, что ОН шершавый. Назовите ЕГО коротким словом.
                   2754: 
                   2755: Ответ:
                   2756:    1. Кот [в колпаке и с волшебной палочкой, который делает "вжух"].
                   2757:    2. Ждун.
                   2758: 
                   2759: Комментарий:
                   2760:    1. ВЖХ - и ты мастер высокоэффективной жидкостной хроматографии.
                   2761:    2. Оригинал скульптуры, которая в оригинале называется "Гомункулос
                   2762: Лаксодонтус", находится в Амстердаме. В его имя входит название
                   2763: африканского слона, а внешний вид во многом позаимствован у морского.
                   2764: 
                   2765: Источник:
                   2766:    1. https://vk.com/himik_psihopat?w=wall-29287308_291342
                   2767:    2. http://www.aif.ru/society/people/skulptura_stavshaya_memom_aif_navestil_zhduna
                   2768: 
                   2769: Автор:
                   2770: Иван Топчий ("Хронически разумные United"), Валерий Семёнов ("Любовь
                   2771: Каксон")
                   2772: 
                   2773: Вопрос 19:
                   2774: [Ведущему: обязательно прочитать комментарий!]
                   2775:    Герой современного сериала, полицейский, наблюдает за напарником
                   2776: через камеры в здании и дает ему по рации указания, в том числе
                   2777: издевательские. Когда напарник начинает игнорировать эти приказы, герой
                   2778: упоминает ПАРК ЮРСКОГО ПЕРИОДА. Что мы заменили словами "ПАРК ЮРСКОГО
                   2779: ПЕРИОДА"?
                   2780: 
                   2781: Ответ:
                   2782: "Мир Дикого Запада".
                   2783: 
                   2784: Зачет:
                   2785: "Западный мир"; "Westworld".
                   2786: 
                   2787: Комментарий:
                   2788: Герой шутит, что напарник - робот, вышедший из-под контроля, как в
                   2789: сериале "Мир Дикого Запада".
                   2790: 
                   2791: Источник:
                   2792: Телесериал "Бруклин 9-9", s04e11.
                   2793: 
                   2794: Автор:
                   2795: Иван Топчий ("Хронически разумные United")
                   2796: 
                   2797: Вопрос 20:
                   2798: В аниме "Ergo Proxy" [Эрго прОкси] описывается эпидемия вируса "Кот",
                   2799: который дает роботам самосознание. Какие три буквы мы пропустили в
                   2800: предыдущем предложении?
                   2801: 
                   2802: Ответ:
                   2803: г, и, о.
                   2804: 
                   2805: Зачет:
                   2806: g, i, o.
                   2807: 
                   2808: Комментарий:
                   2809: Вирус называется "КОгито" - одно из ключевых понятий в философии
                   2810: Декарта, обозначающее представление, мысль, желание. Знаменита фраза
                   2811: Декарта "Cogito ergo sum" [кОгито Эрго сум] - "мыслю, следовательно
                   2812: существую".
                   2813: 
                   2814: Источник:
                   2815:    1. https://en.wikipedia.org/wiki/Ergo_Proxy
                   2816:    2. https://ru.wikipedia.org/wiki/Cogito
                   2817: 
                   2818: Автор:
                   2819: Валентин Копочель ("Автостопом по Прибалтике")
                   2820: 
                   2821: Вопрос 21:
                   2822: Цитата из книги Рона Гарана: "Майк, на счету которого было уже три ИКСА,
                   2823: решил, что должен дать мне какой-нибудь совет: "Ронни, не смотри вниз" -
                   2824: вот всё, что пришло ему в голову". Конец цитаты. 380-й по счету ИКС был
                   2825: совершен 21 декабря 2015 года. Какие несколько слов мы заменили ИКСОМ?
                   2826: 
                   2827: Ответ:
                   2828: Выход в [открытый] космос.
                   2829: 
                   2830: Комментарий:
                   2831: Попробуй еще разбери в космосе, где низ, где верх.
                   2832: 
                   2833: Источник:
                   2834:    1. https://esquire.ru/ronald-garan
                   2835:    2. https://ru.wikipedia.org/wiki/Список_выходов_в_открытый_космос_с_351-го_(с_2012_года)
                   2836: 
                   2837: Автор:
                   2838: Надежда Лейчинская ("Ять")
                   2839: 
                   2840: Вопрос 22:
                   2841: Альбом "Limp Bizkit" [лимп бИскит] "Stampede of the Disco Elephants"
                   2842: [стэмпИд оф зэ дИско Элефантс] записывается с 2012 года. Англоязычная
                   2843: Википедия, говоря об альбоме, упоминает ЕЕ. Статья о возобновлении
                   2844: блокировки интернет-ресурсов после окончания Олимпиады называлась "ОНА
                   2845: закончилась". Назовите ЕЕ двумя словами.
                   2846: 
                   2847: Ответ:
                   2848: Китайская демократия.
                   2849: 
                   2850: Комментарий:
                   2851: Википедия проводит аналогию между альбомом "Limp Bizkit" и альбомом
                   2852: "Guns'n'Roses", которого фанатам пришлось ждать 12 лет. На время
                   2853: Олимпиады в Пекине по просьбе иностранных журналистов был открыт доступ
                   2854: к китайским версиям сайтов BBC и "ГОлоса Америки".
                   2855: 
                   2856: Источник:
                   2857:    1. https://en.wikipedia.org/wiki/Stampede_of_the_Disco_Elephants
                   2858:    2. http://os.colta.ru/news/details/6559/
                   2859: 
                   2860: Автор:
                   2861: Павел Белькевич ("Автостопом по Прибалтике")
                   2862: 
                   2863: Вопрос 23:
                   2864: (pic: 20161025.jpg)
                   2865:    Напишите русское название книги, часть обложки которой перед вами.
                   2866: 
                   2867: Ответ:
                   2868: "Игра в классики".
                   2869: 
                   2870: Комментарий:
                   2871: (pic: 20161026.jpg)
                   2872: 
                   2873: Источник:
                   2874: https://ru.wikipedia.org/wiki/Игра_в_классики_(роман)
                   2875: 
                   2876: Автор:
                   2877: Герман Чепиков ("Хронически разумные United")
                   2878: 
                   2879: Вопрос 24:
                   2880: Одна из современных теорий зарождения жизни предполагает появление
                   2881: первых самокопирующихся молекул РНК не в объеме водного раствора, как
                   2882: считали ранее, а в виде слоя на основании из глины. Такая модель носит
                   2883: название "ОНА". Назовите ЕЕ двумя словами, начинающимися на одну и ту же
                   2884: букву.
                   2885: 
                   2886: Ответ:
                   2887: Первичная пицца.
                   2888: 
                   2889: Комментарий:
                   2890: По аналогии с "первичным бульоном".
                   2891: 
                   2892: Источник:
                   2893: https://theoryandpractice.ru/posts/14132-nikitin
                   2894: 
                   2895: Автор:
                   2896: Иван Топчий ("Хронически разумные United")
                   2897: 
                   2898: Вопрос 25:
                   2899: Филолог Павел БеркОв был арестован в 1938 году по ЛОЖНОМУ обвинению в
                   2900: шпионаже. В надежде на оправдание он сочинил ФАЛЬШИВЫЕ показания. На
                   2901: какой заграничной улице были расположены его ВЫДУМАННЫЕ явки?
                   2902: 
                   2903: Ответ:
                   2904: Унтер ден Линден.
                   2905: 
                   2906: Комментарий:
                   2907: На языке узников того времени обвинения Беркову "липили", вот и
                   2908: показания он дал липовые. Унтер ден Линден - улица "Под липами".
                   2909: 
                   2910: Источник:
                   2911: http://scisne.net/a-1675
                   2912: 
                   2913: Автор:
                   2914: Елизавета Лысенко ("Не вопрос")
                   2915: 
                   2916: Вопрос 26:
                   2917:    <раздатка>
                   2918:    It's art But Is It Art
                   2919:    </раздатка>
                   2920:    В названии статьи, посвященной критике современного романа, мы дважды
                   2921: пропустили одну букву. Назовите автора этого романа.
                   2922: 
                   2923: Ответ:
                   2924: [Донна] Тартт.
                   2925: 
                   2926: Комментарий:
                   2927: Не все критики считают нашумевший роман "Щегол", поднимающий тему
                   2928: значимости искусства, настоящим искусством. Слова в заголовках на
                   2929: английском языке всегда пишутся с большой буквы, это могло служить
                   2930: подсказкой.
                   2931: 
                   2932: Источник:
                   2933: https://www.vanityfair.com/culture/2014/07/goldfinch-donna-tartt-literary-criticism
                   2934: 
                   2935: Автор:
                   2936: Елизавета Лысенко ("Не вопрос")
                   2937: 
                   2938: Вопрос 27:
                   2939: Индастриал-группа "Einst&uuml;rzende Neubauten" [айнштЮрценде
                   2940: нойбАутен], следующая принципам дадаизма, в 1980-е годы не могла
                   2941: позволить себе профессиональные инструменты и сделала своим девизом
                   2942: фразу "Будь умнее, воруй ТАМ". Часть действия фильма 1965 года
                   2943: происходит ТАМ. Назовите этот фильм.
                   2944: 
                   2945: Ответ:
                   2946: "Операция "Ы" и другие приключения Шурика".
                   2947: 
                   2948: Комментарий:
                   2949: ТАМ - это на стройке. Один из принципов дадаизма - это использование
                   2950: готовых объектов в новом ключе (например, строительных инструментов как
                   2951: музыкальных).
                   2952: 
                   2953: Источник:
                   2954: http://www.music-rock.ru/showthread.php?t=4311
                   2955: 
                   2956: Автор:
                   2957: Елизавета Лысенко ("Не вопрос")
                   2958: 
                   2959: Вопрос 28:
                   2960:    <раздатка>
                   2961:    бестиарий
                   2962:    </раздатка>
                   2963:    Еженедельно слушатели Радио Минск получают возможность насладиться
                   2964: коллекцией известных песен и редких композиций в передаче "BESTиарий". В
                   2965: названии передачи на раздаточном материале мы заменили часть букв.
                   2966: Восстановите это название в исходном виде.
                   2967: 
                   2968: Ответ:
                   2969: BESTиарий.
                   2970: 
                   2971: Зачет:
                   2972: С любой капитализацией.
                   2973: 
                   2974: Комментарий:
                   2975: Такая вот игра слов, оценить которую могут не все слушатели, а только
                   2976: те, кто посещал сайт радиостанции.
                   2977: 
                   2978: Источник:
                   2979: http://www.radiominsk.by/Program/GetDetails?composerId=125
                   2980: 
                   2981: Автор:
                   2982: Александр Курзинер ("Не вопрос")
                   2983: 
                   2984: Вопрос 29:
                   2985: В России XIX века Америка воспринималась как что-то очень далекое и
                   2986: практически несуществующее. Поэтому один из персонажей романа
                   2987: Достоевского сообщает, что собирается уехать в Америку, а затем ДЕЛАЕТ
                   2988: ЭТО. Какие слова мы заменили словами "ДЕЛАЕТ ЭТО"?
                   2989: 
                   2990: Ответ:
                   2991: Совершает самоубийство.
                   2992: 
                   2993: Зачет:
                   2994: Убивает себя, совершает суицид и т.п. по смыслу. Незачет: Умирает.
                   2995: 
                   2996: Комментарий:
                   2997: Америка была настолько далеко, что "уехать в Америку" было равносильно
                   2998: выражению "отправиться на тот свет", что и делает Свидригайлов в конце
                   2999: "Преступления и наказания".
                   3000: 
                   3001: Источник:
                   3002: https://magisteria.ru/silver-age/russkij-simvolizm/
                   3003: 
                   3004: Автор:
                   3005: Елизавета Лысенко ("Не вопрос")
                   3006: 
                   3007: Вопрос 30:
                   3008: На одном форуме представительница определенной профессии рассказала, что
                   3009: не боится, а наоборот, очень любит ЕЕ, поскольку в это время ей ничего
                   3010: не нужно делать. Слово "ОНА" происходит от латинского прилагательного со
                   3011: значением "бурный". Мы не спрашиваем вас профессию женщины. Какое слово
                   3012: мы заменили словом "ОНА"?
                   3013: 
                   3014: Ответ:
                   3015: Турбулентность.
                   3016: 
                   3017: Комментарий:
                   3018: Профессия этой женщины - стюардесса. В периоды сильной турбулентности ей
                   3019: не нужно обслуживать пассажиров самолета.
                   3020: 
                   3021: Источник:
                   3022:    1. https://fishki.net/2022873-chto-stjuardessy-dumajut-o-passazhirah-anonimnye-priznanija.html
                   3023:    2. https://ru.wikipedia.org/wiki/Турбулентность
                   3024: 
                   3025: Автор:
                   3026: Павел Малецкий ("Хронически разумные United")
                   3027: 
                   3028: Вопрос 31:
                   3029: В романе "Последний сейм Речи Посполитой" избранное общество
                   3030: наслаждается роскошным приемом, который заканчивается фейерверком.
                   3031: Увидев, что огни фейерверка начинают складываться в буквы, один из
                   3032: героев романа мрачно произносит несколько слов. Кому, как считается, эти
                   3033: слова были адресованы задолго до этого?
                   3034: 
                   3035: Ответ:
                   3036: Валтасару.
                   3037: 
                   3038: Комментарий:
                   3039: Аллюзия на библейский сюжет "Пир Валтасара", когда вавилонский царь
                   3040: беспечно пирует, не зная, что дни его царства сочтены. Слова, о которых
                   3041: идет речь в вопросе: "мене, текел, упарсин".
                   3042: 
                   3043: Источник:
                   3044: В. Реймонт. Последний сейм Речи Посполитой.
                   3045: http://flibusta.is/b/472789/read
                   3046: 
                   3047: Автор:
                   3048: Павел Митар ("Ять")
                   3049: 
                   3050: Вопрос 32:
                   3051: В уже известном вам сообществе "Химик-психопат" была опубликована
                   3052: фотография монаршей семьи азиатской страны. Назовите эту страну.
                   3053: 
                   3054: Ответ:
                   3055: Бутан.
                   3056: 
                   3057: Комментарий:
                   3058: Вряд ли другая страна заинтересовала бы химиков.
                   3059: 
                   3060: Источник:
                   3061: https://vk.com/himik_psihopat?w=wall-29287308_298505
                   3062: 
                   3063: Автор:
                   3064: Иван Топчий ("Хронически разумные United")
                   3065: 
                   3066: Вопрос 33:
                   3067: В начале XIX века спички были не слишком удобны в использовании: они
                   3068: могли сильно вспыхивать, искрить и из-за своего химического состава
                   3069: неприятно пахли. Назовите имя с двумя латинскими корнями, от которого
                   3070: произошло сленговое название таких спичек.
                   3071: 
                   3072: Ответ:
                   3073: Люцифер.
                   3074: 
                   3075: Комментарий:
                   3076: Спички-люциферики несут свет и адски пахнут серой.
                   3077: 
                   3078: Источник:
                   3079:    1. https://en.wikipedia.org/wiki/Match
                   3080:    2. https://ru.wikipedia.org/wiki/Люцифер
                   3081: 
                   3082: Автор:
                   3083: Надежда Лейчинская ("Ять")
                   3084: 
                   3085: Вопрос 34:
                   3086: (pic: 20161027.jpg)
                   3087:    При написании картины "Третье мая 1808 года в Мадриде" Франсиско де
                   3088: Гойя решил ДАТЬ КАРТ-БЛАНШ одному из героев, пренебрегая правилами
                   3089: казней тех времен. Какие два слова, начинающиеся на одну и ту же букву,
                   3090: мы заменили словами "ДАТЬ КАРТ-БЛАНШ"?
                   3091: 
                   3092: Ответ:
                   3093: Развязать руки.
                   3094: 
                   3095: Источник:
                   3096: https://ru.wikipedia.org/wiki/Третье_мая_1808_года_в_Мадриде
                   3097: 
                   3098: Автор:
                   3099: Герман Чепиков ("Хронически разумные United")
                   3100: 
                   3101: Вопрос 35:
                   3102: В списках самых некрасивых зданий мира фигурирует и Национальная
                   3103: библиотека в Буэнос-Айресе. Назовите человека, ответственного за
                   3104: реализацию проекта ее постройки.
                   3105: 
                   3106: Ответ:
                   3107: [Хорхе Луис] Борхес.
                   3108: 
                   3109: Комментарий:
                   3110: На момент выбора проекта Борхес был уже почти слепым.
                   3111: 
                   3112: Источник:
                   3113: https://realt.onliner.by/2016/04/16/brutal
                   3114: 
                   3115: Автор:
                   3116: Николай Будник ("Zемляне")
                   3117: 
                   3118: Вопрос 36:
                   3119: "Мир Дикого Запада" - фильм 1976 года по сценарию Майкла Крайтона,
                   3120: рассказывающий о том, как в высокотехнологичном развлекательном
                   3121: комплексе экспонаты-андроиды выходят из-под контроля и нападают на
                   3122: посетителей. Назовите фильм, снятый по сценарию Крайтона около двадцати
                   3123: лет спустя.
                   3124: 
                   3125: Ответ:
                   3126: "Парк Юрского периода".
                   3127: 
                   3128: Комментарий:
                   3129: Только вместо андроидов - динозавры. Замена в девятнадцатом вопросе была
                   3130: неслучайной.
                   3131: 
                   3132: Источник:
                   3133: https://ru.wikipedia.org/wiki/Западный_мир_(фильм)
                   3134: 
                   3135: Автор:
                   3136: Надежда Лейчинская ("Ять")
                   3137: 
                   3138: Тур:
                   3139: 5 этап
                   3140: 
                   3141: Дата:
                   3142: 18-Mar-2017
                   3143: 
                   3144: Редактор:
                   3145: Сергей Дубелевич (Минск)
                   3146: 
                   3147: Инфо:
                   3148: Ведущие команды: "Middle", "Корпрусариум", "Легионеры Боливарии",
                   3149: "Фунтики", "Хунвейбины".
                   3150: 
                   3151: Вопрос 1:
                   3152: [Ведущему: выделить слово "приветствует".]
                   3153:    Необразованный герой повести Роберта Колотухина приветствует политику
                   3154: Гитлера. В конце своего обращения к жильцам дома он употребляет слово из
                   3155: трех букв. Какое?
                   3156: 
                   3157: Ответ:
                   3158: "Хай".
                   3159: 
                   3160: Комментарий:
                   3161: Вместо "Хайль Гитлер!" герой пишет: "Хай Гитлер!". "Хай" по-украински и
                   3162: по-белорусски - "пусть". Также по звучанию совпадает с английским "hi" -
                   3163: "привет".
                   3164: 
                   3165: Источник:
                   3166: Р.В. Колотухин. Наш дом стоит у моря. http://flibusta.is/b/321080/read
                   3167: 
                   3168: Автор:
                   3169: Сергей Дубелевич ("Middle")
                   3170: 
                   3171: Вопрос 2:
                   3172: На сайте extreme-hunter.ru [экстрИм хАнтер точка ру] ОНА обойдется вам
                   3173: примерно в 200 долларов, однако 150 лет назад ОНА обошлась в 36 тысяч
                   3174: раз дороже. Назовите ЕЕ одним словом.
                   3175: 
                   3176: Ответ:
                   3177: Аляска.
                   3178: 
                   3179: Комментарий:
                   3180: В первом случае имелась в виду куртка.
                   3181: 
                   3182: Источник:
                   3183:    1. http://www.extreme-hunter.ru/shop/odezhda/kurtki/alyaska/
                   3184:    2. https://ru.wikipedia.org/wiki/Продажа_Аляски
                   3185: 
                   3186: Автор:
                   3187: Иван Зайков ("Легионеры Боливарии")
                   3188: 
                   3189: Вопрос 3:
                   3190: Житель индийского Телангана по имени Рамеш бросил колледж и устроился
                   3191: работать на фармацевтический завод. Через некоторое время его
                   3192: арестовали. На сайте Lenta.ru говорится, что Рамеш получил прозвище
                   3193: "теланганский Кларк Кент". Назовите имя и фамилию, которые мы заменили
                   3194: словами "Кларк Кент".
                   3195: 
                   3196: Ответ:
                   3197: Уолтер Уайт.
                   3198: 
                   3199: Комментарий:
                   3200: Рамеш выносил с фармацевтического завода необходимые препараты и варил
                   3201: дома амфетамин. За это он получил прозвище Теланганский Уолтер Уайт.
                   3202: Уолтер Уайт является главным героем сериала "Во все тяжкие". Имя одного
                   3203: героя мы заменили именем другого, которое также начинается на одинаковые
                   3204: буквы.
                   3205: 
                   3206: Источник:
                   3207: https://lenta.ru/articles/2017/02/10/india_frugs/
                   3208: 
                   3209: Автор:
                   3210: Василий Бобков ("Корпрусариум")
                   3211: 
                   3212: Вопрос 4:
                   3213: Героиня мультсериала "Барбоскины" вынуждена во всём угождать младшему
                   3214: брату. В одном из эпизодов она танцует. Ответьте тремя словами: что в
                   3215: это время делает малыш?
                   3216: 
                   3217: Ответ:
                   3218: Играет на дудке.
                   3219: 
                   3220: Зачет:
                   3221: Дудит в дудку; дует в дудку.
                   3222: 
                   3223: Комментарий:
                   3224: Вероятно, обыгрывается выражение "плясать под чужую дудку".
                   3225: 
                   3226: Источник:
                   3227: Мультсериал "Барбоскины", 31-я серия.
                   3228: 
                   3229: Автор:
                   3230: Сергей Дубелевич ("Middle")
                   3231: 
                   3232: Вопрос 5:
                   3233: Внимание, в вопросе есть замена.
                   3234:    В первой половине XX века выпускалась ТАКАЯ косметика. Настоящую
                   3235: славу ей принес постер, на котором женщина буквально светится от средств
                   3236: марки. Что мы заменили словом "ТАКАЯ"?
                   3237: 
                   3238: Ответ:
                   3239: Радиоактивная.
                   3240: 
                   3241: Комментарий:
                   3242: Средства содержали одновременно бромид радия и хлорид тория - сразу два
                   3243: радиоактивных соединения. Кремы, пудры, помады и зубные пасты
                   3244: "Tho-Radia" продавались до начала 1960 годов, когда популярность
                   3245: косметики с радием наконец сошла на нет.
                   3246: 
                   3247: Источник:
                   3248: https://daily.afisha.ru/beauty/3434-nogi-lotosy-kraska-ubiyca-i-drugie-uzhasy-mira-krasoty/
                   3249: 
                   3250: Автор:
                   3251: Елена Гордынец ("Легионеры Боливарии")
                   3252: 
                   3253: Вопрос 6:
                   3254: Дуплет.
                   3255:    1. В одном из эпизодов сериала клиент Шерлока Холмса, прогуливаясь с
                   3256: ним по городу, замечает, что за ними наблюдают, и произносит фразу из
                   3257: произведения XX века. Назовите автора этого произведения.
                   3258:    2. "Аутсайдер супротив тоталитарщины" - так один из
                   3259: интернет-пользователей охарактеризовал основные события фильма
                   3260: "Альфавиль", события которого происходят в XX веке. А в каком году?
                   3261: 
                   3262: Ответ:
                   3263:    1. [Джордж] Оруэлл.
                   3264:    2. 1984.
                   3265: 
                   3266: Комментарий:
                   3267:    1. В сериале за Шерлоком Холмсом часто следит его брат Майкрофт.
                   3268: Клиент произносит фразу: "Большой брат следит за вами?".
                   3269:    2. "Альфавиль" - фильм-антиутопия, где город Альфавиль находится под
                   3270: контролем профессора фон Брауна. В городе объявлены вне закона все
                   3271: гуманные человеческие чувства.
                   3272: 
                   3273: Источник:
                   3274:    1. https://www.afisha.ru/movie/166845/review/363000/
                   3275:    2. https://kinanet.livejournal.com/1327442.html
                   3276:    3. Телесериал "Шерлок", s04e02, 20-я минута.
                   3277: 
                   3278: Автор:
                   3279: Василий Бобков ("Корпрусариум")
                   3280: 
                   3281: Вопрос 7:
                   3282: В вопросе есть замены.
                   3283:    В фильме "Бегущий по лезвию" полицейский, описывая героиню-андроида,
                   3284: причастную к убийству людей, называет ее "Леди и бродягой". Какие слова
                   3285: мы заменили словами "леди" и "бродяга"?
                   3286: 
                   3287: Ответ:
                   3288: Красавица и чудовище.
                   3289: 
                   3290: Комментарий:
                   3291: Название одного голливудского мультфильма мы заменили названием другого.
                   3292: 
                   3293: Источник:
                   3294: Х/ф "Бегущий по лезвию" (1981), реж. Ридли Скотт.
                   3295: 
                   3296: Автор:
                   3297: Команда "Фунтики"
                   3298: 
                   3299: Вопрос 8:
                   3300:    <раздатка>
                   3301:    - Так, значит, это и есть речка?
                   3302:    - Не речка, а река, - поправил его дядюшка Рэт, - а точнее, Река с
                   3303: большой буквы, понимаешь?
                   3304:    </раздатка>
                   3305:    В оригинальной ответной реплике героя повести Кеннета Грэма "Ветер в
                   3306: ивах" было всего два слова. Назовите первое из них.
                   3307: 
                   3308: Ответ:
                   3309: The.
                   3310: 
                   3311: Комментарий:
                   3312: Таким образом переводчик передал смысл реплики с отсутствующим в русском
                   3313: языке артиклем the. В оригинале диалог звучал так:
                   3314:    - So this is a river!
                   3315:    - THE River.
                   3316: 
                   3317: Источник:
                   3318:    1. К. Грэм. Ветер в ивах (перевод И.П. Токмаковой).
                   3319: http://flibusta.is/b/129573/read
                   3320:    2. https://books.google.ru/books?id=NsF6AgAAQBAJ&pg=PA9#v=onepage&q&f=false
                   3321: 
                   3322: Автор:
                   3323: Сергей Дубелевич ("Middle")
                   3324: 
                   3325: Вопрос 9:
                   3326: Норвежская тминная водка обрела популярность в середине XIX века, после
                   3327: того как оказалась никому не нужной в Австралии. В более известной
                   3328: аналогичной истории фигурирует остров. Какой?
                   3329: 
                   3330: Ответ:
                   3331: Мадейра.
                   3332: 
                   3333: Зачет:
                   3334: Мадера.
                   3335: 
                   3336: Комментарий:
                   3337: Согласно легенде, мадера получила свой вкус благодаря тому, что партия
                   3338: вина пережила длительное путешествие из Португалии в Индию и обратно. На
                   3339: норвежской тминной водке благотворно сказалось длительное путешествие с
                   3340: двумя пересечениями экватора.
                   3341: 
                   3342: Источник:
                   3343: Н.В. Будур. Эти странные норвежцы. http://flibusta.is/b/162378/read
                   3344: 
                   3345: Автор:
                   3346: Сергей Дубелевич ("Middle")
                   3347: 
                   3348: Вопрос 10:
                   3349: Персонажей сериала "Американская история ужасов" преследует множество
                   3350: призраков. Например, в доме то и дело появляются две медсестры, а в
                   3351: двери ломятся крестьяне, вооруженные сельскохозяйственным инструментом.
                   3352: Какой культовый фильм часто "цитируется" в сериале?
                   3353: 
                   3354: Ответ:
                   3355: "Сияние".
                   3356: 
                   3357: Комментарий:
                   3358: Две одинаково одетые медсестры и крестьянин, проламывающий двери
                   3359: топором, воспроизводят известные сцены из фильма Кубрика.
                   3360: 
                   3361: Источник:
                   3362: Телесериал "Американская история ужасов: Роанок".
                   3363: 
                   3364: Автор:
                   3365: Артем Пьянков ("Легионеры Боливарии")
                   3366: 
                   3367: Вопрос 11:
                   3368: В вопросе есть замена.
                   3369:    Калмыцкая загадка об ИКСЕ звучит так: "В чашке - пестрое мясо".
                   3370: Разновидности какого блюда дал название ИКС?
                   3371: 
                   3372: Ответ:
                   3373: Яичница.
                   3374: 
                   3375: Комментарий:
                   3376: Глазунья.
                   3377: 
                   3378: Источник:
                   3379:    1. https://ru.wikiquote.org/wiki/Калмыцкие_загадки
                   3380:    2. https://ru.wikipedia.org/wiki/Яичница
                   3381: 
                   3382: Автор:
                   3383: Виталий Захарик ("Middle")
                   3384: 
                   3385: Вопрос 12:
                   3386: В фантастическом романе Артура Кларка строительство ЕГО потребовало от
                   3387: человечества основательной чистки накопившегося мусора. Примером ЕГО в
                   3388: мифологии можно считать Иггдрасиль, который прорастает через шесть
                   3389: миров. Назовите ЕГО двумя словами, которые начинаются на соседние буквы
                   3390: алфавита.
                   3391: 
                   3392: Ответ:
                   3393: Космический лифт.
                   3394: 
                   3395: Комментарий:
                   3396: Строительство космического лифта требовало очистки орбиты от
                   3397: космического мусора.
                   3398: 
                   3399: Источник:
                   3400:    1. http://www.mirf.ru/science/kosmicheskiy-musor
                   3401:    2. https://ru.wikipedia.org/wiki/Космический_лифт
                   3402: 
                   3403: Автор:
                   3404: Василий Бобков ("Корпрусариум")
                   3405: 
                   3406: Вопрос 13:
                   3407: В вопросе есть замена.
                   3408:    Гровер Кливленд в свою бытность шерифом однажды выполнял функции
                   3409: ИКСА. В чешском костеле Святой Варвары ИКСУ предназначалось отдельное и
                   3410: самое удаленное место от всех прихожан. Назовите ИКСА одним словом.
                   3411: 
                   3412: Ответ:
                   3413: Палач.
                   3414: 
                   3415: Комментарий:
                   3416: До того как стать президентом США, Гровер Кливленд работал шерифом
                   3417: города Эри, штат Нью-Йорк. Однажды он должен был либо сам исполнить
                   3418: смертный приговор, либо за 10 долларов нанять палача. Кливленд предпочел
                   3419: казнить заключенного сам. А в средние века в костелах палачу нельзя было
                   3420: молиться вместе с прихожанами.
                   3421: 
                   3422: Источник:
                   3423:    1. http://usa-info.com.ua/interesnye-fakty-o-prezidentakh-ssha/presidents/facts-presidents
                   3424:    2. http://possward.blogspot.com.by/2014/12/what-where-when-2014-winter-game-4.html
                   3425: 
                   3426: Автор:
                   3427: Команда "Хунвейбины"
                   3428: 
                   3429: Вопрос 14:
                   3430: Герой одной из новелл "Декамерона", переодевшись священником, исповедует
                   3431: свою жену. Чтобы она не узнала своего мужа, тот, в числе прочего,
                   3432: предварительно СДЕЛАЛ ЭТО. Назовите того, кто ДЕЛАЛ ЭТО в IV веке до
                   3433: нашей эры.
                   3434: 
                   3435: Ответ:
                   3436: Демосфен.
                   3437: 
                   3438: Комментарий:
                   3439: Чтобы героиня не узнала героя по голосу, он набирает в рот камешков. То
                   3440: же самое делал и Демосфен, совершенствуя свои ораторские способности.
                   3441: 
                   3442: Источник:
                   3443:    1. Дж. Боккаччо. Декамерон. http://flibusta.is/b/342759/read
                   3444:    2. https://ru.wikipedia.org/wiki/Демосфен
                   3445: 
                   3446: Автор:
                   3447: Станислав Адаскевич (Вильнюс)
                   3448: 
                   3449: Вопрос 15:
                   3450: Пришедший на съемку Владимир Познер был удивлен, когда ведущий не
                   3451: подошел к нему на рукопожатие, а публика не встретила аплодисментами. В
                   3452: этот момент на съемочной площадке можно было увидеть, например,
                   3453: медитирующего человека, а также ЕГО вместо оператора. Идея фильма под
                   3454: названием "ОН" возникла у режиссера после прогулки по Пятой авеню,
                   3455: известной своими бутиками. Назовите ЕГО одним словом.
                   3456: 
                   3457: Ответ:
                   3458: Манекен.
                   3459: 
                   3460: Комментарий:
                   3461: На Пятой авеню города Нью-Йорка, как известно, расположено множество
                   3462: разнообразных бутиков. Режиссеру показалось, что один из манекенов,
                   3463: стоящих на витрине, ожил - так и родилась идея фильма. В первом случае
                   3464: речь идет о вирусном флешмобе "Mannequin Challenge" [мАнекен чЕллендж],
                   3465: суть которого заключается в подражании манекенам некоторой группы людей,
                   3466: пока их снимают. В данном случае для придания комичности во время съемки
                   3467: "Челленджа" возле одной из камер стоял манекен.
                   3468: 
                   3469: Источник:
                   3470:    1. https://www.youtube.com/watch?v=ElQ1SVKQiiE&t=9m05s
                   3471:    2. https://ru.wikipedia.org/wiki/Манекен_(фильм)
                   3472: 
                   3473: Автор:
                   3474: Иван Сергиевич ("Корпрусариум")
                   3475: 
                   3476: Вопрос 16:
                   3477:    <раздатка>
                   3478:    Suji wa dokushin ni kagiru
                   3479:    </раздатка>
                   3480:    Перед вам первоначальное название ЭТОГО. Прообраз современного ЭТОГО
                   3481: под названием "Числовая площадь" был впервые опубликован в "Dell
                   3482: Magazines" [делл мЭгэзинз]. Назовите ЭТО.
                   3483: 
                   3484: Ответ:
                   3485: Судоку.
                   3486: 
                   3487: Комментарий:
                   3488: Первоначально игра называлась так, как приведено на раздатке. Позже
                   3489: название сократили до "su doku". В журналах и газетах часто публикуют
                   3490: кроссворды и судоку.
                   3491: 
                   3492: Источник:
                   3493: https://books.google.ru/books?id=bmiYDQAAQBAJ&pg=RA2-PT48#v=onepage&q&f=false
                   3494: 
                   3495: Автор:
                   3496: Василий Бобков ("Корпрусариум")
                   3497: 
                   3498: Вопрос 17:
                   3499: Макиавелли пишет, что юного государя важно обучить тому, что врага можно
                   3500: победить как при помощи умных законов, так и грубой силой, поэтому
                   3501: античные герои пользовались ЕГО услугами. Назовите ЕГО имя.
                   3502: 
                   3503: Ответ:
                   3504: Хирон.
                   3505: 
                   3506: Комментарий:
                   3507: В государе должно содержаться как человеческое, так и животное начало.
                   3508: Кентавр Хирон был учителем Ясона, Ахилла и многих других героев.
                   3509: Макиавелли также известен другой своей животной аналогией - о том, что
                   3510: государь должен содержать в себе черты льва и лисы.
                   3511: 
                   3512: Источник:
                   3513: Н. Макиавелли. Государь. http://flibusta.is/b/506547/read
                   3514: 
                   3515: Автор:
                   3516: Дмитрий Дорожко (Рига)
                   3517: 
                   3518: Вопрос 18:
                   3519: Рассказывая о бедности кубинцев, Борислав Козловский отмечает, что
                   3520: местные уличные торговцы, известные как "фосфореро", не только продают
                   3521: спички, но и ДЕЛАЮТ ЭТО. Какие два слова, начинающиеся на одни и те же
                   3522: две буквы, мы заменили словами "ДЕЛАЮТ ЭТО"?
                   3523: 
                   3524: Ответ:
                   3525: Заправляют зажигалки.
                   3526: 
                   3527: Комментарий:
                   3528: А в остальном мире пластмассовые зажигалки считаются одноразовыми.
                   3529: 
                   3530: Источник:
                   3531: Журнал "GEO", 2014, N 7.
                   3532: 
                   3533: Автор:
                   3534: Сергей Дубелевич ("Middle")
                   3535: 
                   3536: Вопрос 19:
                   3537: Дуплет.
                   3538:    1. Немецкий писатель Зигмунд Графф писал: "АЛЬФА - это политика,
                   3539: которую уже нельзя исправить". Назовите АЛЬФУ.
                   3540:    2. Немецкий писатель Зигмунд Графф писал: "БЕТА - это история,
                   3541: которую еще можно исправить". Назовите БЕТУ.
                   3542: 
                   3543: Ответ:
                   3544:    1. История.
                   3545:    2. Политика.
                   3546: 
                   3547: Источник:
                   3548: https://dic.academic.ru/dic.nsf/aphorism/1637/
                   3549: 
                   3550: Автор:
                   3551: Виталий Захарик ("Middle")
                   3552: 
                   3553: Вопрос 20:
                   3554: У главного героя одного фантастического романа нет ничего лишнего: шесть
                   3555: рук, две ноги, три глаза, хвост, крылья. Он способен на большой скорости
                   3556: рассекать по воздуху и по суше. Неспроста настоящая фамилия этого героя
                   3557: - АЛЬФА. Назовите АЛЬФУ.
                   3558: 
                   3559: Ответ:
                   3560: Бритва.
                   3561: 
                   3562: Комментарий:
                   3563: Олег Бритва - главный герой фантастических произведений Александра
                   3564: Рудазова о существе Яцхен. Фраза "нет ничего лишнего" - небольшой намек
                   3565: на принцип "бритвы Оккама".
                   3566: 
                   3567: Источник:
                   3568:    1. http://ru.anime-characters-fight.wikia.com/wiki/Яцхен
                   3569:    2. А.В. Рудазов. Три глаза и шесть рук.
                   3570: http://flibusta.is/b/47135/read
                   3571:    3. http://fb.ru/article/66328/britva-okkama-otsekaya-lishnee
                   3572: 
                   3573: Автор:
                   3574: Команда "Хунвейбины"
                   3575: 
                   3576: Вопрос 21:
                   3577: (pic: 20161028.jpg)
                   3578:    Как называется заведение, где беседуют персонажи игры "Святые
                   3579: Картофелины, мы что, в космосе?!"?
                   3580: 
                   3581: Ответ:
                   3582: Spacebar.
                   3583: 
                   3584: Зачет:
                   3585: Пробел, space bar, space.
                   3586: 
                   3587: Комментарий:
                   3588: Вывеска заведения стилизована под клавиши command и пробел.
                   3589: 
                   3590: Источник:
                   3591: https://www.youtube.com/watch?v=spVnY3753Nw&t=1m26s
                   3592: 
                   3593: Автор:
                   3594: Артем Пьянков ("Легионеры Боливарии")
                   3595: 
                   3596: Вопрос 22:
                   3597: В одном из эпизодов сериала "Воздействие" герой по имени Чарльз проходит
                   3598: через череду необычных событий, считая, что это сон. Назовите двумя
                   3599: словами животное, которое фигурирует в названии этого эпизода.
                   3600: 
                   3601: Ответ:
                   3602: Белый кролик.
                   3603: 
                   3604: Комментарий:
                   3605: Героя эпизода "Дело о белом кролике" решили назвать так же, как звали
                   3606: Льюиса Кэрролла, - Чарльз Доджсон. Приключения Алисы в Стране чудес
                   3607: происходили в ее снах.
                   3608: 
                   3609: Источник:
                   3610:    1. http://leverage.wikia.com/wiki/The_White_Rabbit_Job
                   3611:    2. http://ru.wikipedia.org/wiki/Алиса_в_Стране_чудес
                   3612: 
                   3613: Автор:
                   3614: Василий Бобков ("Корпрусариум")
                   3615: 
                   3616: Вопрос 23:
                   3617: В начале своего стихотворения Дмитрий Авалиани называет себя "ящеркой
                   3618: ютящейся эпохи", а по ходу произведения неоднократно употребляет
                   3619: церковную лексику. Какое слово, завершающее произведение, рифмуется со
                   3620: словом "горЯ"?
                   3621: 
                   3622: Ответ:
                   3623: Алтаря.
                   3624: 
                   3625: Комментарий:
                   3626: Все слова в этом стихотворении расположены в обратном алфавитном
                   3627: порядке. Концовка выглядит так:
                   3628:    Земная
                   3629:    жизнь
                   3630:    еще
                   3631:    дарит,
                   3632:    горя,
                   3633:    высокое
                   3634:    блаженство
                   3635:    алтаря.
                   3636: 
                   3637: Источник:
                   3638: https://45parallel.net/dmitriy_avaliani/ya_yascherka.html
                   3639: 
                   3640: Автор:
                   3641: Сергей Дубелевич ("Middle")
                   3642: 
                   3643: Вопрос 24:
                   3644: (pic: 20161029.jpg)
                   3645:    Прозвище тройки Тоффоли - Пирсон - Картер совпадает с названием
                   3646: фильма, в котором освещаются война во Вьетнаме, Уотергейтский скандал,
                   3647: сексуальная революция. Напишите это название из одного слова.
                   3648: 
                   3649: Ответ:
                   3650: "Семидесятые".
                   3651: 
                   3652: Комментарий:
                   3653: В 1970-е произошли все указанные события, а хоккеисты играют под
                   3654: номерами 70 - 73 - 77.
                   3655: 
                   3656: Источник:
                   3657:    1. https://www.sports.ru/tribuna/blogs/hokkeinosti/1086710.html
                   3658:    2. https://www.kinopoisk.ru/film/94193/
                   3659: 
                   3660: Автор:
                   3661: Вадим Германенко (Рига)
                   3662: 
                   3663: Вопрос 25:
                   3664: Бутан никогда официально не был ничьей колонией, поэтому в этом
                   3665: государстве нет ИКСА. Один из слоганов "ИКСА" - "Не стройте планы на
                   3666: август". Назовите ИКС двумя словами.
                   3667: 
                   3668: Ответ:
                   3669: День независимости.
                   3670: 
                   3671: Комментарий:
                   3672: Действие фильма "День независимости" происходит в канун дня
                   3673: независимости США, т.е. в начале июля. В Бутане, в отличие от многих
                   3674: государств, нет праздника "День независимости".
                   3675: 
                   3676: Источник:
                   3677:    1. https://en.wikipedia.org/wiki/Independence_of_Bhutan
                   3678:    2. https://ru.wikipedia.org/wiki/День_независимости_(фильм,_1996)
                   3679: 
                   3680: Автор:
                   3681: Сергей Дубелевич ("Middle")
                   3682: 
                   3683: Вопрос 26:
                   3684: Автогонщик из повести Андрея Жвалевского и Евгении Пастернак объясняет
                   3685: спутнику-подростку схему автомобильного трюка, находясь за столиком в
                   3686: кафе. После объяснения он, усмехнувшись, вспоминает об ЭТОМ ЧЕЛОВЕКЕ, но
                   3687: тинейджер его не понимает. Назовите ЭТОГО ЧЕЛОВЕКА.
                   3688: 
                   3689: Ответ:
                   3690: Чапаев.
                   3691: 
                   3692: Комментарий:
                   3693: Для иллюстрации схемы гонщик расставляет предметы на столе, а потом
                   3694: говорит, что поступает, как Чапаев в фильме. Спутница автогонщика, не
                   3695: знакомая с советской культурой, не знает, кто это.
                   3696: 
                   3697: Источник:
                   3698: А.В. Жвалевский, Е.Б. Пастернак. Пока я на краю.
                   3699: http://flibusta.is/b/469108/read
                   3700: 
                   3701: Автор:
                   3702: Сергей Дубелевич ("Middle")
                   3703: 
                   3704: Вопрос 27:
                   3705: В сериале "Конь БоДжек" антропоморфные животные живут и работают наравне
                   3706: с людьми. В одной из серий БоДжек Хорсмэн приходит на съемки телешоу, а
                   3707: камера, следуя за героем, постепенно знакомит зрителя со съемочной
                   3708: группой, плавно переходя от одного персонажа к другому. Ответьте словом
                   3709: с двумя английскими корнями: кем является невезучий помощник режиссера?
                   3710: 
                   3711: Ответ:
                   3712: Бёрдмэн.
                   3713: 
                   3714: Комментарий:
                   3715: Серия копирует манеру съемки "одним дублем", известную по фильмам
                   3716: "Бёрдмэн" и "Выживший".
                   3717: 
                   3718: Источник:
                   3719: Мультсериал "Конь БоДжек", s02e08.
                   3720: 
                   3721: Автор:
                   3722: Артем Пьянков ("Легионеры Боливарии")
                   3723: 
                   3724: Вопрос 28:
                   3725: В конце XIX века дирижер Уолтер Дамрош сумел завести нового друга -
                   3726: крупного американского предпринимателя. Дамрош оказал на него такое
                   3727: сильное влияние, что бизнесмен стал жертвовать крупные суммы на развитие
                   3728: искусства. Напишите фамилию этого предпринимателя.
                   3729: 
                   3730: Ответ:
                   3731: Карнеги.
                   3732: 
                   3733: Комментарий:
                   3734: По инициативе Эндрю Карнеги в Нью-Йорке был построен крупный концертный
                   3735: зал, известный как Карнеги-холл. В отличие от другого Карнеги - Дейла,
                   3736: автора книги "Как завоевывать друзей и оказывать влияние на людей",
                   3737: Эндрю Карнеги сам подвергся чужому влиянию.
                   3738: 
                   3739: Источник:
                   3740:    1. http://www.states-of-america.ru/new-york-city/dostoprimechatelnosti/karnegi-holl/
                   3741:    2. https://ru.wikipedia.org/wiki/Карнеги,_Дейл
                   3742: 
                   3743: Автор:
                   3744: Виталий Захарик ("Middle")
                   3745: 
                   3746: Вопрос 29:
                   3747: Внимание, в вопросе есть замены.
                   3748:    Комик Александр Бёресс в своих интервью говорил, что не любит свое
                   3749: довольно редкое имя еще со школы, потому что его ровесники ничего не
                   3750: слышали о полководце, зато знали о Доценте. Какие два имени собственных
                   3751: мы заменили в этом вопросе?
                   3752: 
                   3753: Ответ:
                   3754: Ганнибал, Лектер.
                   3755: 
                   3756: Комментарий:
                   3757: Комик Ганнибал Бёресс жалуется, что в школе сверстники не слышали о
                   3758: карфагенском Ганнибале, в честь которого он был назван, но знали
                   3759: Ганнибала Лектера, из-за чего и посмеивались над ним.
                   3760: 
                   3761: Источник:
                   3762: https://www.youtube.com/watch?v=W_4dMWkg0rIe&t=4m22s
                   3763: 
                   3764: Автор:
                   3765: Артем Пьянков ("Легионеры Боливарии")
                   3766: 
                   3767: Вопрос 30:
                   3768: Персонаж современной повести Катажины Грохоли, увидев возобновление
                   3769: работ на замороженной стройке, удивляется: неужели снова ОНО? Назовите
                   3770: ЕГО несклоняемым словом.
                   3771: 
                   3772: Ответ:
                   3773: Евро.
                   3774: 
                   3775: Комментарий:
                   3776: Катажина Грохоля - польская писательница. Перед чемпионатом Европы по
1.4     ! rubashki 3777: футболу 2012 года в Польше было построено много новых объектов.
1.1       rubashki 3778: 
                   3779: Источник:
                   3780: К. Грохоля. Хьюстон, у нас проблема. http://flibusta.is/b/412937/read
                   3781: 
                   3782: Автор:
                   3783: Сергей Дубелевич ("Middle")
                   3784: 
                   3785: Вопрос 31:
                   3786: В Вышнем Волочке есть каналы и дамбы. Неудивительно, что на
                   3787: привокзальной площади этого города находится ОН. Назовите ЕГО тремя
                   3788: словами, начинающимися на одну и ту же букву.
                   3789: 
                   3790: Ответ:
                   3791: Памятник Петру Первому.
                   3792: 
                   3793: Комментарий:
                   3794: После пребывания в Голландии Петр I решил построить каналы в России. На
                   3795: памятнике он одобряет схему проекта водной системы, подготовленную
                   3796: Михаилом Сердюковым.
                   3797: 
                   3798: Источник:
                   3799:    1. https://ru.wikipedia.org/wiki/Вышний_Волочёк
                   3800:    2. https://www.votpusk.ru/country/dostoprim_info.asp?ID=15621
                   3801: 
                   3802: Автор:
                   3803: Иван Зайков ("Легионеры Боливарии")
                   3804: 
                   3805: Вопрос 32:
                   3806: Говоря о засекреченности нахождения в СССР Бруно Понтекорво, сайт
                   3807: colta.ru [кОлта ру] пишет, что Понтекорво стал "ИКСОМ". Можно сказать,
                   3808: что один сирота стал "ИКСОМ" из-за страха окружающих. Назовите имя,
                   3809: данное этому сироте при рождении.
                   3810: 
                   3811: Ответ:
                   3812: Том.
                   3813: 
                   3814: Зачет:
                   3815: Томас Марволо Реддл.
                   3816: 
                   3817: Комментарий:
                   3818: ИКС - "Тот-Кого-Нельзя-Называть". Выросшего в приюте Тома Реддла,
                   3819: известного также как Волан-де-Морт, в мире волшебников боялись до такой
                   3820: степени, что даже его имя, как правило, не произносили. Целых пять лет
                   3821: нахождение Понтекорво было под секретом.
                   3822: 
                   3823: Источник:
                   3824:    1. http://www.colta.ru/articles/science/6410
                   3825:    2. https://ru.wikipedia.org/wiki/Волан-де-Морт
                   3826: 
                   3827: Автор:
                   3828: Иван Сергиевич ("Корпрусариум")
                   3829: 
                   3830: Вопрос 33:
                   3831: Согласно исследованиям астрофизиков, в верхних слоях атмосферы Сатурна
                   3832: разряды молнии периодически затрагивают молекулы метана, при этом
                   3833: высвобождаются атомы углерода. Эти атомы соединяются друг с другом и,
                   3834: продолжая сжиматься, начинают движение к каменному ядру планеты.
                   3835: Поэтому, по мнению журналиста Аси Гориной, на Сатурне можно увидеть ЕГО.
                   3836: Ивану обещали, что он тоже увидит ЕГО. Назовите ЕГО тремя словами.
                   3837: 
                   3838: Ответ:
                   3839: Небо в алмазах.
                   3840: 
                   3841: Комментарий:
                   3842: Углерод, продолжая сжиматься, превращается в алмазы. Героиня пьесы
                   3843: Чехова "Дядя Ваня" обещает заглавному персонажу, что они тоже увидят
                   3844: небо в алмазах.
                   3845: 
                   3846: Источник:
                   3847:    1. http://www.vesti.ru/doc.html?id=1140230
                   3848:    2. http://www.bibliotekar.ru/encSlov/13/167.htm
                   3849: 
                   3850: Автор:
                   3851: Виталий Захарик ("Middle")
                   3852: 
                   3853: Вопрос 34:
                   3854: Внимание, в вопросе есть замены.
                   3855:    Нарушая правила, нетрезвый герой современной песни утверждает, что
                   3856: правая НОГА интереснее левой. Какое слово мы заменили словом "НОГА"?
                   3857: 
                   3858: Ответ:
                   3859: Педаль.
                   3860: 
                   3861: Комментарий:
                   3862: Главный герой, злоупотребив алкоголем, садится за руль и, разумеется,
                   3863: предпочитает жать педаль газа, которая в автомобиле обычно расположена
                   3864: справа. Слово "педаль" происходит от латинского "pedis" [пЕдис] -
                   3865: "нога".
                   3866: 
                   3867: Источник:
                   3868:    1. http://www.gl5.ru/noize_mc_jameson.html
                   3869:    2. https://ru.wiktionary.org/wiki/педаль
                   3870: 
                   3871: Автор:
                   3872: Иван Сергиевич ("Корпрусариум")
                   3873: 
                   3874: Вопрос 35:
                   3875: Персонаж повести Бориса Акунина, действие которой происходит в XIX веке,
                   3876: скептически отзывается о своих перспективах: "Так и пробегаешь всю
                   3877: жизнь, не заслужив хорошего чина". Какое прилагательное в его цитате мы
                   3878: заменили?
                   3879: 
                   3880: Ответ:
                   3881: Классного.
                   3882: 
                   3883: Комментарий:
                   3884: В данном случае это слово обозначает "относящегося к одному из классов
                   3885: табели о рангах".
                   3886: 
                   3887: Источник:
                   3888: Б. Акунин. Особые поручения: Пиковый валет.
                   3889: http://flibusta.is/b/187523/read
                   3890: 
                   3891: Автор:
                   3892: Сергей Дубелевич ("Middle")
                   3893: 
                   3894: Вопрос 36:
                   3895: Внимание, в вопросе есть замена.
                   3896:    Говоря о вероятном конфликте между Британской и Японской империями,
                   3897: Ниал Фергюсон сравнивает Британию с ТАКИМ ИМ. Какие два слова,
                   3898: начинающиеся на парные согласные, мы заменили словами "ТАКОЕ ОНО"?
                   3899: 
                   3900: Ответ:
                   3901: Заходящее солнце.
                   3902: 
                   3903: Комментарий:
                   3904: Фергюсон сравнивает возможный конфликт старой Британской и новой
                   3905: Японской империй со столкновением заходящего и восходящего солнца.
                   3906: 
                   3907: Источник:
                   3908: Н. Фергюсон. Империя: чем современный мир обязан Британии.
                   3909: http://flibusta.is/b/378579/read
                   3910: 
                   3911: Автор:
                   3912: Елена Гордынец ("Легионеры Боливарии")
                   3913: 
                   3914: Тур:
                   3915: 6 этап
                   3916: 
                   3917: Дата:
                   3918: 15-Apr-2017
                   3919: 
                   3920: Редактор:
                   3921: 1-18 - Вера Рабкина (Минск); 19-36 - Анастасия Балмакова (Минск)
                   3922: 
                   3923: Инфо:
                   3924: Вера Рабкина выражает благодарность Михаилу Карпуку, Александру
                   3925: Марцинкевичу, Анастасии Балмаковой, Павлу Чернявскому, Виктории
                   3926: Чернявской, Павлу Полярушу, Евгению Миротину, а также командам
                   3927: "Енотики-7" и "Эталон этанола" (все - Минск) за ценные замечания и
                   3928: помощь в подготовке пакета. Анастасия Балмакова выражает благодарность
                   3929: Михаилу Карпуку, Дарье Соловей, Александру Марцинкевичу, Александре
                   3930: Ермалович, Елене Ваксман-Атроховой, а также командам "Енотики-7" и
                   3931: "Эталон этанола" (все - Минск) за ценные замечания и помощь в подготовке
                   3932: пакета.
                   3933: 
                   3934: Вопрос 1:
                   3935: Особенность ИХ образа жизни связана со сложным способом пищеварения,
                   3936: отнимающим много энергии. Сотрудники заповедника в Коста-Рике
                   3937: рассказывают, как после каждого мытья вешают ИХ на просушку. Назовите
                   3938: ИХ.
                   3939: 
                   3940: Ответ:
                   3941: Ленивцы.
                   3942: 
                   3943: Комментарий:
                   3944: Питание ленивца и является основной причиной его "ленивости" - животное
                   3945: старается тратить поменьше энергии, так как почти вся она нужна ему для
1.3       rubashki 3946: переваривания. При этом листья являются очень низкокалорийной пищей и
1.1       rubashki 3947: дают крайне мало энергии.
                   3948: 
                   3949: Источник:
                   3950:    1. https://www.factroom.ru/facts/12064
                   3951:    2. https://www.youtube.com/watch?v=q1mAGQAw3Oc
                   3952: 
                   3953: Автор:
                   3954: Вера Рабкина ("Енотики-7")
                   3955: 
                   3956: Вопрос 2:
                   3957: В романе Дэна Симмонса рассказывается о том, как Уилки Коллинз создавал
                   3958: образ одного из первых английских детективов - сержанта Каффа. Сначала
                   3959: Коллинз предположил, что хобби Каффа будет ОНО, но потом передумал в
                   3960: пользу выращивания роз. Назовите ЕГО одним словом.
                   3961: 
                   3962: Ответ:
                   3963: Пчеловодство.
                   3964: 
                   3965: Комментарий:
                   3966: Все английские детективы немного похожи.
                   3967: 
                   3968: Источник:
                   3969: Д. Симмонс. Друд, или Человек в черном. http://flibusta.is/b/229505/read
                   3970: 
                   3971: Автор:
                   3972: Никита Геер ("Енотики-7")
                   3973: 
                   3974: Вопрос 3:
                   3975: Здание Центрального ЗАГСа города Киева за необычную форму получило
                   3976: прозвище ИКС. Местные так и говорят об этом ЗАГСе: "Как туда пошел -
                   3977: пропал!". Назовите ИКС двумя словами.
                   3978: 
                   3979: Ответ:
                   3980: Бермудский треугольник.
                   3981: 
                   3982: Комментарий:
                   3983: Здание имеет форму треугольника, а о многих молодых мужчинах так
                   3984: говорят: "Женился - пропал!".
                   3985: 
                   3986: Источник:
                   3987:    1. http://wedding.ua/company/register/kiev/81/
                   3988:    2. https://www.interesniy.kiev.ua/zags-bermudskiy-treugolnik/
                   3989: 
                   3990: Автор:
                   3991: Евгений Зайцев ("Умник")
                   3992: 
                   3993: Вопрос 4:
                   3994: Один немецкий ресторан предлагает своим посетителям преодолеть особенно
                   3995: высокий бургер под названием АЛЬФА. На другой АЛЬФЕ нанесены примеры
                   3996: европейского стрит-арта. Назовите АЛЬФУ двумя словами.
                   3997: 
                   3998: Ответ:
                   3999: Берлинская стена.
                   4000: 
                   4001: Комментарий:
                   4002: Несмотря на то что бургер Берлинская Стена очень высокий, некоторым
                   4003: посетителям всё же удается его преодолеть, равно как и настоящую стену
                   4004: удалось преодолеть некоторым берлинцам.
                   4005: 
                   4006: Источник:
                   4007: https://www.facebook.com/hamburgburger/photos/a.688653644482783.1073741828.688640321150782/1200039526677523/?type=3&theater
                   4008: 
                   4009: Автор:
                   4010: Вера Рабкина ("Енотики-7")
                   4011: 
                   4012: Вопрос 5:
                   4013: (pic: 20161030.jpg)
                   4014:    Перед вами один из возможных вариантов ИКСА. Назовите ИКС двумя
                   4015: словами.
                   4016: 
                   4017: Ответ:
                   4018: Флаг Марса.
                   4019: 
                   4020: Комментарий:
                   4021: На розданном изображении мы видим, что Марс - четвертая планета от
                   4022: Солнца, которое изображено справа.
                   4023: 
                   4024: Источник:
                   4025: https://hi-news.ru/space/kakim-by-mog-byt-flag-marsa.html
                   4026: 
                   4027: Автор:
                   4028: Кирилл Севкович ("Эталон этанола")
                   4029: 
                   4030: Вопрос 6:
                   4031: Недавно компания, выпускающая АЛЬФУ, решила обновить набор из восьми
                   4032: предметов. Как ни удивительно, она отказалась от исключительного права
                   4033: единолично принимать решение и провела голосование среди фанатов. В
                   4034: результате АЛЬФА пополнится резиновой уточкой, тираннозавром и
                   4035: пингвином. Что мы заменили АЛЬФОЙ?
                   4036: 
                   4037: Ответ:
                   4038: Монополия.
                   4039: 
                   4040: Комментарий:
                   4041: Компания "Hasbro" отказалась от монополии на выбор новых фигурок и
                   4042: провела голосование.
                   4043: 
                   4044: Источник:
                   4045: https://joinfo.ua/sociaty/1200833_Tirannozavr-rezinovaya-utochka-pingvin-Monopoliya.html
                   4046: 
                   4047: Автор:
                   4048: Надежда Потрихалина ("Енотики-7")
                   4049: 
                   4050: Вопрос 7:
                   4051: В книге Джареда Даймонда описывается сцена пленения Атауальпы отрядом
                   4052: Франсиско Писарро. В ней упоминается имя Сантьяго. Автор вопроса
                   4053: предположил, что в версии, адаптированной для современного американского
                   4054: читателя, могло бы присутствовать другое имя. Какое?
                   4055: 
                   4056: Ответ:
                   4057: Джеронимо.
                   4058: 
                   4059: Комментарий:
                   4060: Отряд Писарро использовал испанский боевой клич "Сантьяго!". Интересно,
                   4061: что клич "Джеронимо!", который также является именем, используется в США
                   4062: десантниками-парашютистами во время прыжка из самолета.
                   4063: 
                   4064: Источник:
                   4065: Дж. Даймонд. Ружья, микробы и сталь. Судьбы человеческих обществ.
                   4066: http://flibusta.is/b/237834/read
                   4067: 
                   4068: Автор:
                   4069: Никита Геер ("Енотики-7")
                   4070: 
                   4071: Вопрос 8:
                   4072: Согласно одной из версий, современный вариант ЕЕ появился в 1904 году
                   4073: под названием "ОНА для холостяков" и предназначался для мужчин, которые
                   4074: не хотели пришивать пуговицы. В английском языке она получила название
                   4075: от АЛЬФЫ. Что мы заменили АЛЬФОЙ?
                   4076: 
                   4077: Ответ:
                   4078: [Буква] T.
                   4079: 
                   4080: Комментарий:
                   4081: Речь идет о майке с короткими рукавами, также известной как T-shirt
                   4082: [ти-шот]. Удивительно, но ее современный вариант появился только в 1904
                   4083: году под названием "майка для холостяков". Рекламный слоган гласил:
                   4084: "Никаких булавок, пуговиц, ниток и иголок", а главной целевой аудиторией
                   4085: были холостые мужчины, которым некому было пришить оторвавшиеся
                   4086: пуговицы.
                   4087: 
                   4088: Источник:
                   4089: https://www.nytimes.com/2013/09/22/magazine/who-made-that-t-shirt.html
                   4090: 
                   4091: Автор:
                   4092: Вера Рабкина ("Енотики-7")
                   4093: 
                   4094: Вопрос 9:
                   4095: В 1950-е годы в СССР остро стояла проблема давки на стадионах после
                   4096: окончания матча. Тогда организаторы пошли на хитрость, сделав ЕГО еще и
                   4097: ТАКИМ. Это в большой степени решило проблему, а также мотивировало
                   4098: зрителей оставаться до конца матча. Что мы заменили словом "ТАКИМ"?
                   4099: 
                   4100: Ответ:
                   4101: Лотерейным.
                   4102: 
                   4103: Комментарий:
                   4104: После матча по очереди для каждой трибуны разыгрывались призы. В
                   4105: качестве лотерейного выступал входной билет на матч.
                   4106: 
                   4107: Источник:
                   4108: Журнал "Большой", октябрь 2016 г.
                   4109: 
                   4110: Автор:
                   4111: Вера Рабкина ("Енотики-7")
                   4112: 
                   4113: Вопрос 10:
                   4114: В тексте сценария ЭТО было помечено так: "Крылова приветствует гостей".
                   4115: В окончательном варианте ЭТО длится 222 секунды. Сколько ЭТО длилось, по
                   4116: словам самой Крыловой?
                   4117: 
                   4118: Ответ:
                   4119: Пять минут.
                   4120: 
                   4121: Комментарий:
                   4122: Имеется в виду сценарий фильма Рязанова "Карнавальная ночь", согласно
                   4123: которому героиня Гурченко Леночка Крылова пела приветственную песню.
                   4124: Длительность оригинального трека этой песни составляет 3 минуты и 42
                   4125: секунды (или 222 секунды). Песня называется "Пять минут". По тексту
                   4126: песни "... но, пока я песню пела, пять минут уж пролетело...".
                   4127: 
                   4128: Источник:
                   4129:    1. http://www.radioshanson.fm/news/istoriya_pesni__pyat__minut
                   4130:    2. https://zf.fm/song/2243
                   4131: 
                   4132: Автор:
                   4133: Евгений Зайцев ("Умник")
                   4134: 
                   4135: Вопрос 11:
                   4136: ТАКОЙ ОН в произведении 1925 года является символом надежды,
                   4137: недосягаемым ориентиром для главного героя. Клятва ТАКОГО ЕГО содержала
                   4138: следующие слова: "И должен я пролить мой свет на темное зло, ибо тьма не
                   4139: устоит перед светом...". Что мы заменили словами "ТАКОЙ ОН"?
                   4140: 
                   4141: Ответ:
                   4142: Зеленый фонарь.
                   4143: 
                   4144: Комментарий:
                   4145: Зеленый цвет в литературе традиционно считается символом надежды. В
                   4146: романе "Великий Гэтсби" зеленый фонарь маяка имеет символический
                   4147: подтекст. Зеленые Фонари из одноименного комикса приносили клятву
                   4148: бороться со злом с помощью Зеленого фонаря.
                   4149: 
                   4150: Источник:
                   4151:    1. http://fitzgerald.narod.ru/critics-rus/itkina-gatsby.html
                   4152:    2. https://ru.wikipedia.org/wiki/Зелёный_Фонарь
                   4153: 
                   4154: Автор:
                   4155: Надежда Потрихалина ("Енотики-7")
                   4156: 
                   4157: Вопрос 12:
                   4158: Английская инструкция 1672 года упоминает кварту воды, а также
                   4159: рекомендует подержать АЛЬФУ над паром. АЛЬФУ в классическом мультфильме
                   4160: озвучивал девятилетний Брэдли Пирс. Назовите АЛЬФУ двумя словами,
                   4161: начинающимися на одну и ту же букву.
                   4162: 
                   4163: Ответ:
                   4164: Чайная чашка.
                   4165: 
                   4166: Зачет:
                   4167: Чашка чая.
                   4168: 
                   4169: Комментарий:
                   4170: Это инструкция по завариванию чая. Британцы еще в XVII веке нашли способ
                   4171: сберечь особенно ценный в ту эпоху фарфор. Брэдли Пирс озвучил Чипа -
                   4172: превращенного в чайную чашку мальчика из мультфильма "Красавица и
                   4173: Чудовище".
                   4174: 
                   4175: Источник:
                   4176:    1. https://www.thevintagenews.com/2016/09/01/priority-act-drinking-tea-uk-introduced-1662-portuguese-queen/
                   4177:    2. https://www.imdb.com/name/nm0682300/
                   4178: 
                   4179: Автор:
                   4180: Надежда Потрихалина ("Енотики-7")
                   4181: 
                   4182: Вопрос 13:
                   4183: Прослушайте цитату: "Недавно читал книгу про ПРОПУСК. Жуткое,
                   4184: отвратительное чтиво. Но под конец мне даже начало нравиться". Заполните
                   4185: пропуск двумя словами.
                   4186: 
                   4187: Ответ:
                   4188: Стокгольмский синдром.
                   4189: 
                   4190: Источник:
                   4191: https://www.reddit.com/r/Jokes/comments/2d4v7o/i_just_read_a_book_about_stockholm_syndrome/
                   4192: 
                   4193: Автор:
                   4194: Вера Забавская
                   4195: 
                   4196: Вопрос 14:
                   4197:    <раздатка>
                   4198:    Авиакомпания "Etihad Airways" запустила новый сервис "Etihad Wi-Fi" -
                   4199: широкополосный доступ в Интернет и мобильную связь на борту самолетов.
                   4200: Система разработана компанией "Panasonic" - одним из мировых лидеров
                   4201: среди поставщиков систем развлечения и связи в полете.
                   4202:    </раздатка>
                   4203:    В розданном отрывке мы слегка видоизменили одно слово. Напишите его в
                   4204: исходном варианте.
                   4205: 
                   4206: Ответ:
                   4207: Wi-Fly.
                   4208: 
                   4209: Зачет:
                   4210: Fly.
                   4211: 
                   4212: Комментарий:
                   4213: Компания "Etihad" [этихАд] гордится тем, что предоставляемый вайфай
                   4214: доступен во время полета, что и подчеркнула в названии.
                   4215: 
                   4216: Источник:
                   4217: https://www.airlines-inform.ru/news/etihad-wi-fly.html
                   4218: 
                   4219: Автор:
                   4220: Вера Рабкина ("Енотики-7")
                   4221: 
                   4222: Вопрос 15:
                   4223: [Ведущему: отточия в вопросе голосом никак не выделять.]
                   4224:    Прослушайте отрывок из книги Наринэ Абгарян: "Она спрятала портрет за
                   4225: большим деревянным ларем <...>, где он с тех пор лежал <...>, безнадежно
                   4226: отсыревший и выцветший за ПРОПУСК, на которые его обрекли равнодушные
                   4227: дальние потомки". Заполните пропуск названием произведения 1967 года.
                   4228: 
                   4229: Ответ:
                   4230: Сто лет одиночества.
                   4231: 
                   4232: Источник:
                   4233:    1. Н.Ю. Абгарян. С неба упали три яблока.
                   4234: http://flibusta.is/b/498016/read
                   4235:    2. https://ru.wikipedia.org/wiki/Сто_лет_одиночества
                   4236: 
                   4237: Автор:
                   4238: Вера Рабкина ("Енотики-7")
                   4239: 
                   4240: Вопрос 16:
                   4241: Современная художница Мун Рибас в качестве арт-проекта вживила в свои
                   4242: ступни специальные микрочипы. Она отмечает, что некоторые ее знакомые
                   4243: были буквально потрясены ее решением. Когда эти чипы вибрируют?
                   4244: 
                   4245: Ответ:
                   4246: Когда где-то в мире происходит землетрясение.
                   4247: 
                   4248: Зачет:
                   4249: Когда приближается землетрясение и другие близкие по смыслу ответы,
                   4250: содержащие слово "землетрясение" или "сейсмическая активность".
                   4251: 
                   4252: Комментарий:
                   4253: С помощью этих микрочипов Мун Рибас ощущает все землетрясения мира, и ее
                   4254: тело трясется вместе с другими жителями Земли, находящимися в эпицентре
                   4255: катастрофы.
                   4256: 
                   4257: Источник:
                   4258: https://realnoevremya.ru/articles/43899
                   4259: 
                   4260: Автор:
                   4261: Вера Рабкина ("Енотики-7")
                   4262: 
                   4263: Вопрос 17:
                   4264: Александр Мещеряков отмечает, что ОНО не устоялось в японской культуре,
                   4265: так как с его помощью нельзя выразить разную степень уважения. Назовите
                   4266: ЕГО одним словом.
                   4267: 
                   4268: Ответ:
                   4269: Рукопожатие.
                   4270: 
                   4271: Комментарий:
                   4272: Рукопожатие подразумевает, что все равны, в то время как поклон,
                   4273: принятый в Японии, может быть разной степени глубины.
                   4274: 
                   4275: Источник:
                   4276: https://arzamas.academy/courses/21/4
                   4277: 
                   4278: Автор:
                   4279: Вера Рабкина ("Енотики-7")
                   4280: 
                   4281: Вопрос 18:
                   4282:    <раздатка>
                   4283:    "История Древнего Рима за 20 минут"
                   4284:    </раздатка>
                   4285:    Перед вами название видеоролика, посвященного истории Древнего Рима.
                   4286: Что мы заменили в этом названии?
                   4287: 
                   4288: Ответ:
                   4289: XX.
                   4290: 
                   4291: Комментарий:
                   4292: Чтобы подчеркнуть тематику, авторы ролика заменили арабские цифры
                   4293: римскими.
                   4294: 
                   4295: Источник:
                   4296: http://arzamas.academy/likbez/antiquity
                   4297: 
                   4298: Автор:
                   4299: Дмитрий Медведев ("Енотики-7")
                   4300: 
                   4301: Вопрос 19:
                   4302: БАджо - это народ в Юго-Восточной Азии. Многим его представителям еще в
                   4303: детстве прокалывают барабанные перепонки. Те, кто подвергся этой
                   4304: процедуре, впоследствии зарабатывают больше, чем их соплеменники.
                   4305: Назовите основной промысел бАджо.
                   4306: 
                   4307: Ответ:
                   4308: Ловля жемчуга.
                   4309: 
                   4310: Зачет:
                   4311: Добыча жемчуга; ловля/добыча морских огурцов; по смыслу.
                   4312: 
                   4313: Комментарий:
                   4314: Баджо живут в домах на сваях и зарабатывают добычей жемчуга. Человек,
                   4315: которому с детства повредили барабанные перепонки, может без боли нырять
                   4316: на глубину до тридцати метров и добывать гораздо больше жемчуга.
                   4317: 
                   4318: Источник:
                   4319: http://www.factroom.ru/facts/9890
                   4320: 
                   4321: Автор:
                   4322: Дарья Соловей ("Одушевленные аэросани")
                   4323: 
                   4324: Вопрос 20:
                   4325: В романе, действие которого происходит в начале XX века, надзиратели
                   4326: разработали новый вид кандалов. Кого они пригласили для испытания своего
                   4327: изобретения?
                   4328: 
                   4329: Ответ:
                   4330: [Гарри] Гудини.
                   4331: 
                   4332: Комментарий:
                   4333: Гудини прославился сложными трюками с побегами и освобождениями.
                   4334: 
                   4335: Источник:
                   4336:    1. Э. Доктороу. Рэгтайм. http://flibusta.is/b/491173/read
                   4337:    2. https://ru.wikipedia.org/wiki/Гарри_Гудини
                   4338: 
                   4339: Автор:
                   4340: Александр Марцинкевич ("Одушевленные аэросани")
                   4341: 
                   4342: Вопрос 21:
                   4343: В 1994 году в одной стране проводился конкурс семейных автомобилей. По
                   4344: условиям конкурса каждый автомобиль должен был комплектоваться неким
                   4345: устройством, причем только одним. Мы не просим вас назвать это
                   4346: устройство. Ответьте, в какой стране проводился этот конкурс.
                   4347: 
                   4348: Ответ:
                   4349: Китай.
                   4350: 
                   4351: Зачет:
                   4352: КНР; Китайская Народная Республика.
                   4353: 
                   4354: Комментарий:
                   4355: Конкурс семейных автомобилей объявило китайское правительство в
                   4356: соответствии с китайской государственной программой "Одна семья - один
                   4357: ребенок", поэтому и детское автокресло должно было быть в автомобиле
                   4358: только одно.
                   4359: 
                   4360: Источник:
                   4361: https://www.drive2.ru/b/1911467/
                   4362: 
                   4363: Автор:
                   4364: Евгений Зайцев ("Умник")
                   4365: 
                   4366: Вопрос 22:
                   4367: С тех пор как в Китае стало модным пить дорогое вино, там можно купить
                   4368: бутылку настоящего "ЛафИт-РОтшильд" за 200-300 евро. Какое слово мы
                   4369: пропустили в предыдущем предложении?
                   4370: 
                   4371: Ответ:
                   4372: Из-под.
                   4373: 
                   4374: Зачет:
                   4375: От; использованную; выпитую; пустую; по смыслу.
                   4376: 
                   4377: Комментарий:
                   4378: 200-300 евро за бутылку такого вина - это очень дешево. Купив же пустую
                   4379: бутылку из-под элитного вина и наполнив ее любым другим, китаец может
                   4380: пустить пыль в глаза другим китайцам.
                   4381: 
                   4382: Источник:
                   4383: Журнал "GEO", 2016, N 1.
                   4384: 
                   4385: Автор:
                   4386: Анастасия Балмакова ("Одушевленные аэросани")
                   4387: 
                   4388: Вопрос 23:
                   4389: В фильме Нила Бломкампа в неком здании герой видит сувениры в виде
                   4390: одного из представителей отряда приматов. Назовите это здание.
                   4391: 
                   4392: Ответ:
                   4393: Эмпайр-стейт-билдинг.
                   4394: 
                   4395: Комментарий:
                   4396: Сражение с гигантской гориллой Кинг-Конгом в одноименном фильме
                   4397: происходило на крыше Эмпайр-стейт-билдинг.
                   4398: 
                   4399: Источник:
                   4400:    1. Х/ф "Элизиум: Рай не на Земле" (2013), реж. Нил Бломпкамп.
                   4401:    2. https://ru.wikipedia.org/wiki/Кинг-Конг_(фильм,_1933)
                   4402: 
                   4403: Автор:
                   4404: Александр Марцинкевич ("Одушевленные аэросани")
                   4405: 
                   4406: Вопрос 24:
                   4407: В вопросе есть замена.
                   4408:    По форме расторОпша похожа на зерна, но растет быстрее. Отсюда, по
                   4409: одной из версий, и происходит ее русское название. Какое слово мы
                   4410: заменили словом "расторОпша"?
                   4411: 
                   4412: Ответ:
                   4413: СпорыньЯ.
                   4414: 
                   4415: Комментарий:
                   4416: СпорыньЯ - род грибов, паразитирующий на злаках. Когда спорынья заражает
                   4417: злаки, в колосьях пораженных растений вместо зерен образуются твердые
                   4418: рОжки, по форме похожие на зерна и растущие быстрее них - "споро",
                   4419: отсюда и название. Расторопша - это тоже растение.
                   4420: 
                   4421: Источник:
                   4422: https://drugoj-m.livejournal.com/27362.html
                   4423: 
                   4424: Автор:
                   4425: Анастасия Балмакова ("Одушевленные аэросани")
                   4426: 
                   4427: Вопрос 25:
                   4428: Для того чтобы происходил фотоэффект, фотоны должны иметь достаточно
                   4429: высокую энергию. Если фотоны имеют низкую энергию, электроны перестают
                   4430: вылетать из металла, и фотоэффект прекращается. Рассказывая об этом,
                   4431: Майкл Файер прибегает к аналогии с детской игрой. Одно из названий этой
                   4432: игры совпадает с именем персонажа. Какого?
                   4433: 
                   4434: Ответ:
                   4435: Али-Баба.
                   4436: 
                   4437: Комментарий:
                   4438: В этой игре группа детей из одной команды растягивается в шеренгу,
                   4439: держась за руки. Игрок из другой команды с разбегу бросается на эту
                   4440: шеренгу и, если бежит достаточно быстро (имеет высокую энергию),
                   4441: разрывает ее и продолжает двигаться, хотя и медленнее. При несколько
                   4442: меньшей скорости он всё еще может прорвать шеренгу. Однако если он будет
                   4443: бежать достаточно медленно, то не сможет пробиться сквозь нее, поскольку
                   4444: энергии не хватит, чтобы преодолеть энергию связи в шеренге. Али-Баба -
                   4445: герой арабской сказки "Али-Баба и сорок разбойников".
                   4446: 
                   4447: Источник:
                   4448:    1. М. Файер. Абсолютный минимум. Как квантовая теория объясняет наш
                   4449: мир. http://flibusta.is/b/430636/read
                   4450:    2. https://ru.wikipedia.org/wiki/Али-Баба
                   4451: 
                   4452: Автор:
                   4453: Анастасия Балмакова ("Одушевленные аэросани")
                   4454: 
                   4455: Вопрос 26:
                   4456: Подвид одного соревнования включает выполнение таких фигур, как бабочка,
                   4457: океанская волна, карусель и обручальное кольцо. Ответьте несклоняемыми
                   4458: словами: что это за соревнование и с помощью чего выполняются эти
                   4459: фигуры?
                   4460: 
                   4461: Ответ:
                   4462: Родео, лассо.
                   4463: 
                   4464: Комментарий:
                   4465: Сегодня родео - это серьезный вид спорта, который включает целый ряд
                   4466: дисциплин, в том числе различные трюки с лассо.
                   4467: 
                   4468: Источник:
                   4469: https://www.popmech.ru/adrenalin/10393-bremya-nastoyashchikh-kovboev-russkoe-rodeo/
                   4470: 
                   4471: Автор:
                   4472: Команда "Одушевленные аэросани"
                   4473: 
                   4474: Вопрос 27:
                   4475: [Ведущему: в последнем предложении четко произнести слово "фамилии" во
                   4476: множественном числе.]
                   4477:    Рассуждая о непринятии обществом генетически модифицированных
                   4478: организмов, Александр ПАнчин пишет о необходимости появления двух тезок
                   4479: от мира фермерства и генной инженерии, которые подарили бы миру такой
                   4480: трансгенный продукт, что его захотел бы надкусить каждый. Назовите
                   4481: фамилии этих тезок.
                   4482: 
                   4483: Ответ:
                   4484: Джобс, ВОзняк.
                   4485: 
                   4486: Комментарий:
                   4487: Надкушенное яблоко - логотип компании "Apple" [эпл], основанной двумя
                   4488: Стивами - Джобсом и ВОзняком. Известна фраза Джобса о том, что "люди не
                   4489: понимают, что им на самом деле нужно, пока сам им этого не покажешь".
                   4490: Воображаемые фермер вроде Стива Джобса и генный инженер вроде Стива
                   4491: Возняка могли бы подарить миру такое трансгенное яблоко, которое захотел
                   4492: бы надкусить каждый.
                   4493: 
                   4494: Источник:
                   4495:    1. А.Ю. Панчин. Сумма биотехнологии. Руководство по борьбе с мифами о
                   4496: генетической модификации растений, животных и людей.
                   4497: http://flibusta.is/b/500145/read
                   4498:    2. https://ru.wikiquote.org/wiki/Стив_Джобс
                   4499: 
                   4500: Автор:
                   4501: Александр Круглик ("Эталон этанола")
                   4502: 
                   4503: Вопрос 28:
                   4504: Кубинская культура представляет собой смешение индейских, испанских,
                   4505: африканских и азиатских элементов. Антрополог Фернандо ОртИс сравнил
                   4506: кубинскую культуру с ахиАко. Ответьте односложным словом: что такое
                   4507: ахиАко?
                   4508: 
                   4509: Ответ:
                   4510: Суп.
                   4511: 
                   4512: Комментарий:
                   4513: ОртИс говорит, что кубинская культура подобна хорошему кубинскому супу
                   4514: ахиАко, где смешаны разнообразные традиции, обычаи и вкусы.
                   4515: 
                   4516: Источник:
                   4517: http://www.mid.ru/ru/maps/cu/-/asset_publisher/ZCoR8WfDPJng/content/id/331612
                   4518: 
                   4519: Автор:
                   4520: Команда "Одушевленные аэросани"
                   4521: 
                   4522: Вопрос 29:
                   4523: Виктору МартинОвичу движение ИХ напомнило чаинки в чашке, в которой
                   4524: только что размешали сахар. ИХ можно увидеть на картине 1890 года.
                   4525: Назовите ИХ точно.
                   4526: 
                   4527: Ответ:
                   4528: ВорОны.
                   4529: 
                   4530: Комментарий:
                   4531: Движение кружащихся в небе ворон Мартинович сравнил с движением чаинок в
                   4532: чашке. В 1890 году Винсент ван Гог написал "Пшеничное поле с ворОнами".
                   4533: 
                   4534: Источник:
                   4535:    1. В.В. Мартинович. Сфагнум. http://flibusta.is/b/328933/read
                   4536:    2. http://ru.wikipedia.org/wiki/Пшеничное_поле_с_воронами
                   4537: 
                   4538: Автор:
                   4539: Александр Марцинкевич ("Одушевленные аэросани")
                   4540: 
                   4541: Вопрос 30:
                   4542: На советской карикатуре, высмеивающей хиппи, парень и девушка смотрят на
                   4543: картину с изображением ИХ. Какой художник однажды заплатил крестьянину,
                   4544: чтобы тот не трогал ИХ?
                   4545: 
                   4546: Ответ:
                   4547: [Клод] Моне.
                   4548: 
                   4549: Комментарий:
                   4550: (pic: 20161031.jpg)
                   4551:    Карикатура высмеивает лохматые прически хиппи, которые на вид не
                   4552: слишком отличаются от изображенных на картине стогов сена. Кисти Клода
                   4553: Моне принадлежит серия из нескольких десятков картин, на которых стога
                   4554: сена изображены в разное время суток и при разной погоде. Однажды Моне
                   4555: даже заплатил крестьянину, чтобы тот не убирал сено в положенное время.
                   4556: 
                   4557: Источник:
                   4558:    1. http://modagid.ru/articles/4625
                   4559:    2. М. де Декер. Клод Моне. http://flibusta.is/b/326460/read
                   4560:    3. http://www.liveinternet.ru/users/2010239/post106324013
                   4561: 
                   4562: Автор:
                   4563: Команда "Одушевленные аэросани"
                   4564: 
                   4565: Вопрос 31:
                   4566: Сюрстрёмминг - шведский деликатес из консервированной квашеной сельди.
                   4567: Автор одной поваренной книги насмешливо писал, что на банкете
                   4568: сюрстрёмминг подадут лишь в двух случаях: если хозяин пожелает есть в
                   4569: одиночестве или если позовет гостей ПРОПУСК. А кто оказался ПРОПУСК в
                   4570: произведении XIX века?
                   4571: 
                   4572: Ответ:
                   4573: [Коллежский асессор / майор] [Платон Кузьмич] Ковалёв.
                   4574: 
                   4575: Комментарий:
                   4576: Пропущенные слова - "без носа". В процессе приготовления сюрстрёмминга
                   4577: при брожении образуется множество сильнопахнущих веществ, в том числе
                   4578: сероводород. Иногда даже в шутку советуют пробовать это блюдо, надев
                   4579: противогаз. В повести Николая Васильевича Гоголя "Нос" майор Ковалёв
                   4580: оказался на некоторое время без носа.
                   4581: 
                   4582: Источник:
                   4583:    1. https://ru.wikipedia.org/wiki/Сюрстрёмминг
                   4584:    2. https://ru.wikipedia.org/wiki/Нос_(повесть)
                   4585: 
                   4586: Автор:
                   4587: Анастасия Балмакова ("Одушевленные аэросани")
                   4588: 
                   4589: Вопрос 32:
                   4590: На памятнике в Нью-Йорке, установленном 125 лет назад, можно увидеть три
                   4591: однотипных объекта. Напишите название любого из этих объектов.
                   4592: 
                   4593: Ответ:
                   4594: "Санта-Мария".
                   4595: 
                   4596: Зачет:
                   4597: "Пинта"; "Нинья".
                   4598: 
                   4599: Комментарий:
                   4600: В год 400-летия открытия Америки (в 1892 году) был установлен памятник
                   4601: Колумбу на площади Коламбус-Сёркл. Причем это не просто памятник, а
                   4602: своего рода ростральная колонна - на ней в качестве элементов
                   4603: присутствуют носы трех кораблей Колумба. В предыдущем вопросе носа не
                   4604: было, а здесь целых три!
                   4605: 
                   4606: Источник:
                   4607: http://www.vanderkrogt.net/statues/object.php?webpage=CO&record=usny28
                   4608: 
                   4609: Автор:
                   4610: Анастасия Балмакова ("Одушевленные аэросани")
                   4611: 
                   4612: Вопрос 33:
                   4613: [Ведущему: объявить, что ответы сдаются на раздаточном материале, и
                   4614: попросить команды написать на нем игровые номера.]
                   4615:    (pic: 20161032.jpg)
                   4616:    Слева - альтернативное написание гласных искусственного языка
                   4617: волапЮк, предложенное его создателем ЙОханном ШлЕйером. Дорисуйте то,
                   4618: что мы заретушировали на раздатке.
                   4619: 
                   4620: Ответ:
                   4621: (pic: 20161033.jpg)
                   4622: 
                   4623: Комментарий:
                   4624: Язык волапЮк [Volap&uuml;k] в своем написании на волапЮке также содержит
                   4625: &uuml; [у-умлЯут]. Разработавший его немец ЙОханн ШлЕйер позаимствовал
                   4626: для нового языка умляутированные гласные из родного немецкого языка.
                   4627: Правда, альтернативное написание не прижилось и используется достаточно
                   4628: редко.
                   4629: 
                   4630: Источник:
                   4631: https://ru.wikipedia.org/wiki/Волапюк
                   4632: 
                   4633: Автор:
                   4634: Анастасия Балмакова ("Одушевленные аэросани")
                   4635: 
                   4636: Вопрос 34:
                   4637: В книге Евгения Клюева можно встретить подразделы "выступление",
                   4638: "наступление", "преступление", "исступление". Какое слово мы несколько
                   4639: раз пропустили в предыдущем предложении?
                   4640: 
                   4641: Ответ:
                   4642: Лирическое.
                   4643: 
                   4644: Комментарий:
                   4645: Писатель Евгений Клюев в своей книге "Между двух стульев" очень любит
                   4646: играть словами. По аналогии с лирическим отступлением автор делает
                   4647: "лирическими" и слова с другими приставками.
                   4648: 
                   4649: Источник:
                   4650: Е.В. Клюев. Между двух стульев. http://flibusta.is/b/67087/read
                   4651: 
                   4652: Автор:
                   4653: Анастасия Балмакова ("Одушевленные аэросани")
                   4654: 
                   4655: Вопрос 35:
                   4656: В эпоху плейстоцена остров был покрыт льдом, а когда ледник растаял,
                   4657: остров уже был отделен от других земель проливом. Так ученые объясняют
                   4658: то, что легенда приписывает ЕМУ. Назовите ЕГО.
                   4659: 
                   4660: Ответ:
                   4661: [Святой] Патрик.
                   4662: 
                   4663: Комментарий:
                   4664: Таково научное объяснение отсутствия змей в Ирландии. На льду они жить
                   4665: не могли, а потом не смогли переплыть пролив. Некоторые люди считают,
                   4666: что змей из Ирландии изгнал лично святой Патрик.
                   4667: 
                   4668: Источник:
                   4669: http://www.vokrugsveta.ru/quiz/263387/
                   4670: 
                   4671: Автор:
                   4672: Команда "Одушевленные аэросани"
                   4673: 
                   4674: Вопрос 36:
                   4675: Рассказывая о том, что трилобиты вымерли не все сразу, а, так сказать,
                   4676: "ушли со сцены" эволюции постепенно, палеонтолог Ричард ФОрти упоминает
                   4677: австрийца. Назовите этого австрийца.
                   4678: 
                   4679: Ответ:
                   4680: [Йозеф] Гайдн.
                   4681: 
                   4682: Комментарий:
                   4683: Игравшие важную роль в палеозое членистононие организмы трилобиты
                   4684: вымирали постепенно. Это был длительный процесс, а не результат некоего
                   4685: природного катаклизма или массового вымирания видов. Автор книги
                   4686: "Трилобиты: свидетели эволюции" Ричард Форти упоминает в связи с этим
                   4687: "Прощальную симфонию" Йозефа Гайдна, в финальной части которой музыканты
                   4688: постепенно, один за другим, прекращают играть, гасят свечи и покидают
                   4689: сцену. Это был последний вопрос, и мы тоже прощаемся с вами.
                   4690: 
                   4691: Источник:
                   4692:    1. Р. Форти. Трилобиты. Свидетели эволюции.
                   4693: http://flibusta.is/b/363864/read
                   4694:    2. https://ru.wikipedia.org/wiki/Симфония_%E2%84%96_45_(Гайдн)
                   4695: 
                   4696: Автор:
                   4697: Анастасия Балмакова ("Одушевленные аэросани")
                   4698: 

FreeBSD-CVSweb <freebsd-cvsweb@FreeBSD.org>